Está en la página 1de 306

Capítulo I

NÚMEROS REALES Y COMPLEJOS

INTRODUCCION.

Las fórmulas matemáticas pueden compararse con piezas de maquinaria que

trabajan sobre materias primas para obtener de éstas, productos acabados. En

este caso, tales materias primas proceden de los p-.roblemas que nos presenta el

mundo físico, usualmente en forma de números. Los productos acabados son las

soluciones a los problemas.

Los números que se emplean para este objeto son elementos de ciertos conjuntos

de números designados como sistemas numéricos. Las propiedades de estos

conjuntos son las de las materias primas. Por lo tanto es natural que se comience

este estudio con un examen amplio de las propiedades de los sistemas numéricos.

Las fórmulas son entramados matemáticos en que entran símbolos literales se

colocan o disponen las materias primas en las piezas de esta maquinaria

matemática y se reemplazan los símbolos de números. Luego las fórmulas constan

de letras y símbolos matemáticos relacionados entre sí, más bien que de números

particulares.

1
Claro está que es solamente mediante el uso de letras para representar números

no especificados que se pueden obtener resultados de algún significado general.

Al usar letras como símbolos es conveniente elegirlas de manera que sugiera cierto

orden (a, b, c,…….).

Algunas veces es conveniente usar varios conjuntos de letras para denotar ternas

de números relacionados de cierta manera. [(a, b, c), (A, B, C) (𝛼, 𝛽, 𝛾)].

Muchas proposiciones o enunciados de resultados matemáticos contiene la frase

“si” y solo si. Tales proposiciones constan realmente de dos partes distintas, y

expresan una conclusión, es decir, una consecuencia, es válida si y solamente si,

una hipótesis, o sea, una suposición, es verdadera. Esto significa, primero, que si

se hizo la suposición, la conclusión será válida y significa al decir que la

suposición es una condición suficiente. También expresa que la conclusión no

puede ser válida, sin ser verdadera la hipótesis, por lo que esta es una condición

necesaria. Luego las dos partes de la proposición, hipótesis y conclusión, pueden

llamarse condiciones equivalentes.

En estos apuntes, se harán muchas cosas con los números reales, sumarlos,

multiplicarlos, comparar sus magnitudes, etc. Para asegurarse que puede hacerse

con ellos y que no, se presentan algunas suposiciones básicas de los reales a lo

largo del capítulo, así como algunas reglas que son consecuencia de estas

suposiciones.

2
Origen del Álgebra

Los babilonios desarrollaron técnicas y métodos para medir y contar,

impulsados en parte por la necesidad de resolver problemas prácticos de

agrimensura, de intercambio comercial y del desarrollo de las técnicas

cartográficas. Entre las tablillas babilónicas descubiertas se han encontrado

ejemplos de tablas de raíces cuadradas y cúbicas, y el enunciado y solución de

varios problemas puramente algebraicos, entres ellos algunos equivalentes a lo

que hoy se conoce como una ecuación cuadrática. Un examen cuidadoso de las

tablillas babilónicas muestra claramente que mediante esos cálculos sus

autores no sólo intentaban resolver problemas del mundo real, sino otros más

abstractos y artificiales, y que lo hacían para desarrollar técnicas de solución y

ejercitarse en su aplicación.

En cuanto que, hasta la mitad del siglo XIX, el álgebra se ocupó principalmente

de resolver ecuaciones de este tipo, puede decirse que fue en Babilonia donde

tuvo su origen esta ciencia.

Fueron los árabes quienes le dieron a la nueva ciencia de plantear y resolver

ecuaciones un nombre: aljabr. La nueva civilización que surgió en la península

arábiga en la primera mitad del siglo VII, habría de transformar muy pronto la

vida de gran parte del mundo habitado de entonces. Menos de un siglo

después de la captura de La Meca por Mahoma en el año 630 d.C., el ejército

3
islámico había convertido a las tribus politeístas del Medio Oriente y usurpado

al imperio bizantino los territorios de Siria y Egipto. La conquista de Persia se

completó hacia el año 641 d.C.

Los sucesores de Mahoma, los califas, primero establecieron su capital en

Damasco pero, tras cien años de guerras, el califato se dividió en varias partes.

La fundación en 766 d.C. por parte del califa Al — Mansur de Bagdad como la

nueva capital de su califato, significó el comienzo de una etapa más tolerante

del islamismo y permitió el desarrollo intelectual de sus habitantes. Su sucesor,

el califa Harun al — Rashid, quien gobernó entre 786 y 809, estableció en

Bagdad una biblioteca en la que se reunieron manuscritos provenientes de

varias academias del Cercano Oriente, algunas de las cuales habían sido

establecidas por miembros de las antiguas academias de Atenas y Alejandría

que tuvieron que cerrarse a raíz de la persecución de los romanos.

Un programa de traducciones al árabe de textos clásicos de la matemática y

ciencia de los griegos y los hindúes era una de las actividades del Bayal al—

Iliktna (Casa de la sabiduría), un instituto de investigaciones que fundara el

califa al — Ma'mun y que funcionó durante más de 200 años.

Muhammad Musa al — Khwarizmi, un miembro del Bayal al—Hikma fue el

autor de varios tratados sobre astronomía y matemáticas, entre ellos uno de

los primeros tratados islámicos acerca del álgebra. Fue gracias a la traducción

4
al latín de su libro acerca del sistema de numeración hindú, Algorithmi de

numero indorum, que Europa Occidental conoció ese novedoso sistema de

numeración. Su obra más importante, sin embargo, fue su tratado de álgebra

que, con el título Ílisab al—/abra wal— muqabala (La ciencia de la reducción y

confrontación) probablemente significaba la ciencia de las ecuaciones.

El Álgebra de Muhammad contiene instrucciones prácticas para resolver ciertas

ecuaciones lineales y cuadráticas. “Lo que la gente quiere, dice el autor,

cuando realiza sus cálculo..., es un número”. Ese número no es más que la

solución de una ecuación.

Otro importante algebrista árabe fue Omar Khayyam (1048—1131), mejor

conocido en Occidente por su Rubaiyat, una colección de unos 600 poemas.

Fue él el primero en hacer una clasificación sistemática de las ecuaciones

cúbicas y resolver algunas de ellas.

La contribución de los algebristas islámicos de los siglos XI y XII en el

desarrollo del álgebra habría sido más notoria si no hubiera tardado tanto en

ejercer su influencia en Europa, donde, un poco después, el álgebra habría de

consolidarse definitivamente.

5
Historia del Álgebra

La historia del álgebra comenzó en el antiguo Egipto y Babilonia, donde fueron

capaces de resolver ecuaciones lineales (ax = b) y cuadráticas (ax2 + bx = c),

así como ecuaciones indeterminadas con varias incógnitas. Los antiguos

babilonios resolvían cualquier ecuación cuadrática empleando esencialmente

los mismos métodos que hoy se enseñan.

Los matemáticos alejandrinos Herón y Diofante continuaron con la tradición de

Egipto y Babilonia, aunque el libro “Las aritméticas de Diofante” es de bastante

más nivel y presenta muchas soluciones sorprendentes para ecuaciones

indeterminadas difíciles. Esta antigua sabiduría sobre resolución de ecuaciones

encontró, a su vez, acogida en el mundo islámico, en donde se la llamó

“ciencia de reducción y equilibrio”. (La palabra árabe al-jabr que significa

`reducción', es el origen de la palabra álgebra). En el siglo IX, el matemático

al-Jwarizmi escribió uno de los primeros libros árabes de álgebra, una

presentación sistemática de la teoría fundamental de ecuaciones, con ejemplos

y demostraciones incluidas. A finales del siglo IX, el matemático egipcio Abu

Kamil enunció y demostró las leyes fundamentales e identidades del álgebra.

En las civilizaciones antiguas se escribían las expresiones algebraicas utilizando

abreviaturas sólo ocasionalmente; sin embargo, en la edad media, los

matemáticos árabes fueron capaces de describir cualquier potencia de la

incógnita x, y desarrollaron el álgebra fundamental de los polinomios, aunque

sin usar los símbolos modernos.

6
Esta álgebra incluía multiplicar, dividir y extraer raíces cuadradas de

polinomios, así como el conocimiento del teorema del binomio.

El matemático, poeta y astrónomo persa Omar Khayyam mostró cómo expresar

las raíces de ecuaciones cúbicas utilizando los segmentos obtenidos por

intersección de secciones cónicas, aunque no fue capaz de encontrar una

fórmula para las raíces. La traducción al latín del Álgebra de al-Jwarizmi fue

publicada en el siglo XII. A principios del siglo XIII, el matemático italiano

Leonardo Fibonacci consiguió encontrar una aproximación cercana a la solución

de la ecuación cúbica x3 + 2x2 + cx = d. Fibonacci había viajado a países

árabes, por lo que con seguridad utilizó el método arábigo de aproximaciones

sucesivas.

A principios del siglo XVI los matemáticos italianos Scipione del Ferro, Tartaglia

y Gerolamo Cardano resolvieron la ecuación cúbica general en función de las

constantes que aparecen en la ecuación. Ludovico Ferrari, alumno de Cardano,

pronto encontró la solución exacta para la ecuación de cuarto grado y, como

consecuencia, ciertos matemáticos de los siglos posteriores intentaron

encontrar la fórmula de las raíces de las ecuaciones de quinto grado y superior.

Sin embargo, a principios del siglo XIX el matemático noruego Niels Henrik

Abel y el francés Évariste Galois demostraron la inexistencia de dicha fórmula.

Un avance importante en el álgebra fue la introducción, en el siglo XVI, de

símbolos para las incógnitas y para las operaciones y potencias algebraicas.

Debido a este avance, el Libro III de la Geometría (1637), escrito por el

7
matemático y filósofo francés René Descartes se parece bastante a un texto

moderno de álgebra. Sin embargo, la contribución más importante de

Descartes a las matemáticas fue el descubrimiento de la geometría analítica,

que reduce la resolución de problemas geométricos a la resolución de

problemas algebraicos. Su libro de geometría contiene también los

fundamentos de un curso de teoría de ecuaciones, incluyendo lo que el propio

Descartes llamó la regla de los signos para contar el número de raíces

verdaderas (positivas) y falsas (negativas) de una ecuación.

Durante el siglo XVIII se continuó trabajando en la teoría de ecuaciones y en

1799 el matemático alemán Carl Friedrich Gauss publicó la demostración de

que toda ecuación polinómica tiene al menos una raíz en el plano complejo.

En los tiempos de Gauss, el álgebra había entrado en su etapa moderna. El

foco de atención se trasladó de las ecuaciones polinómicas al estudio de la

estructura de sistemas matemáticos abstractos, cuyos axiomas estaban

basados en el comportamiento de objetos matemáticos, como los números

complejos.

Dos ejemplos de dichos sistemas son los grupos y las cuaternas, que

comparten algunas de las propiedades de los sistemas numéricos, aunque

también difieren de ellos de manera sustancial. Los grupos comenzaron como

sistemas de permutaciones y combinaciones de las raíces de polinomios, pero

8
evolucionaron para llegar a ser uno de los más importantes conceptos

unificadores de las matemáticas en el siglo XIX. Los matemáticos franceses

Galois y Augustin Cauchy, el británico Arthur Cayley y los noruegos Niels Abel y

Sophus le hicieron importantes contribuciones a su estudio. Las cuaternas

fueron descubiertas por el matemático y astrónomo irlandés William Rowan

Hamilton, quien desarrolló la aritmética de los números complejos para las

cuaternas; mientras que los números complejos son de la forma (a + bi), las

cuaternas son de la forma (a + bi + cj + dk).

Después del descubrimiento de Hamilton, el matemático alemán Hermann

Grossmann empezó a investigar los vectores. A pesar de su carácter abstracto,

el físico estadounidense J. W. Gibbs encontró en el álgebra vectorial un sistema

de gran utilidad para los físicos, del mismo modo que Hamilton había hecho

con las cuaternas.

La amplia influencia de este enfoque abstracto llevó a George Boole a escribir

Investigación sobre las leyes del pensamiento (1854), un tratamiento

algebraico de la lógica básica. Desde entonces, el álgebra moderna —también

llamada álgebra abstracta— ha seguido evolucionando; se han obtenido

resultados importantes y se le han encontrado aplicaciones en todas las ramas

de las matemáticas y en muchas otras ciencias.

9
 PERSONAJES DEL ÁLGEBRA

George Boole (1815-1864):

Recluyó la lógica a un álgebra simple. También trabajó en ecuaciones

diferenciales, el cálculo de diferencias finitas y métodos generales en probabilidad.

Boole primero concurrió a una escuela en Lincoln, luego a un colegio comercial.

Sus primeras instrucciones en matemática, sin embargo fueron de su padre quién

le dio también a George la afición para la construcción de instrumentos ópticos. El

interés de George se volvió a los idiomas y recibió instrucción en Latín de una

librería local.

A la edad de 12 años había llegado a ser tan hábil en Latín que provocaba

controversia. Él tradujo del Latín una Oda del poeta Horacio de lo cual su padre

10
estaba tan orgulloso que tenía su publicación. No obstante el talento era tal que un

maestro de escuela local cuestionaba que nadie con 12 años podría haber escrito

con tanta profundidad.

Boole no estudió para un grado académico, pero a la edad de 16 años fue un

profesor auxiliar de colegio. Él mantuvo su interés en idiomas e intentó ingresar a

la Iglesia. Desde 1835, sin embargo, pareció haber cambiado de idea ya que abrió

su propio colegio y empezó a estudiar matemáticas por sí mismo. Tardó en darse

cuenta que había perdido casi cinco años tratando de aprender las materias en vez

de tener un profesor experto.

En ese periodo Boole estudió los trabajos de Laplace y Lagrange, tomando

apuntes, los cuales llegaron a ser más tarde las bases para sus primeros papeles

matemáticos. De cualquier modo el recibió estímulos de Duncan Gregory quién se

encontraba en Cambridge por ese tiempo y del editor "Cambridge Mathematical

Formal" recientemente fundado.

Boole fue incapaz de tomar los consejos de Duncan Gregory y estudiar cursos en

Cambridge; ya que necesitaba los ingresos de su colegio para cuidar a sus padres.

No obstante él comenzó a estudiar álgebra. Una aplicación de métodos algebraicos

para la solución de ecuaciones diferenciales fue publicada por Boole en el

"Transaction of the Royal Society" y por este trabajo recibió la medalla de la Real

Sociedad. Su trabajo matemático fue el comienzo que le trajo fama.

11
Boole fue nominado para una cátedra de matemática en el Queens College, Cork

en 1849. Él enseñó allí por el resto de su vida, ganándose una reputación como un

prominente y dedicado profesor.

En el 1854 publicó Una investigación de las leyes del pensamiento sobre las cuales

son basadas las teorías matemáticas de Lógica y Probabilidad. Boole aproximó la

lógica en una nueva dirección reduciéndola a un álgebra simple, incorporando

lógica en las matemáticas.

Agudizó la analogía entre los símbolos algebraicos y aquellos que representan

formas lógicas. Comenzaba el álgebra de la lógica llamada Algebra Booleana la

cual ahora encuentra aplicación en la construcción de computadoras, circuitos

eléctricos, etc.

Boole también tradujo en ecuaciones diferenciales, el influyente "Tratado en

Ecuaciones Diferenciales" apareció en 1859, el cálculo de las diferencias finitas,

"Tratado sobre el Cálculo de las Diferencias Finitas" (1860), y métodos generales

en probabilidad. Publicó alrededor de 50 escritos y fue uno de los primeros en

investigar las propiedades básicas de los números, tales como la propiedad

distributiva que fundamento los temas del álgebra.

Muchos honores le fueron concedidos a Boole, fue reconocido como el genio en su

trabajo recibió grandes honores de las universidades de Dublín y Oxford y fue

elegido miembro académico de la Real Sociedad (1857).

12
Su trabajo fue elogiado por De Morgan quién dijo:

El sistema de lógica de Boole es una de las muchas pruebas de genio y paciencia

combinada. Esta el proceso simbólico del álgebra, inventado como herramienta de

cálculos numéricos, sería competente para expresar cada acto del pensamiento, y

proveer la gramática y el diccionario de todo el contenido de los sistemas de

lógica, no habría sido creíble hasta probarlo. Cuando Hobbes publicó su

"Computación ó Lógica" él tenía un remoto reflejo de algunos de los puntos que

han sido ubicados en la luz del día por Mr. Boole.

El álgebra Booleana tiene una amplia aplicación el switch telefónico y en el diseño

de computadoras modernas. El trabajo de Boole ha llegado a ser como un paso en

la revolución de las computadoras hoy en día.

13
NÚMEROS REALES

1.1. CONCEPTOS Y DEFINICIONES

 AXIOMAS. (POSTULADO DE PEANO)

1-.1 ∈ N
Esto afirma, que por lo menos N posee un elemento.

2-. Si n  N ∃ un número natural único llamado el siguiente de n [SIG(n)].

Esto indica que N es un conjunto infinito.

N = {1…………} ∈

3-. SIG (n) ≠ 1; ∀ n ε N; este axioma dice que el 1, es el primer elemento del
conjunto N, es decir ∉ ningún elemento antes del 1.

4-. Si SIG (n) = SIG (m) n = m, o sea que no hay 2 naturales diferentes
que tenga el mismo siguiente.

5-. Si K ⊂ N; que tiene las siguientes propiedades:

a)-. 1 ∈ K.
b)-. Si P ∈ K SIG (P) ∈ K
K=N

Definición: A cada 2 números naturales podemos asociarles un número natural por


medio de una operación llamada suma en N, cuyas propiedades son:

a)-. n + 1 = SIG (n)


b)-. n + SIG (m) = SIG (n + m)

14
DECIMALES PERIODICOS Y NO PERIODICOS

La presentación racional de un número decimal o bien es finita (como en 3/8 =


0.375000) o se repite en ciclos regulares hasta ∞ (como en 13/11 = 1.181818) un
poco de experimento por el proceso de la división le demostrara (por que solo
puede haber un número finito de residuos diferentes).
Un decimal finito puede ser considerado como uno en el que se repite ceros.
Por ejemplo: 3/8 = 0.375000…

Por lo tanto todo número racional puede escribirse como un decimal periódico. Es
un hecho notable que el inverso también es cierto. Todo decimal periódico
representa un número racional. Esto es obvio en el caso de un decimal finito (P.E.
3137/1000) y es fácil de demostrar en el caso general.

Ejemplo: (Los decimales periódicos son racionales):

Demuestre que X = 0.136136136 y Y = 0.271717 representan un número racional.

Solución: Restamos X de 1000x y después despejamos a X.

1000x = 136.136136 100 𝑦 = 27.171717


− x = 0.136136 − 𝑦 = 0.271717

999𝑥 = 136 99𝑦 = 26.9


136 26.9 269
x= 𝑦= =
999 99 990

En general nuestro primer paso consiste en multiplicar un decimal periódico Z por


10m si los decimales que se repiten en cada ciclo constan de m dígitos.
Las representaciones decimales de números irracionales no se repiten en ciclos
recíprocamente, un decimal no periódico debe representarse un número irracional.
P. E. 0.101001000100001
Debe representar un número irracional.

15
1.2. PROPIEDADES Y OPERACIONES

En este capítulo introduciremos y estudiaremos el conjunto de los axiomas y


términos indefinidos (como punto y línea) que forman la base o fundamento para
la prueba de teoremas; la utilización de estos conceptos se le conoce como
sistema deductivo o axiomático.
Clasificación de los números

NUMEROS
COMPLEJOS (Z)
(a + bi)

Números Reales (R)

Números Racionales Q Números Irracionales Q


Q = r / r = m /n;  m, n  R; n  0  Q =  r / r es no racional
Estos pueden ser decimales finitos o Estos pueden ser numéricos
infinitos periódicos. decimales, no periódicas.

Fraccionarios
Enteros (𝑍)
𝑍= r /r = m – n; m, n  N 

Entero + ( 𝑍 + ) Cero Entero Negativo 𝑍 − = r / r = m-n;


𝒁+ =N =r / r= m – n; m, n  N, m >n} m, n  N; m < n 

16
Consideraremos un conjunto de elementos llamados números reales, e
indicaremos el conjunto con R.

Aun cuando los números reales no se definan, obtendremos un conocimiento


intuitivo de su naturaleza al estudiar algunas de sus propiedades.

En el cuadro de arriba, se dan algunas de las definiciones.

1.2.1. NÚMEROS NATURALES. (𝐙 + = 𝐍)

Es el conjunto de las “r” tal que o tales que 𝑟 = 𝑚 − 𝑛 donde m, n ∈ N y m>n.

1.2.2. NUMEROS ENTEROS (Z).

Es el conjunto de las “r” tal que o tales que 𝑟 = 𝑚 − 𝑛 ; donde m, n ∈ N.

Como se puede observar el Conjunto de los números enteros es un subconjunto


infinito del sistema de los números reales R.

1.2.3. NUMEROS RACIONALES (Q)


𝑚
Es el conjunto de las “r” tal o tales que 𝑟 = ; ∀ 𝑚, 𝑛 ∈ 𝑅; 𝑛 ≠ 0 estos pueden ser
𝑛
decimales finitos o infinitos periódicos.

1.2.4. NÚMEROS IRRACIONALES (Q).

Es el conjunto de las “r” tal o tales que “r” es no racional; estos pueden ser
números decimales no periódicos.

El conjunto de los números reales, tiene dos operaciones definidas básicas


conocidas como adición y multiplicación, llamadas también suma y producto
(a+b, ab), El signo de igualdad es muy importante ya que denota la relación
propiamente dicha es decir a=b significa que a y b, son símbolos del mismo
número (VI = 6 ó 3/2 = 1.5).

17
Esta relación de igualdad se efectúa a través de los siguientes Axiomas:

1.- Propiedad reflexiva (Para ∀ a ∈ R, a = a).


2.- Propiedad simétrica (Para ∀ a, b ∈ R, si a =b; → b = a)
3.- Propiedad transitiva (Para ∀ a, b, c ∈ R, si a =b; y b = c;→ a=c)
4- Propiedad de la adición (Para ∀ a, b, c. d ∈ R, si a =b; y
c = d;→ a+c = b+d y como d=c → a+c =b+c).
5.- Propiedad de la multiplicación (Para ∀ a, b, c. d ∈ R, si a =b; y
c = d;→ ac = bd; y como d=c;→ ac = bc).
Por lo general los últimos 3 axiomas se escriben de la siguiente forma:
a) cantidades iguales a la misma cantidad son iguales entre sí.
b) Si cantidades iguales se suman a cantidades iguales, las sumas son iguales.
c) Si cantidades iguales se multiplican a cantidades iguales, los productos son
iguales.

También, tendremos otros axiomas llamados de campo para los números reales,
los cuales se representan a continuación y se plantean tanto para la suma como
para el producto:
Si x, y, z  R entonces:

1) x+y R Cerradura
x.y R
2) x+y = y+x R Conmutativa
x▫y = y▫x R
3) x + (y + z) = (x + y) + z  R Asociativa
x (y ▫ z) = (x ▫y) z R
4)  x-1 / x ▫ x-1 = 1 R Inverso
 - x / x + (-x) = 0 R
5) 1/x▫1=x R Neutro
0/x+0=x R
6) x (y + z) = x▫y + x ▫ z R Distributivo

18
Del diagrama de la clasificación de los números, se puede observar que:

1) 𝐙 + ⊂ 𝐙 ⊂ 𝐐 ⊂ 𝐑
2) 𝐐 ∩ 𝐐′ = ф; 𝐙 + ∪ 𝐙− = 𝐙
3) 𝐐 ∪ 𝐐′ = 𝐔 = 𝐑

Otras propiedades de los números reales son las de orden que se indican a
continuación.

Propiedades de orden

7) x<y
x=y Propiedad de la tricotomía.
x>y

8) Si x < y  x + z < y + z; ∀ z  R

9) Si x<y y z>0  xy < yz

10) Si x<y y z<0  x •z > y • z

11) Si x<y y y<z  x<z

12) Si x < y  -x > -y

13) Si x > 1  x2  x

14) Si 0 < x < 1  x2 < x

15) Si 0  x  y  x2  y2

16) Si 0  x  y y 0  z K  x•z  y•K

19
“Ejercicios Propuestos”

1.- A partir de la clasificación de los números reales escoja el nombre que mejor
describa a cada uno de los siguientes conjuntos de números:

a) 6, −4, √2 , 2, 7
1
b) −1, −2, 0, 3
1
c) √2, 𝜋 , , −3, 2
2
d) 4, 2, −3, 1
2 4
e) − , 3, , 0
3 6
1
f) , 10,4
3

2.- Diga cuál de las siguientes propiedades o axiomas estudiados, justifica cada
uno de los enunciados siguientes:

1
a) (x + y) =1
x+y
b) Si x + 2 = y y y=5 → x+2 =5
1
c) x ( ) = 1
x
d) – [−(x + y)] = x + y
e) Si a + c = b + c → a = b
3 a+b 3(a+b)
f) • =
4 5 4•5
g) 𝑥 + (𝑦 + 0) + [−(𝑦 + 𝑥)] = 0
h) 𝑥 + (5 + 𝑦) = (5 + 𝑥) + 𝑦
i) Si ac = bc → a = b

20
1.2.6. VALOR ABSOLUTO

Denotamos el valor absoluto de un número real “a” denotado por |𝑎|, a un número
real “a” tal que:

|a| = a Si a > 0
|a| = −a Si a < 0

Conforme a la definición anterior, el valor absoluto de todo número no nulo es


positivo y el valor absoluto de cero es igual a cero. Así

|4| = 4 |−4| = −(−4) = 4 |0| = 0

Podemos interpretar el valor absoluto de un número en forma geométrica con la


ayuda de la recta de los números reales (véase la fig.). El valor absoluto de indica
la distancia de la gráfica del número a partir del origen, aunque no especifica en
qué dirección. Por ejemplo, si |𝑥| = 2, la grafica de “x”puede estar dos unidades a
la derecha o dos unidades a la izquierda.

Es decir si

|x| < 2 Significa −2 < 𝑥 < 2

Y en general

|x| < 𝑎 Significa −𝑎 < 𝑥 < 𝑎

Observación:
El valor absoluto tiene las propiedades siguientes.

1.- |𝑥| ≥ 0 ≥ 𝑦 |𝑥| = 0 𝑆𝑜𝑙𝑜 𝑠𝑖 𝑥 = 0


2.- → |𝑥𝑦| = |𝑥||𝑦|
3. −|𝑥 + 𝑦| ≤ |𝑥| + |𝑦|

La propiedad 1 es consecuencia inmediata de la definición. La 2 se puede


demostrar con la regla de los signos.

La propiedad tres, puede consultarla en el libro de algebra superior de Humberto


Cárdenas y Francisco Raggi.

21
1.3. INDUCCIÓN MÁTEMATICA

Uno de los métodos de demostración de más importancia en matemática es lo que


se conoce como Inducción matemática. El nombre no es muy afortunado ya que
es natural asociar la palabra inducción con razonamiento inductivo, que consiste
en sacar una conclusión a partir de un gran número de casos especiales. En
realidad la inducción matemática es de naturaleza deductiva, puesto que lleva a
una conclusión segura.

La inducción matemática se utiliza generalmente para demostrar la validez de


proposiciones que incluyen todos los valores enteros positivos de n.

Ilustraremos el método mediante un ejemplo. Supongamos que tenemos una


escalera con un número indefinido de peldaños y queremos demostrar que
podemos subir hasta un peldaño determinado cualquiera. Podemos hacerlo si
conocemos dos hechos: (a) podemos subir al primer peldaño, (b) si estamos en un
peldaño cualquiera, podemos subir al superior siguiente. De (a) sabemos que
podemos subir al primer peldaño; de esto y de (b) sabemos que podemos subir al
segundo peldaño; nuevamente, de esto y de (b), sabemos que podemos subir al
tercer peldaño, etc.
Uno de los métodos para determinar raíces cuadradas con una máquina
calculadora o sumadora está basado en el hecho que la suma de los n primeros
enteros impares es n2; específicamente,

1 = 1 = 12
1 + 3 = 4 = 22
1 + 3 + 5 = 9 = 32
1 + 3 + 5 + 7 = 16 = 42

Esta propiedad es válida para todo entero positivo n y no es difícil de demostrar.

22
Ejemplo 1: Demuéstrese por inducción matemática que para todo valor entero
positivo n,
1 + 3 + 5 + ⋯ + (2n − 1) = n2

Solución: Parte(a): Verificación para un valor específico. Ya lo verificamos para


n=1, 2, 3, y 4, aunque basta la verificación para un valor solamente.

Parte (b): Propiedad de inducción. Si la proposición es verdadera para n=k donde


k es un valor arbitrario de n, debemos demostrar que también es válido para,
n=k+1. Suponiendo que la proposición vale para n=k tenemos:

1 + 3 + 5 + ⋯ + (2k − 1) = k 2 ___________________________________(A)

Ahora para n = k + 1 tenemos:

1 + 3 + 5 + ⋯ + [2(k + 1) − 1] = (k + 1)2
1 + 3 + 5 + ⋯ + [2k + 2 − 1] = (k + 1)2
1 + 3 + 5 + ⋯ + (2k + 1) = (k + 1)2 _____________________________(B)

A la ecuación (A), le sumamos el último término [(2k + 1)] de la ecuación (B)


tanto al lado derecho como al lado izquierdo, obtenemos:
1 + 3 + 5 + ⋯ + (2k − 1)(2k + 1) = k 2 + (2k + 1)
1 + 3 + 5 + ⋯ + (2k − 1)(2k + 1) = (k + 1)2 ___________________(C)

Concluimos que la parte derecha de la ecuación (B) es igual que la parte derecha
de la ecuación (C), con lo cual queda concluida la demostración.

Observamos entonces que una demostración por inducción matemática consiste de


tres partes:

23
Parte(a), Verificación de la validez de la proposición o teorema para el menor valor
de n para el cual el teorema debe valer (esto es análogo al, hecho de poder subir
el primer peldaño de la escalera).
Parte (b) demostración de la propiedad inductiva. Si la proposición o teorema es
válida para n=k, donde k designa un valor cualquiera de n, entonces vale para
n=k+1 (esto es equivalente a subir de un peldaño cualquiera al superior
siguiente.).

Parte(c). Conclusión. La proposición o teorema vale para todo valor de n igual o


mayor que aquel para el cual se verifico en la parte (a).

Ejemplo 2: Utilizando el método de inducción matemática demuéstrese que la


proposición siguiente es válida para todo valor entero positivo de n.
n(n + 1)
1 + 2 + 3 + ⋯+ n =
2
Demostración:

Para n = 1, 2, 3, 4
1=1
2(3)
1+2= 3= =3
2
3(4)
1+2+3= 6= =6
2
4(5)
1 + 2 + 3 + 4 = 10 = = 10
2

Para n=k

k(k + 1)
1 + 2 + 3 + ⋯+ k = _____________________________________(A)
2

Para n=k+1

(k + 1)[(k + 1) + 1]
1 + 2 + 3 + ⋯ + (k + 1) =
2

(k + 1)(k + 2)
1 + 2 + 3 + ⋯ + (k + 1) = ______________________(B)
2

24
Agregando (k+1) a ambos miembros de la ecuación (A) se tiene

k(k + 1)
1 + 2 + 3 + ⋯ + k + (k + 1) = + [(k + 1)]
2

[k(k + 1) + 2(k + 1)]


1 + 2 + 3 + ⋯ + k + (k + 1) =
2
(𝑘+1)(𝑘+2)
1 + 2 + 3 + ⋯ + 𝑘 + (𝑘 + 1) = ________________________(C)
2

El lado derecho de la ecuación (C) = Lado derecho de la ecuación (B), lo cual


queda demostrado que es válido para cualquier valor de n la proposición.

25
1.4. NÚMEROS COMPLEJOS
(Z)

Con el objeto de despojar a los números complejos de su aspecto “irreal” o


“complejo” con el que se fueron abriendo paso en las matemáticas a través de la
historia tratamos este tema.

En este caso trataremos las raíces pares, especialmente las raíces cuadradas de
números negativos que son importantes en las matemáticas. Su uso ha contribuido
al desarrollo de una gran parte de las matemáticas, parte importante de las cuales
tienen aplicaciones vitales en ingeniería, y en las ciencias físicas. Numerosas
ecuaciones y muchos problemas no se pueden investigar en el sistema de los
números reales. La ecuación 𝑥 2 + 1 = 0 por ejemplo posee una solución si solo si
𝑥 2 = −1. Pero el cuadrado de cualquier número real no es negativo y, en
consecuencia no tiene solución real. Con el objeto de resolver esta ecuación y
algunas otras más complicadas, necesitamos aumentar nuestro sistema numérico.

Así un número complejo se puede escribir de tres maneras diferentes que son:

1.- Rectangular o Binómica.


2.- Polar ó Trigonométrica.
3.- Exponencial.

26
1.4.1. FORMA RECTANGULAR O BINOMICA.

Esta se puede escribir de la siguiente manera:

(𝑥 + 𝑦𝑖)

Donde:

x =Numero real

yi =Numero imaginario

i =Base de los números imaginarios = √−1

De tal manera que si se quiere resolver la ecuación 𝑥 2 + 4 = 0 se procedería de la


siguiente manera:
𝑥2 + 4 = 0
𝑥 = ±√−4 = ±√4√−1 = ±2√−1
𝑥 = ±2𝑖

De lo anterior, se puede deducir, en base a la definición del número complejo, la


siguiente tabla, que nos permitirá escribir cualquier número en la forma
Rectangular o Binómica.

TABLA:
i2 = (√−1)2 = −1
i3 = i2 i = −1i = −i
i4 = i3 i = i2 i2 = (−1)(−1) = −ii = −i2 = −(−1) = 1
i5 = i4 i = i
i6 = i5 i = ii = i2 = −1

Se deduce, de las expresiones planteadas arriba, que: Cuando “i” esta elevado a
un número par nos da como resultado 1 o -1, y cuando esta elevado a un número
impar da i o –i.
Entonces en general in = (i)n-1(i).

27
1.4.2. OPERACIONES BÁSICAS
Conjugado. (𝒁
̅)

Se define el conjugado de un numero complejo, de la forma Z=x +yi, como aquel


número, en el cual solamente se le cambia de signo a la parte imaginaria de dicho
número, Donde Zc se le denominará como Z conjugada.

Zc =x-yi.

Producto.

z1 z2 = (x1 + y1 i)(x2 + y2 i) = (x1 x2 − y1 y2 ) + (x1 y2 + x2 y1 )i

División.

𝑧1 (𝑥1 + 𝑦1 𝑖)(𝑥2 − 𝑦2 𝑖) (𝑥1 𝑥2 + 𝑦1 𝑦2 ) + (𝑥2 𝑦1 − 𝑥1 𝑦2 )𝑖


= =
𝑧2 (𝑥2 + 𝑦2 𝑖)(𝑥2 − 𝑦2 𝑖) x22 + y22

Nota: Para efectuar la operación anterior, deberá de multiplicarse tanto el


numerador como el denominador, por el conjugado del denominador, tal como se
muestra. Esto se realiza, para poder obtener en el denominador solamente una
parte real, y así poder escribir la operación de división en forma Binómica.

Exponenciación.

z1n = (x1 + y1 i)(x1 + y1 i)(x1 + y1 i) = z1 z1 z1 z1 …

Valor absoluto.

|𝑧1 − 𝑧2 | = √(𝑥1 − 𝑥2 )2 + (𝑦1 − 𝑦2 )2

Que es la distancia entre los dos puntos 𝑧1 𝑦 𝑧2 .

28
Propiedades

Suma y Multiplicación

𝐒𝐢 𝐙𝟏 , 𝐙𝟐 , 𝐙𝟑 , ∈ ∁

1.- 𝑍1 + 𝑍2 𝜖 ∁
𝑍1 𝑍2

2.- Z1 + Z2 = Z2 + Z1 ∈ ∁
Z1 Z2 = Z2 Z1

3.-Z1 (Z2 + Z3 ) = Z1 Z2 + Z1 Z3 ∈ ∁

4.- Z1 + (Z2 + Z3 ) = (Z1 + Z2 ) + Z3 ∈ ∁


Z1 (Z2 Z3 ) = (Z1 Z2 )Z3

5.- ∃ − 𝑍1 / 𝑍1 + (−𝑍1 ) = 0 + 0i
∃ Z1 −1 ≠ 0 / 𝑍1 (𝑍 −1 ) = 1 + 0i

6.- ∃ 𝒁𝟏 = 𝟎 + 𝟎𝒊 / (𝟎 + 𝟎𝒊) + 𝑍1 = Z1

∃ 𝒁𝟏 = (1 + 0i) / 𝑍1 (1 + 0i) = Z1

Propiedades para los conjugados

7.- 𝑍1 𝑐𝑐 = 𝑍1̅
8.- (𝑍1 + 𝑍2 )𝑐 = 𝑍1 𝑐 + 𝑍2𝑐
9.- (𝒁𝒄𝟏 𝒁𝒄𝟐 ) = (𝒁𝟏 𝒁𝟐 )𝒄
10.- 𝒁𝒄𝟏 = 𝒁𝟏 ↔ 𝒁𝟏 ∈ 𝑹
11.- 𝒁𝒄𝟏 𝒁𝟏 ∈ 𝑹
12.- 𝒁𝒄𝟏 + 𝒁𝟏 ∈ 𝑹

EJEMPLO:

29
1.- Realizar la siguiente operación de números complejos y obtener lo que se pide.
Z1 = −i
𝑍2 = 3
𝑍3 = (√2 + 𝑖)
𝑍4 = (2 + 3𝑖)

Hallar

Z2 𝐳𝟏𝐜 𝐳𝟐 𝒛𝟑 −𝒛𝟒
𝐚) − Z3 , 𝐛) 𝐜) + 𝒛𝒄𝟑 𝒛𝒄𝟒
Z1 𝐳𝟏 𝐳𝟑 𝒛𝟑

Solución:

z2 3
𝒂) − z3 = − (√2 + i) = −√2 + 2i
z1 −i

𝒛𝒄𝟏𝒁𝟐 𝒊(𝟑) 𝟑(√𝟐 − 𝒊)


𝒃) = =− = −√𝟐 + 𝒊
𝒛𝟏 𝒛𝟑 −𝒊(√𝟐 + 𝒊) (√𝟐 + 𝒊)(√𝟐 − 𝒊)

𝑧3 − 𝑧4 (√2 + 𝑖) − (2 + 3𝑖)
𝑐) + 𝑧3𝑐 𝑧4𝑐 = + (√2 − 𝑖)(2 − 3𝑖)
𝑧3 √2 + 𝑖

√2 + 𝑖 − 2 − 3𝑖 −2 + √2 − 2𝑖 √2 − 𝑖 −2√2 + 2 − 3√2 𝑖
= ° = + (√2 − 𝑖)(2 − 3𝑖)
√2 + 𝑖 √2 + 𝑖 √2 − 𝑖 3

−9 + 4√2 − 6 − 12√2𝑖 −3.3431 − 22.97056𝑖


= = −1.1143 − 7.6568𝑖
3 3

1.5. Forma Polar o Trigonométrica.

30
Como ya sabemos, un número complejo, viene dado por z= x + yi; por lo tanto
podemos representarlo en un plano complejo de la siguiente manera:

Del diagrama se ve por el Teorema de Pitagoras


𝑟 = √𝑥 2 + 𝑦 2
Y por Trigonometría
𝑦
𝑠𝑒𝑛 𝜃 = → 𝑦 = 𝑟 𝑠𝑒𝑛 𝜃
𝑟
𝑥 I
cos 𝜃 = → 𝑥 = 𝑟 cos 𝜃
𝑟
Pero como 𝑧 = 𝑥 + 𝑦𝑖 r
y
→ 𝑧 = 𝑟(cos 𝜃 + 𝑖 𝑠𝑒𝑛 𝜃) = 𝑟 𝐶𝑖𝑠 𝜃 ______(1) 
𝑦 x
𝑑𝑜𝑛𝑑𝑒 𝜃 = 𝐴𝑟𝑐 𝑡𝑎𝑛𝑔
𝑥

Donde la ecuación (1) se le conoce como la forma polar o Trigonométrica de un


número complejo.

Así también, se le conoce a “r” como el modulo, y a “Ə” como el argumento.


Debe notarse que “Cis=Cos +i Sen”, se le considera un operador, es decir una
forma de simplificación para escribir un número complejo en forma polar.

Algunas veces se quiere escribir los valores de algunas funciones trigonométricas


como una razón de valores numéricos, es por eso que a continuación se presenta
una forma de hallar esta relación, aquí solamente se presentan para ángulos de
30, 45 y 60 grados a partir del triangulo equilátero y de un cuadrado unitario.

31
2 2
30º
3
√2

1
60 º
1 2 1 45º

1 1 √2 √2
𝑆𝑒𝑛 30° = 𝑆𝑒𝑛 45° = ° =
2 √2 √2 2
√3 √2
𝐶𝑜𝑠 30° = 𝑆𝑒𝑛 45° =
2 2
1 √3 √3 √2
𝑇𝑎𝑛𝑔 30° = ° = 𝐶𝑜𝑠 45° =
√3 √3 3 2
√3
𝑇𝑎𝑛𝑔 30° = 𝑡𝑎𝑛𝑔 45° = 1
3

√3
𝑆𝑒𝑛 60° =
2
1
𝐶𝑜𝑠 60° =
2
𝑇𝑎𝑛𝑔 60° = √3

Para cualquier número complejo 𝑍 ≠ 0, corresponde solamente un valor de Ə en


0 ≤ θ ≤ 2π ; no obstante cualquier otro intervalo de longitud 2π, por ejemplo
−π < 𝜃 ≤ π se puede emplear.

32
Ejemplos:
1.-Obtener la forma polar de los siguientes números complejos:

Z1 = 2 − 2i → 2.8284 Cis − 45°


𝑍2 = −3 → 3 𝐶𝑖𝑠 180°
𝑍3 = 5𝑖 → 5 𝐶𝑖𝑠 90°
𝑍4 = −2𝑖 → 2 𝐶𝑖𝑠 − 90°

2.- Escribir el número complejo siguiente en forma polar trigonométrica

𝑍 = 1+𝑖
𝑟 = √12 + 12
𝑟 = √2
𝜃 = 𝑡𝑔−1 (1)
𝜃 = 45° Z = √2 Cis 45° (Forma Polar)

3.- Dado el número complejo siguiente, representarlo en forma polar, a partir de


la forma polar encontrada, regresarlo a la forma Binómica, y dibujar un diagrama
del vector en un plano complejo.

a)
I
𝐙=𝟏−𝐢 𝐙 = √𝟐 𝐂𝐢𝐬 𝟑𝟏𝟓°
𝐫 = √(𝟏)𝟐 + (𝟏)𝟐 𝐙 = √𝟐 (𝐂𝐨𝐬 𝟑𝟏𝟓° + 𝐢 𝐬𝐞𝐧 𝟑𝟏𝟓°)
√𝟐 √𝟐
𝐫 = √𝟐 𝐙 = √𝟐 ( − 𝐢)
𝟐 𝟐
x
𝛉 = 𝐭𝐠 −𝟏
𝛉 = −𝟒𝟓° + 𝟑𝟔𝟎° 𝐙=𝟏−𝐢 -45º
𝛉 = 𝟑𝟏𝟓°

𝐙 = √𝟐 𝐂𝐢𝐬 𝟑𝟏𝟓°

33
b)

𝐙 = −𝟏 + 𝐢 𝐙 = √𝟐 𝐂𝐢𝐬 𝟏𝟑𝟓° I
𝐙 = √𝟐 𝐙 = √𝟐 (𝐂𝐨𝐬 𝟏𝟑𝟓° + 𝐢 𝐒𝐞𝐧 𝟏𝟑𝟓°)
√𝟐 √𝟐 135º
𝛉 = 𝐭𝐠 −𝟏 𝐙 = √𝟐 (− + 𝐢)
𝟐 𝟐 45º
𝛉 = −𝟒𝟓° + 𝟏𝟖𝟎° 𝐙 = −𝟏 + 𝐢 X
𝛉 = 𝟏𝟑𝟓° 𝐙 = −𝟏 + 𝐢
𝐙 = √𝟐 𝐂𝐢𝐬 𝟏𝟑𝟓°

c)

𝐙 = −𝐢 𝐙 = 𝐂𝐢𝐬 𝟐𝟕𝟎°
𝐫=𝟏 𝐙 = (𝐂𝐨𝐬 𝟐𝟕𝟎° − 𝐢 𝐒𝐞𝐧 𝟐𝟕𝟎°)
−𝟏
𝛉= = 𝐙 = 𝟏 (𝟎 − 𝟏)
𝟎
𝛉 = 𝟐𝟕𝟎° 𝐙 = −𝐢
𝐙 = 𝐂𝐢𝐬 𝟐𝟕𝟎°

i
d)
𝐙 = −𝟏 𝐙 = 𝐂𝐢𝐬 𝟏𝟖𝟎° 180º
𝐫=𝟏 𝐙 = 𝟏( 𝐂𝐨𝐬 𝟏𝟖𝟎° + 𝐢 𝐒𝐞𝐧 𝟏𝟖𝟎°)
𝟎
𝛉 = 𝐭𝐠 −𝟏 𝟏 𝐙 = −𝟏 + 𝟎 X
𝛉 = 𝟏𝟖𝟎° 𝐙 = −𝟏 -1
𝐙 = 𝐂𝐢𝐬 𝟏𝟖𝟎°

e)

𝐙𝟏 = 𝟐 𝐂𝐢𝐬 𝟔𝟎°

𝐙𝟏 = 𝟐(𝐂𝐨𝐬 𝟔𝟎° + 𝐢 𝐒𝐞𝐧 𝟔𝟎°)


𝟏 √𝟑
𝐙𝟏 = 𝟐( + 𝐢)
𝟐 𝟐
𝐙𝟏 = 𝟏 + √𝟑𝐢

𝐫 𝐂𝐢𝐬 (𝟐𝛑 + 𝛉) = 𝐫 𝐂𝐢𝐬 𝛉

f)

𝐙𝟐 = 𝟐 𝐂𝐢𝐬 𝟒𝟐𝟎°
𝐙𝟐 = 𝟐( 𝐂𝐨𝐬 𝟒𝟐𝟎° + 𝐢 𝐒𝐞𝐧 𝟒𝟐𝟎°)
𝟏 √𝟑
𝐙𝟐 = 𝟐( + 𝐢)
𝟐 𝟐
𝒁𝟐 = 𝟏 + √𝟑𝒊
34
1.5.1. Operaciones Básicas.

Nota: Las operaciones de suma y resta por esta forma no están definidas, hay
que regresar a la forma rectangular, para poder efectuarlas.

 Multiplicación

𝐒𝐢 𝐙𝟏 = 𝐫𝟏 𝐂𝐢𝐬 Ѳ𝟏 𝐲 𝐙𝟐 = 𝐫𝟐 𝐂𝐢𝐬 Ѳ𝟐
→ Z1 Z2 = (r1 Cis Ѳ1 )(r2 Cis Ѳ2 )
Z1 Z2 = r1 r2 Cis (Ѳ1 + Ѳ2 )

 División

𝐙𝟏 = 𝐫𝟏 𝐂𝐢𝐬 Ѳ𝟏
𝐙𝟐 = 𝐫𝟐 𝐂𝐢𝐬 Ѳ𝟐
𝒁 𝟏 𝒓𝟏
= 𝒄𝒊𝒔 (Ѳ1 − Ѳ2 )
𝒁 𝟐 𝒓𝟐

 Potenciación

𝐙𝐧 = (𝐫𝟏 𝐂𝐢𝐬 Ѳ)𝐧


𝐙𝐧 = 𝐫𝟏 𝐧 𝐂𝐢𝐬 (𝐧Ѳ)

 Radicalización
𝒏 𝟏 𝟏
√𝒛 = 𝒛𝒏 = ( 𝒓 𝑪𝒊𝒔 𝜽)𝒏
𝒏 𝟏 𝜽
√𝒛 = 𝒓𝒏 = 𝑪𝒊𝒔
𝒏
𝒏 𝟏 𝜽 𝟐𝝅𝒌
√𝒛 = 𝒓𝒏 = 𝑪𝒊𝒔( + )
𝒏 𝒏

𝒅𝒐𝒏𝒅𝒆 𝒌 = 𝟎, 𝟏, 𝟐, 𝟑, … 𝒏 − 𝟏

35
Ejemplos:

1.- Hallar las operaciones indicadas, de los números complejos siguientes:


𝐒𝐞𝐚 𝐙𝟏 = √𝟐 𝐂𝐢𝐬 𝟕𝟎° 𝐲 𝐙𝟐 = 𝟑 𝐂𝐢𝐬 𝟐𝟐𝟓°

a.- 𝒁𝟏 𝟐 = (√𝟐 𝑪𝒊𝒔 𝟕𝟎°)𝟐 b.- 𝐙𝟐 𝟑 = (𝟑 𝐂𝐢𝐬 𝟐𝟐𝟓°)


= (√𝟐)𝟐 𝑪𝒊𝒔 𝟐(𝟕𝟎°) = (𝟑)𝟑 𝐂𝐢𝐬 𝟑(𝟐𝟐𝟓°)
𝐙𝟏 𝟐 = 𝟐 𝐂𝐢𝐬 𝟏𝟒𝟎° 𝟑
𝐙𝟐 = 𝟐𝟕 𝐂𝐢𝐬 𝟔𝟕𝟓°

c.- 𝒁𝟐 𝟒 = (𝟑 𝑪𝒊𝒔 𝟐𝟐𝟓°)𝟒 d.- 𝐙𝟏 + 𝐙𝟐 = (√𝟐 𝐂𝐢𝐬 𝟕𝟎°) + (𝟑 𝐂𝐢𝐬 𝟐𝟐𝟓°)


= (3)4 𝐶𝑖𝑠 4(225°) Z1 = √2 (Cos 70° + i Sen 70°)
𝐙𝟒 = 𝟖𝟏 𝐂𝐢𝐬 𝟗𝟎𝟎° Z1 = √2( 0.3420 + 0.9396 i)
𝑍2 = 3 𝐶𝑖𝑠 225°
Z2 = 3(Cos 225° + i Sen 225°)
√2 √2
𝑍2 = 3(− − 𝑖)
2 2
3√2 3√2
Z2 = − − i
2 2

𝑍1 + 𝑍2 = 0.4836 + 1.3287i − 2.1213 − 2.1213i


𝑍1 + 𝑍2 = −1.6377 − 0.7926𝑖

e.- Z1 Z2 = (√2 Cis 70°)(3 Cis 225°)


Z1 Z2 = 3√2 Cis (225° + 70°)
Z1 Z2 = 3√2 Cis 295°

f.- 𝐙𝟏 = √𝟐 𝐂𝐢𝐬 𝟕𝟎° 𝐙𝟏 = √𝟐 𝐂𝐢𝐬 𝟕𝟎°


𝐙𝟐 = 𝟑 𝐂𝐢𝐬 𝟐𝟐𝟓° 𝐙𝟐 = 𝟑 𝐂𝐢𝐬 𝟐𝟐𝟓°
𝒁𝟏 √𝟐
𝐂𝐢𝐬 (𝛉 + 𝟐𝛑) = 𝐂𝐢𝐬 𝛉 = 𝑪𝒊𝒔 (𝟕𝟎° − 𝟐𝟐𝟓°)
𝒁𝟐 𝟑
𝒁𝟏 √𝟐
𝐂𝐢𝐬 (𝟕𝟎° + 𝟑𝟔𝟎°) = 𝐂𝐢𝐬 𝛉 𝒁𝟐
= 𝟑
𝑪𝒊𝒔 (−𝟏𝟓𝟓°)
𝐞𝐧𝐭𝐨𝐧𝐜𝐞𝐬 𝐂𝐢𝐬𝟒𝟑𝟎° = 𝐂𝐢𝐬𝟕𝟎° Es decir que :

√𝟐 √𝟐
𝐂𝐢𝐬 𝟐𝟎𝟓 = 𝐂𝐢𝐬 (−𝟏𝟓𝟓°) = −𝟎. 𝟒𝟐𝟕𝟐𝟑 − 𝟎. 𝟏𝟗𝟗𝟐𝐢
𝟑 𝟑

36
2.- 𝐒𝐞𝐚 𝐳𝟏 = √𝟐 𝐂𝐢𝐬 𝟗𝟎° 𝐳𝟐 = 𝟑 + 𝟑√𝟑 𝐢
𝐳𝟑 = 𝟐 𝐂𝐢𝐬 𝟔𝟎° 𝐳𝟒 = −𝟐

Obtener la cuarta potencia de:

4 4
  2Cis 60º (3  3 3 i 
 z 3  z 2

 c
 z1    
 2

 2Cis 90 
 z 4     

a. - Z3 +Z2
Z3 + Z2 = (2 Cis 60°) + (3 + 3√3 i); Nota: Z3 = 2 Cis 60°
𝑍3 + 𝑍2 = (1 + √3𝑖) + (3 + 3√3𝑖) Z3 = 2(Cos 60° + i Sen 60°)
1 √3
𝑍3 + 𝑍2 = 1 + √3𝑖 + 3 + 3√3𝑖 Z3 = 2( + i)
2 2
𝑍3 + 𝑍2 = 4 + 4√3𝑖 Z3 = 1 + √3i

𝑧3 +𝑧2
b.- [ ]z
̅1 𝑁𝑜𝑡𝑎: 𝑍1 = √2 𝐶𝑖𝑠 90°
𝑧4
𝑍1 = √2 (𝐶𝑜𝑠 90° + 𝑖 𝑠𝑒𝑛 90°)
(4+4√3𝑖)
= 𝑍1 = √2(0 + 𝑖)
−2
−2 − 2√3𝑖 𝑍1 = √2 𝑖
𝑧 +𝑧 𝑐
[ 3 2] 𝑧̅1 = (−2 − 2√3 𝑖)(√2 𝐶𝑖𝑠 90°) 𝑍 1 = −√2 𝑖
𝑧4
= (−2 − 2√3 𝑖)(−√2𝑖)
= 2√2 𝑖 + 2√6 𝑖 2
= −2√6 + 2√2 𝑖

4
  z3  z 2  

 
 z1    2 6  2 2 i 4

  z4  

Pasando −2√6 + 2√2 i a la forma polar tendremos que:

r = √(−2√6)2 + (2√2)2 = √24 + 8 = √32 = √16 √2 = 4√2 = 5.6568


2√2 √2
θ = tg −1 = tg −1 (− ) = −30 + 180 = 150
−2√6 √6

37
Por lo tanto

  z3  z 2  
4
i
  z1  = (4√2 𝐶𝑖𝑠 150°) =
4

 z 4   150º
= (4√2)4 𝐶𝑖𝑠 4(150°) X
= 1024 𝐶𝑖𝑠 600° 30º
= 1024 𝐶𝑖𝑠 (240° + 360°)
= 1024 𝐶𝑖𝑠 240°

4
  z3  z 2  
  z1 

  z 4   = 1024 𝐶𝑖𝑠 240°
Diagrama del ángulo
L.Q.Q.D.

̅̅̅
z1 +z z1
C.- 2
+ z3 z2 −
z4 z3

(√2 Cis 90°) + (3 − 3√3 i) √2 Cis 90°


+ (2 Cis 60°)(3 + 3√3 i) −
−2 2 Cis 60°

Nota: √2 Cis 90° = √2 (Cos 90° + i Sen 90°) = √2 (0 + i) = √2 i

2 Cis 60° = 2 (Cos 60° + i Sen 60°) = 1 + √3 i

(√2 i) + (3 − 3√3 i) √2
+ (2 Cis 60°)(3 + 3√3 i) − Cis 30°
−2 2
√2 i + 3 − 3√3 i √2
+ (2 Cis 60°)(3 + 3√3 i) − Cis 30°
−2 2

Transformando a (3 + 3√3 𝑖) a la forma polar; tendremos:

r = √32 + (3√3)2 = √36 = 6


3√3
θ = Tang −1 ( ) = 60
3
3 − 3.7819 i √2 √3 1
= + (2 Cis 60°)(6 Cis 60°) − ( + i)
−2 2 2 2
3 − 3.7819 𝑖 √6 √2 3 − 3.7819 𝑖 1 √3 √6 √2
= + 12 𝐶𝑖𝑠 120 − − 𝑖= + 12 (− + 𝑖) − − 𝑖
−2 4 4 −2 2 2 4 4
−6+7.5638 𝑖−24+24√3 𝑖−√6−√2 𝑖
= = −8.1124 + 11.9297𝑖
4

38
Entonces:
̅2
Z1 +Z Z1 y
+ Z3 Z2 − = −8.1124 + 11.9297 i L.Q.Q.D
Z4 Z3
335º

x
-45º

3.- Dado el número complejo en forma polar z = 8 Cis 210°


3
Hallar la √z
Solución:
Formula:
n n θ 2πk θ 2πk
√z = √r Cis ( + ) = rn1 Cis ( + )
n n n n

Para k=0 (primera raíz)


210 2πk
813 Cis ( + )
3 3
= 𝟐 𝐂𝐢𝐬 𝟕𝟎°

Para k=1 (segunda raíz)


210 360
= 2 Cis ( + )
3 3
= 𝟐 𝐂𝐢𝐬 𝟏𝟗𝟎°

Para k=2 (tercera raíz)


210 (360)2
= 2 Cis ( + )
3 3
= 𝟐 𝐂𝐢𝐬 𝟑𝟏𝟎°

39
4.-Resolver la ecuación, x3 + 8 = 0 y hallar las tres raíces, tanto en forma polar.
𝑥 3 = −8 Formula
1
3 𝑛 𝜃 2𝑘𝜋
𝑥 = √−8 √𝑧 = 𝑧 𝑛 𝐶𝑖𝑠 (𝑛 + )
𝑛

𝑟=8 𝜃 = 180°
𝑧 = 8 𝐶𝑖𝑠 180°

Si k = 0
1 180 360(0) 1 √3
x0 = 83 Cis ( + ) = 2 Cis 60° = 2 (Cos 60° + i Sen 60°) = 2 ( + i)
3 3 2 2

1 √3
= 2( + 𝑖)
2 2

𝑥0 = 1 + √3𝑖

Ahora si k=1
1 180 360
x1 = 83 Cis ( + ) = x1 = 2 Cis 180° → = 2 (Cos 180 + i Sen 180)
3 3
x1 = 2(−1 + 0) = −2

Para k=2
1 180 (360)2 1 √3
𝑥2 = 83 𝐶𝑖𝑠 ( + ) = 2 𝐶𝑖𝑠 300° = 2(𝐶𝑜𝑠 300° + 𝑖 𝑆𝑒𝑛 300°) = 2 ( − )
3 3 2 2

𝑥2 = 1 − √3𝑖

5.- Resolver la ecuación 𝒁𝟒 − 𝟏 + 𝒊 = 𝟎


Solución:
𝑍4 − 1 + 𝑖 = 0
𝑍4 = 1 − 𝑖
4
𝑍 = √1 − 𝑖

40
Pero
1 − i = √2 Cis (−45°) = √2 Cis 315°
r = √2
θ = −45°
4
Z = √√2 cis 315°

1 𝜃 2𝑘𝜋
𝑛
√𝑧 = 𝑧 𝑛 𝐶𝑖𝑠 (𝑛 + 𝑛
)

Para k=0, usando la fórmula para sacar raíz en forma polar, obtenemos:

1 1 1
315° 2kπ 315° 2kπ
Z0 = (√2)4 Cis ( + ) = 28 Cis ( + ) = 28 Cis 78.75°
4 4 4 4

Para k=1
1 315° 360° 1
Z1 = 28 Cis ( + ) = 28 Cis 168.75°
4 4

Para k=2
1 315° (360°)(2) 1
Z2 = 28 Cis ( + ) = 28 Cis 258.75°
4 4

Para k=3
1 315° (360°)(3) 1
Z3 = 28 Cis ( + ) = 28 Cis 348.75°
4 4

6.- Obtener Z  C tal que:


1
4𝑧 = 2𝑧̅ + (√3 − 𝑖)6 ( 𝑐𝑖𝑠 30°)
8
Suponemos que z = a + bi
(√3 − i)6 = (2 Cis 30°)6 = 26 Cis 6(30°) = 64 Cis 180°

41
Entonces:
1
4𝑧 − 2𝑧 𝑐 = 4(𝑎 + 𝑏𝑖) − 2(𝑎 − 𝑏𝑖) = (√3 − 𝑖)6 ( 𝑐𝑖𝑠 30°)
8
1
= 2a + 6bi = (64 Cis 180°)( Cis 30°)
8
= 2a + 6bi = 8 Cis 210°
= 2a + 6bi = 8(Cos 210° + i Sen 210°)
√3 1
= 2a + 6bi = 8 (− − i)
2 2

Por lo tanto por el principio de la igualdad


−4√3
2a = −4√3 → a = → a = −2√3
2
4 2
6b = −4 → b = − → b = −
6 3
Así es que finalmente
2
z = −2√3 − i
3

42
1.6. FORMA EXPONENCIAL.

Como ya habíamos visto, hay 3 formas de representar un número complejo, y esta


es la última que trataremos, ya que veremos, que con esta forma podemos hallar
los logaritmos naturales de estos.

Los complejos, de lo que se han visto se pueden escribir como Z= a+bi=r Cis θ y
como se dijo “Cis” es un operador, si cambiamos ese operador por la letra “e” de
los logaritmos naturales, entonces z se puede escribir como:

Z = r eiθ

Que sería la forma exponencial de Z

Deducción:

Si 𝑍 = 𝑟𝑒 𝑖𝜃 y w = a + bi
Entonces, de la definición que dice que ew = Z, tendremos que sustituyendo a w y
z. lo siguiente:

𝑒 𝑎+𝑏𝑖 = 𝑟𝑒 𝑖𝜃
𝑒 𝑎 . 𝑒 𝑏𝑖 = 𝑟𝑒 𝑖𝜃
→ 𝑒 𝑎 = 𝑟 𝑦 𝑒 𝑏𝑖 = 𝑒 𝑖𝜃

Por definición: Es decir de la ecuación (𝑒 𝑎 = 𝑟) le sacamos logaritmo natural a


ambos miembros, y a la ecuación (𝑒 𝑏𝑖 = 𝑒 𝑖𝜃 ) igualamos sus exponentes,
tendremos lo siguiente:

𝐿𝑛𝑒 𝑒 𝑎 = 𝐿𝑛(𝑟) 𝑦 𝑏=𝜃


𝑎 = 𝐿𝑛(𝑟)
Como Z = ew
Aplicando el logaritmo a ambos miembros tenemos Ln (z) = w=a+bi
Por lo tanto:

Ln(z) = Ln(r) + (θ + 2πk) donde k = 0,1,2,3 … n − 1

En el cual “n” es el número de logaritmos que se quiere obtener.

43
También, podemos obtener la ecuación para calcular el logaritmo natural de un
número complejo a partir de que, si consideramos que 𝑧 = 𝑟𝑒 𝑖𝜃 , aplicando la
propiedad de los logaritmos a ambos miembros, tendríamos:

ln z = ln(r eiθ ) = ln(r) + Ln(eiθ ) = ln(r) + (θ + 2πk) k = 0,1,2, … . . n − 1

Nota:
Ln(AB) = Ln A + Ln B
Entonces
Ln(z) = Ln(r eiθ ) = Ln(r) + L(eiθ )
Ln(z) = Ln(r) + (θ + 2πk)i k = 0,1,2, … . . n − 1 (1)

Expresiones de los números complejos en forma exponencial, que se utilizan para


efectuar las operaciones básicas.

Sea z1 = r1 eiθ1 y z2 = r2 eiθ2

1.6.1. Operaciones Básicas.

1) SUMA Y RESTA

La suma y la resta de números complejos en esta forma, no se puede efectuar,


hay que regresar a la forma Binómica y efectuarla desde ahí.

a) PRODUCTO
𝑧1 𝑧2 = (𝑟1 𝑒 𝑖𝜃1 )(𝑟2 𝑒 𝑖𝜃2 ) = 𝑟1 𝑟2 𝑒 𝑖(𝜃1 +𝜃2 )
b) DIVISIÓN

𝑧1 (𝑟1 𝑒 𝑖𝜃1 ) 𝑟1 𝑖(𝜃 −𝜃 )


= = 𝑒 1 2
𝑧2 (𝑟2 𝑒 𝑖𝜃2 ) 𝑟2

1) RADICALIZACIÓN.

1 1 𝜃 2𝜋𝑘
𝑛 𝑛 𝑖( + )𝑖
√𝑧 = √𝑟𝑒 𝑖𝜃 = (𝑟𝑒 𝑖𝜃 )𝑛 = 𝑟 𝑛 𝑒 𝑛 𝑛

2) LOGARITMO NATURAL.
Ln(z) = Ln(reiθ ) = Ln(r) + (θ + 2πk)i (1)

44
Ejemplo:

1.- Hallar los logaritmos naturales de las expresiones que se muestran:

πi c
Si z1 = 2eπi z2 = 5e 2 z3 = √2 𝑒 2πi z4 = √3 Cis 60° y z 5 = −1 − 2i

a) Los dos primeros logaritmos de 𝑧1


𝑐
b) un logaritmo de 𝑧 5
c) Los dos primeros logaritmos de la operación.

z2
− z4
z3

3
d) El logaritmo de 𝑧, de la ecuación z1 − z4 = z2c + z 2 (z3 z1 )2

Solución:

a) Como piden los dos primeros logaritmos, eso quiere decir, que de la
ecuación (1) k debe tomar los valores de 0 y 1, (es decir n=2) por lo tanto
tendremos:

Ln(z1 ) = Ln(2eπi) = Ln(2) + (θ + 2πk)i

Primer logaritmo para k =0


Ln(z0 ) = 0.6931 + [π + 2π(0)]i = 0.6931 + 3.1416i

Segundo logaritmo ahora k=1

Ln(z1 ) = 0.6931 + [π + 2π(1)]i = 0.6931 + (3.1416 + 6.2832)i


= 0.6931 + (9.4248)i

b)
2.0344
Ln(z5c ) = Ln(−1 − 2i) = Ln(−1 + 2i) = Ln (√5e ) = Ln(√5 ) + (2.0344 + 2πk)i

45
Como no se puede sumar grados con radianes en el último término de la expresión
anterior, debe de cambiarse los 63.47 grados a radianes, usando la relación
siguiente:

π radianes x radianes
=
180 grados 63.47 grados

Por lo tanto

π(63.47)
x radianes = = 0.3526π = 0.3526(3.1416) = 1.1077 radianes
180
De lo anterior se deduce, que como piden un solo logaritmo (Que también se
llama el principal) entonces, el único valor que tomara “k” es cero.
Así es que la respuesta se escribe como se muestra a continuación:

Ln(z5c ) = 0.8047 + [−1.1077 + 2π(0)]i = 0.8047 − 1.1077i

c)
π
i π 2π
z2 5e 2 5
Ln ( − z4 ) = Ln ( 2π − √3 Cis 60) = Ln ( e( 2 − 3 )i ) − (√3 Cis 60)
z3 i √2
√2 e 3

3π−4π
5 i
Ln ( e 6 − √3 Cis 60)
√2

π
Ln (3.5355 e− 6 i − √3 Cis 60) = Ln [3.5355 Cis (−30) − √3 Cis 60]
Ln [3.5355 (0.8660 − 0.5i) − 1.7320(0.5 + 0.866i)] = Ln(2.1957 − 3.2676 i)
Ln(3.9367e−56.2764 i )
Ln(3.9367) + (−56.2764 + 2πk)i = 1.3703 + (−0.3126π + 2π)i
= 1.3703 + 1.6874πi = 1.3703 + 5.3011i

𝑧2
𝐿𝑛 ( − 𝑧4 ) = 1.3703 + 5.3011𝑖
𝑧3

46
Nota: Para realizar los cambios de rectangular (Rec) a polar (Pol) o de polar a
rectangular, se puede usar una calculadora (Puede ser la Casio fx-82MS con la fx--
350ES se obtiene en forma más directa) de la siguiente manera:

1.- Se oprime la tecla Rec. (Pol.) dependiendo del cambio que se quiere realizar.

2.- Se introduce los datos en el paréntesis que muestra el display, primero el


modulo una coma y después el argumento o Angulo y cerrar el paréntesis (o si es
polar primero la parte real del numero complejo una coma y después la parte
imaginaria).

3.- Se oprime igual o enter y se obtiene el primer resultado que sería la parte real
y después se teclea “RCL” y “TAN” y se obtendrá la parte imaginaria resultante, a
esta última se la asocia la tecla “i” para obtener el resultado total (si es Pol. Se
obtendría el modulo y después de oprimir “RCL” y “TAN” el argumento o Angulo).

47
48
1.7. Ejercicios Resueltos:

1.-Demostrar que las operaciones con números reales pueden efectuarse


considerándolos como binomios; es decir, que considerándolos como binomios se
obtienen los mismos resultados que con las expresiones II. 1.3 y las formulas de
uso práctico que se emplean para la sustracción y la división.

a) Z1 +0+0i
a+bi+0+0= Z1
a+bi= Z1

b) (a+bi) (1+0i)
a+bi+0ai+0bi2 = Z1
a+bi= Z1

c) Z1 (Z2 + Z3) = Z1 Z2 + Z1 Z3
a+bi [(c+e)+ (d+f) i]
a (c+e) +a (d+f) i +b (c+e) i+ b(d+f)i2
ac + ae + adi + afi +bci+bei+bdi2 +bfi2
a+bi (c+di) + a+bi (e+fi)
Z (Z2) + Z1 (Z3)

2.- Demostrar que para todo Z1 ε c

Si 𝑍1 = 𝑎 + 𝑏𝑖 → 𝑍1̅ = 𝑎 − 𝑏𝑖
Pero como R(Z1 ) = 0
→ Z1 = bi y Z̅1 = −bi
→ Z1 = −Z̅1 → L. Q. Q. D.

49
3.- Z1 ▫ Ẑ1 = ε R

(a+bi)(a-bi)

a2 +abi-abi-(bi)2

a2 +b2 ε R

4.- Determina las soluciones de la ecuación

a) Z4 + α4 = 0 Para cualquier α ∈ R
𝑍 = −𝛼 4
4

4 4
√𝑍 4 = √−𝛼 4
4 4
𝑍 = √𝛼 4 ▫ √−1
4 4
𝑍 = √𝛼 4 ▫ √1 𝐶𝑖𝑠 180° → √α4 Cis 180°
4

1 1
Z = r ncis n(θ+2πk)

Para k=0
1
Z0 = α Cis (180°)
4
Z0 = α Cis (45°)

Para k=1
1
Z1 = α Cis [180° + (2π)(1)]
4
Z1 = α Cis (135°)

Para k=2
1
Z2 = α Cis [180° + (2π)(2)]
4
Z2 = α Cis (225°)

Para k=3
1
Z3 = α Cis [180° + (2π)(3)]
4
Z3 = α Cis (315°)

50
b.- Determinar los valores de 𝛂 𝛆 𝐑 para los que, Z1=1+i y Z2 = 1-i, son soluciones
de la ecuación, y obtener las otras ecuaciones.

P Z1 1+i
→α= →α=
Z1 Cis 135° Cis 135°

√2 Cis 45°
α= → α = √2 Cis − 90 → α = −√2i
Cis 135°

P Z2 1−i
→α= →α=
Z2 Cis 225° Cis 225°

√2 Cis − 45°
α= → α = √2 Cis − 270 → α = √2 i
Cis 225°

5.- Obtener la forma Binómica de los siguientes números complejos.

Z1= Cis 150° = -.8660+ 0.5i


Z2= 4 Cis 210° = -3.4641 -2i
Z3 = 2√2 Cis 375° = 2.7320 + 0.7320i

6.-Escribir el siguiente número, dado en forma polar, a la forma Binómica ó


rectangular:

Z = √2 Cis 45°
Z = √2(Cos 45° + i Sen 45°)
√2 √2
Z = √2( + i)
2 2
𝐙=𝟏+𝐢

7.- Calcular la forma binómica:

(1 − i) − (3 + i) 1−i−3−i −2 − 2i
= →
2 Cis 120° 2 Cis 120° 2 Cis 120°

2.8284 Cis − 135° −1.9999 − 1.9999𝑖 −2 − 2𝑖


= → =
2 Cis 120° −1 + √3𝑖 −1 + √3𝑖

51
(−2 − 2i) (−1 − √3i)
= •
(−1 + √3i) (−1 − √3i)
2 + 2√3𝑖 + 2𝑖 + 2√3 𝑖 2
=
1+3
2 − 2√3 (2 + 2√3)𝑖
= +
4 4
1 − √3 1 + √3
+ 𝑖
2 2
= 𝑎 + 𝑏𝑖

8.-Calcular la forma polar del siguiente ejercicio.


2 𝑐𝑖𝑠 60°
(1 − 𝑖)4 ( )
−√3 + 𝑖
2 cis 60°
[1.4142 Cis (−45°)]4 ▫
2 cis 150°

(4 Cis − 180°)(1 Cis − 90°) → 4 Cis − 270°

9.- Diga cuál de las siguientes propiedades o axiomas estudiados, justifica cada
uno de los enunciados siguientes.

1
a) (x + y) = 1 − − − − − − − − − − − − − − − − − Inverso multiplicativo
x+y

b) Si x + 2 = y y = 5 → x + 2 = 5 − − − − − − − − − Transitiva

1
c) x ( ) = 1 − − − − − − − − − − − − − − − − − − − Inverso multiplicativo
x

d) – [−(x + y)] = x + y − − − − − − − − − − − − − − − 2 veces Distributiva

e) Si a + c = b + c → a = b − − − − − − − − − − − − − −Transitiva

3 a+b 3(a+b)
f) • = − − − − − − − − − − − − − − − − − − − Asociativa
4 5 4(5)

g) x + (y + 0) + [−(y + x)] = 0 − − − − − − − − − − − Neutro aditivo(y + 0)


Asociativa (x + y)
Conmutativa para la adicion − (x + y)
El inverso aditivo. (x + y) + [−(x + y)] = 0
h) x + (5 + y) = (5 + x) + y − − − − − −Distributiva x + 5 + y = (5 + x) + y
Asociativa (x + 5) + y = (5 + x) + y
Conmutativa para la adición (5 + x) + y = (5 + x) + y

52
i) Si ac = bc → a = b − − − − − − − − − − − − − − − − − Multiplicativa

10.- Indique la(s) propiedades que se usaron en el siguiente desarrollo de las


ecuaciones que se muestran.

(x + y) + (a + b) = [(x + y) + a] + b − − − − − Distributiva (2) y asociativa (2)

𝑂 𝑙𝑎 𝑐𝑜𝑛𝑚𝑢𝑡𝑎𝑡𝑖𝑣𝑎 𝑝𝑎𝑟𝑎 𝑙𝑎 𝑎𝑑𝑖𝑐𝑖ó𝑛 𝑦 𝑙𝑢𝑒𝑔𝑜 𝑙𝑎 𝑎𝑠𝑜𝑐𝑖𝑎𝑡𝑖𝑣𝑎

(𝑥 + 𝑦) + (𝑎 + 𝑏) = [𝑎 + (𝑥 + 𝑦) + 𝑏]
Distributiva y despues asociativa
(x + y) + (a + b) = [(a + x + y) + b]
Distributiva y asociativa
(𝑥 + 𝑦) + (𝑎 + 𝑏) = (𝑎 + 𝑥) + (𝑦 + 𝑏)

11.- Obtener todos los valores X, y ε R que se satisfagan las siguientes


igualdades:
𝐱+𝐲𝐢
a) = 𝐱 + 𝐲𝐢
𝐱−𝐲𝐢

x + yi = (x + yi)(x − yi)
x + yi = x 2 − xyi + xyi − yi2
x + yi = (x 2 + y 2 ) + 0i → no hay imaginario

Entonces y=0
Por lo tanto
𝑥 = 𝑥2
1=x →x=1

12.- Resuelva la siguiente ecuación y halle el valor (es) de 𝑧̅; las tres primeras
raíces.

𝑖 + 𝑧̅ 𝑒 2𝑖 + 𝑧
+2 = +𝑧
2 𝐶𝑖𝑠 45° 2

𝑖 + 𝑧̅ + 4 𝐶𝑖𝑠 45° 𝑒 2𝑖 + 𝑧 + 2𝑧
=
2 𝐶𝑖𝑠 45° 2

2(i + z̅ + 4 cis 45°) = (e2i + z + 2z)(2 cis 45°)

2(i + z̅ + 4 cis 45°) = (e2i + 3z)(2 cis 45°)

2i + 2z̅ + 8 cis 45° = 2e2i cis 45° + 6z cis 45°

53
2z̅ − 6zcis 45° = (2e2i cis 45°) − (8 cis 45°) − 2i

2[z̅ − 3z cis 45°] = 2[e2i cis 45° − 4 cis 45° − i]

2z̅ − (√2 + √2 i)(3z) = (2 cis 45°)(1 cis 114.60) − (5.6568 + 5.6568 i) − 2i

2(𝑎 − 𝑏𝑖) − √2 + √2 𝑖)(3(𝑎 + 𝑏𝑖)) = (2 𝑐𝑖𝑠 159.60°) − 5.6568 − 5.6568 𝑖 − 2𝑖

2𝑎 − 2𝑏𝑖 − (√2 + √2 𝑖)(3𝑎 + 3𝑏𝑖) = (2 𝑐𝑖𝑠 159.60°) − 5.6568 − 7.6568 𝑖

2𝑎 − 2𝑏𝑖 − (3𝑎√2 + 3𝑏𝑖√2 + 3𝑎 √2 𝑖 + 3𝑏𝑖√2 𝑖 2 ) = −1.8745 + 0.6971𝑖 − 5.6568 − 7.6568𝑖

2𝑎 − 2𝑏𝑖 − 3𝑎√2 − 3𝑎 √2 𝑖 − 3𝑏𝑖 √2 + 3𝑏 √2 = −7.5313 − 6.9597 𝑖

(2a − 3a √2 ) − (2bi + 3bi √2 + 3a √2i − 3b√2 = −7.5313 − 6.9597 i

(2a − 4.2426 a) − (2bi + 4.2426 bi) − 3a √2i + 3b √2 = −7.5313 − 6.9597 i

−2.2426 a − 6.2426 bi − 4.2426ai + 4.2426 b = −7.5313 − 6.9597 i

(−2.2426 a + 4.2426 b) − (6.2426 b + 4.2426a)i = −7.5313 − 6.9597i

−2.2426a + 4.2426b = −7.5313 − − − − − −ec. 1

−6.2426b − 4.2426a = −6.9597 − − − − − −ec. 2

Despejamos “a” en la ecuación 1

−7.5313 − 4.2426b
a=
−2.2426
𝑎 = 3.3583 + 1.8918𝑏 − − − − − − − 𝑒𝑐. 3

El valor de “a” lo sustituimos en la ecuación 2

−6.2426b − 4.2426a = −6.9597 − − − − − −ec. 2

−6.2426b − 4.2426(3.3583 + 1.8918b) = −6.9597

−6.2426b − 14.2479 − 8.0261b = −6.9597

−14.2687 𝑏 = −6.9597 + 14.2479

−14.2687𝑏 = 7.2882

Despejamos a “b”

54
7.2882
b=
−14.2687
𝑏 = −0.5107

Sustituimos el valor de “b” en la ecuación 3

𝑎 = 3.3583 + 1.8918𝑏 − − − − − − − 𝑒𝑐. 3

𝑎 = 3.3583 + 1.8918(−0.5107)

𝑎 = 2.3921

𝑧 = 2.3921 − 0.5107 𝑖

𝑧̅ = 2.3921 + 0.5107 𝑖

1° raíz para k=0


3
√2.3921 − 0.5107𝑖
3
√2.4460 𝑐𝑖𝑠 − 12.0514°

Z = r n cis n(ѳ + 2πk)

3 1
𝑍 = √(2.4460)1 𝐶𝑖𝑠 [−12.0514° + 2𝜋(0)]
3
𝑍 = 1.3473 𝐶𝑖𝑠 − 4.01713

2° raíz para k=1

3 1
𝑍 = √(2.4460)1 𝐶𝑖𝑠 [−12.0514° + 2π(1)]
3

3 1
𝑍 = √(2.4460)1 𝐶𝑖𝑠 [−12.0514° + 360°]
3
Z = 1.3473 Cis 115.9828°

3° raíz para k=2

3 1
𝑍 = √(2.4460)1 𝐶𝑖𝑠 [−12.0514° + 2π(2)]
3
𝑍 = 1.3473 cis 235.9828°

55
13.- Halle las raíces de z en la ecuación siguiente:
𝜋
2𝑧 2 − (1 − 𝑖 )3 𝑍 + 2𝑒 3 𝑖

Identificamos las variables a, b, y c para poder resolverlo como una cuadrática


𝜋
𝐴 = 2𝑖 𝐵 = −(1 − 𝑖)3 𝐶 = 2𝑒 3 𝑖

𝐴 = 2 𝑐𝑖𝑠 90° 𝐵 = −[√2 𝑐𝑖𝑠( −45°)]3 = −(√8 𝑐𝑖𝑠 − 135°) 𝐶 = 2 𝑐𝑖𝑠 60°

−𝑏 ± √𝑏 2 − 4𝑎𝑐
𝑥=
2𝑎

[√8 cis (−135°)] ± √[−√8 cis (−135°)]2 − 4(2 cis 90°)(2 cis 60°)
2(2 cis 90°)

[√8 cis(−135°)] ± √8 cis (−270°) − 4(4 cis 150°)


2(2 cis 90°)

[√8 cis (−135°)] ± √8 cis (−270°) − (16 cis 150°)


2(2 cis 90°)

[√8 cis (−135°)] ± √0 + 8i − (−13.8564 + 8i)


2(2 cis 90°)

[√8 cis (−135°)] ± √0 + 8i + 13.8564 − 8i)


4 cis 90°
[√8 cis (−135°)] ± √13.8564
4 cis 90°
[√8 cis (−135°)] ± 3.7224
4 cis 90°
−2 − 2𝑖 ± 3.7224
4𝑖
Para la raíz primera

−2 − 2i + 3.7224 1.7224 − 2i 2.6394 cis (−49.2649°)


= =
4i 4i 4 cis 90°

56
= 0.6598 cis [(−49.2649°)(−90°)]

R1 = 0.6598 cis (−139.2649°)

Para la raíz segunda

−2 − 2i − 3.7224 −5.7224 − 2i 6.0618 cis (−160.7352°)


= =
4i 4i 4 cis 90°
= 1.5154 cis[ (−160.7352°)(−90°)]

R 2 = 1.5154 cis (−250.7352)

14.- Resuelva la siguiente ecuación:

−2e−πi + z 1 − 3Z̅
+ Z̅ = + 2i
2 cis 45° 2i

−2e−πi + z + 2Z̅ cis 45° 1 − 3Z̅ + 4i2


=
2 cis 45° 2i

2i(−2e−πi + z + 2Z̅ cis 45°) = (1 − 3Z̅ + 4i2 )(2 cis 45°)

2i(2 + z + 2Z̅ cis 45°) = (−3Z̅ − 3)(2 cis 45°)

4i + 2zi + 4Z̅i cis 45° = −6 cis 45° − 6Z̅ cis 45°

2zi + 4Z̅i cis 45° + 6 Z̅cis 45° = −6 cis 45° − 4i

2[zi + z̅ Cis 45°(3 + 2i)] = −6 Cis 45° − 4i

2[(a + bi)i + (a − bi)(1.4142 + 1.4142i)(3 + 2i)] = −6(1.4142 + 1.4142i) − 4i

2[ai − b + 1.4142a + 7.071b + 7.071ai − 1.4142bi] = −8.4852 − 8.4852i − 4i

2[1.4142a + 6.0716b + 8.071ai − 1.4142bi] = −8.4852 − 12.4852i

1.4142𝑎 + 6.0716𝑏 = −4.2426

8.071𝑎 − 1.4142𝑏 = −6.2426

−4.2426 − 6.071𝑏
𝑎= = −3 − 4.2928𝑏
1.4142
8.071(−3 − 4.2928𝑏) − 1.4142𝑏 = −6.2426

−24.213 − 34.647𝑏 − 1.4142𝑏 = −6.2426

57
−36.06𝑏 = 17.9704

17.9704
𝑏=
−36.06

𝑏 = −0.4983

𝑎 = −3 − 4.2928(−0.4983)

𝑎 = −0.8608

𝑍 = −0.8608 − 0.4983𝑖 𝑦 𝑍 = −0.8608 + 0.4983𝑖

15.- Encontrar el valor de Z

2 cis 30° = 2i Z − (2 − i)2 Z2

(2 − i)2 Z2 − 2iZ + 2 cis 30° = 0

Identificamos las variables A, B Y C, y sustituimos en la ecuación de la formula general


del binomio.

A = (2 − i)2 = [2.2360 cis (−26.5660)]2 = [5 cis (−53.132)] = (3 − 4i)

−b ± √b 2 − 4ac
x=
2a

2i ± √4i2 − 4[5 cis (−53.132°)](2 cis 30°)


x=
2[5 cis (−53.132°)]

2i ± √−4 − 4[10 cis (−23.132°)]


x=
[10 cis (−53.132°)]

2i √−4 − [40 cis (−23.132°)]


x=
[10 cis (−53.132°)]

2i ± √−4 − (36.7840 − 15.7140i)


x=
[10 cis (−53.132°)]

2i ± √−4 − 36.7840 + 15.7140i


x=
[10 cis (−53.132°)]

2i ± √−40.784 + 15.7140i
x=
[10 cis (−53.132°)]

58
2i ± √43.7065 cis 158.9284°
x=
[10 cis (−53.132°)]

2i ± (6.6110 cis 79.4642°)


x=
[10 cis (−53.132°)]

2i ± (1.2088 + 6.50i)
x=
[10 cis (−53.132°)]

1.2088 + 8.50i
x1 =
[10 cis (−53.132°)]

1.2088 + 8.50i
x1 =
[10 cis (−53.132°)]

(8.5855 cis 81.9061°)


x1 = = 0.8585 cis 135.038 = −0.6074 + 0.6066i
[10 cis (−53.132°)]
−1.2088 − 4.50i +[4.6595 cis (+74.9640°)]
x2 = = = 0.4659cis (−51.9039°) = 0.2874 − 0.3666i
[10 cis (−53.132°)] [10 cis (−53.132°)]

16.-Efectuar las siguientes operaciones

a) 1 − 𝑒 −𝜋𝑖 = 1 − (1 𝑐𝑖𝑠 − 180°)

1 − (−1 + 0𝑖) = 1 + 1 + 0𝑖 = 2

π
i
1−e 2 1−(1 cis 90°) 1−(0+1i) 1−i √2 Cis−45°
b) π = = = = = Cis − 90° = −i
1+e 2
i 1+(1 cis 90°) 1+(0+1i) 1+i √2 Cis 45°

c) i + e2πi = i + (1 cis 360°)

𝑖 + (1 + 0𝑖) = 1 + 𝑖 = √2 𝐶𝑖𝑠 45°

59
17.- Obtener todos los valores de X y ε R tales que:
𝐚) x + yi = xeyi x sen y = y
= x[cos y + i sen y] x sen 0 = 0
= x cos y + xi sen y xεR
x cos y = x
y = cos −1 1 = 0
y=0

𝐛) ex+yi = −1
exyi = cis 180° = eπi
x + yi = πi
x=0 y=π

60
CAPITULO II
Polinomios

Introducción.
En muchos problemas de la física y la matemática nos encontramos con
expresiones de la forma P(x) = a0 x0 + a1 x1 + a2 x 2 + … + an x n los cuales se
conocen como polinomios.

Los polinomios pueden ser tratados formalmente desde dos puntos de vista.

El primero considera un polinomio como una expresión, mientras que el segundo lo


considera un polinomio como una función. Tanto la función como la expresión son
de la forma siguiente.

En el primer caso, es decir, al considerar al polinomio como una expresión, los


símbolos 𝑥 0 , 𝑥1 , 𝑥 2 , … 𝑥 𝑛 son indicadores de la posición de los números
𝑎0 , 𝑎1 , 𝑎2 , … 𝑎𝑛 y los signos (+) deben interpretarse únicamente como medios de
conexión. En cambio cuando el polinomio es considerado como una función, los
símbolos 𝑥 0 , 𝑥1 , 𝑥 2 , … 𝑥 𝑛 representan potencias de la variable x;
𝑎𝑥 0 , 𝑎1 𝑥1 , 𝑎2 𝑥 2 , … 𝑎𝑛 𝑥 𝑛 representan productos, y los signos (+) se interpretan
como símbolos de adición.

De acuerdo con el primer punto de vista deben definirse conceptos tales como
igualdad, adición, y multiplicación de polinomios, mientras que de acuerdo con el
segundo, dichos conceptos están determinados por la igualdad de funciones la
adición y la multiplicación de números.

En este capítulo consideramos los polinomios como expresiones, lo que es más


acorde con un tratamiento algebraico de los mismos, aunque tendremos siempre
en cuenta que los polinomios pueden también ser considerados como funciones,
por lo que se hará hincapié en que las definiciones sean consistentes con este otro
enfoque.

Definición:

Un polinomio, es un arreglo de términos, ligados por un signo más (+) ó menos


(-), es decir este puede tener la forma que se muestra a continuación:
P(x) = an x n + an−1 x n−1 + an−2 x n−2 + ⋯ + a0

61
En donde los coeficientes a0 , a1 , a2 , … an−1 , an−2 an son elementos de los reales y
a0 ≠ 0 .
De lo anterior, podemos decir que el polinomio es de coeficientes reales o pueden
ser todos racionales (o enteros), y diremos entonces que el polinomio tiene
coeficientes racionales (o enteros). Pueden también considerarse polinomios cuyos
coeficientes pertenecen a alguna estructura algebraica distinta de los complejos.
Haremos varias observaciones:

a) Cuando a1 = 0 se conviene, en que se puede omitir el termino 𝑎1 𝑥1 al


escribir el polinomio, siempre que no se omitan todos los términos. Así, los
polinomios

2 + 3𝑥 + 0𝑥 2 + 5𝑥 2
0 + 2𝑥 + 0𝑥 2
0 + 0𝑥

Pueden escribirse, respectivamente como


2 + 3𝑥 + 5𝑥 2
2𝑥
0

b) se conviene en escribir 𝑥1 en lugar de 1x1 y −ax, en lugar de (−a)x1 o


de +(−a)x1 . Así el polinomio.
(−1)x + 0x + 1x 3 + (−2)x 4 .
Se escribe
−x + x 3 − 2x 4

c) el término a0 también puede escribirse como a0 x 0 nos referimos al término


ai x i como al término de grado i. Al término de grado cero le llamamos
término independiente. Al polinomio cero le llamamos polinomio nulo.

d) No es necesarios escribir los términos de un polinomio siempre en el mismo


orden. Tampoco es necesario denotar siempre con a i el coeficiente del
termino de grado i. Por ejemplo, el polinomio a + bx + cx 2 puede escribirse
también bx + cx 2 + a; a + cx 2 + bx, etc., pero la forma más usual, y
utilizada, es ax 2 + bx + c, es decir en orden decreciente de los grados.

e) Otra manera de denotar los polinomios consiste en escribir la sucesión de


sus coeficientes en orden creciente, sin omitir ninguno, conviniendo en que
todos los términos que no aparecen tiene coeficiente cero. Con esta

62
convención los polinomio −x 2 + 2x 5 ; 0, x 2 + ax + b, 5, se representa así (0,0,-
1,0,2,0,0..); (0,0,0,…); (b,a,1,0,0..) Y (5, 0,0,…)

La siguiente noción es muy importante:

Definición: el grado de un polinomio no nulo es el mayor de los grados de los


términos que tienen coeficiente diferente de cero.

Según esto los grados de los polinomios.

2 − 3x 2 , x + 0x 4 , − 1 + ix + (1 + i)x 2 + 0x 3 , x 3 + bx 2 + cx + d

Son respectivamente, 2, 1, 2, 3, y 0.

2.1. DEFINICIÓN DE IGUALDAD.

Se dice que dos polinomios son iguales, si y solo si, los términos semejantes
correspondientes son iguales.

Esto es, si tenemos un polinomio P1 (x) = a1 x n + a2 x n−1 + a3 x n−2 + ⋯ + a0


P2 (x) = b1 x n + b2 x n−1 + b3 x n−2 + ⋯ + b0 .

Entonces P1 (x) = P2 (x)

Cuando

a1 x n = b1 x n
a2 x n−1 = b2 x n−1
a3 x n−2 = b3 x n−2


a0 = b0

63
2.2. PROPIEDADES DE LAS OPERACIONES
FUNDAMENTALES.

Suma y productos de polinomios

Supongamos el polinomio.
P(x) = an x n + an−1 x n−1 + an−2 x n−2 + ⋯ + a0

Como si tuviera una infinidad de términos, conviniendo en que a partir del grado n
+ 1 todos los coeficientes son cero;

Por ejemplo:
2 − 5x = 2 − 5x 2 + 0x 3 + 0x 4 + 0x 5 … … … ….
a0 + a1 x + ⋯ an x n = a0 + a1 x … … donde ai = 0 para i > 𝑛

Con esta convención es muy fácil definir la suma del producto:

Definición de suma:
(a0 + a1 x + ⋯ ) + (b0 + b1 x + ⋯ ) =
(𝑎0 + 𝑏0 ) + (𝑎1 + 𝑏1 )𝑥 + (𝑎2 + 𝑏2 )𝑥 2 + ⋯ +

Definición del producto:


(a0 + a1 x + ⋯ ) + (… b0 + b1 x + ⋯ ) =
= a0 b0 + (a0 b1 + a1 b0 )x + (a0 b2 + a1 b1 + a2 b0 )x 2 + ⋯

El coeficiente de 𝑥 𝑛 en la suma es an + bn y en el producto es

∑ ai bj
i+j=n

Es importante observar que la suma y el producto puede obtenerse manejando a x


como si fuera un número y aplicando las reglas usuales de las operaciones con
números complejos que son de la conmutatividad y asociatividad de la suma y
productos y la distributividad. Por ejemplo para calcular el producto tendremos que
multiplicar cada término del primer factor por cada uno de los del segundo
obteniéndose productos.

64
ai bj x i+j

Podemos entonces observar que para un exponente fijo “n” aparece un termino de
grado “n” cada vez que, i + j = n y así vemos que el coeficiente de x n , es la
suma de todos los productos ai bj . Para los cuales i + j = n tal como aparece en la
forma que define el producto. Una discusión parecida puede hacerse para la suma.

Resumiendo las observaciones anteriores podemos plantear lo siguiente:


1.- Sean g(x) = f1 (x) + f2 (x) y h(x) = f1 (x)f2 (x) para toda alfa, elementos de los
complejos (𝛼 𝜀 𝐶). Se cumple:

g(α) = f1 (α) + f2 (α)


h(α) = f1 (α)f2 (α)

2.- El grado de la suma de los dos polinomios no nulos, es menor o igual que el
máximo de los grados de los sumandos.

3.- El grado del producto de dos polinomios no nulos es la suma de los grados de
los factores.

65
2.3. DIVISIÓN.

2.3.1 Algoritmo de la división para polinomios

Ahora trataremos la división entre los polinomios primeramente definiremos lo que


entenderemos como factor.

Definición 1:
Sean f(x) y g(x) dos polinomios “x” con coeficiente en los complejos, donde
g(x) ≠ 0.

g(x) Es un factor de f(x) si existe un polinomio q(x) con coeficientes en los


complejos tal que:

f(x) = g(x)q(x) + r(x)

Se dice entonces que 𝑓(𝑥) es divisible entre 𝑔(𝑥).

TEOREMA (algoritmos de la división)

Sean 𝑓(𝑥) 𝑦 𝑔(𝑥) dos polinomios en “x” con coeficientes en los complejos. Si
𝑔(𝑥) ≠ 0, existen dos polinomios únicos 𝑞(𝑥) 𝑦 𝑟(𝑥) con coeficientes en los
complejos tales que:

𝑓(𝑥) = 𝑔(𝑥)𝑞(𝑥) + 𝑟(𝑥) 𝑔𝑟(𝑟) < 𝑔𝑟(𝑔) 𝑜 𝑟(𝑥) = 0

Otra forma de explicar lo anterior sería:

Sea 𝑓(𝑥) cualquier polinomio y sea 𝑔(𝑥), un polinomio no nulo. Existen dos únicos
polinomios, 𝑞(𝑥) 𝑦 𝑟(𝑥) que satisfacen las condiciones siguientes:

𝑎) 𝑓(𝑥) = 𝑔(𝑥)𝑞(𝑥) + 𝑟(𝑥) 𝑏) 𝑔𝑟(𝑟) < 𝑔𝑟(𝑔)

Donde “gr” es grado de.

Los polinomios 𝑞(𝑥) 𝑦 𝑟(𝑥) son el cociente y el residuo de la división de 𝑓(𝑥)


entre 𝑔(𝑥), respectivamente. Donde los polinomios 𝑓(𝑥) 𝑦 𝑔(𝑥) son el dividendo y
el divisor.

66
Otra forma de plantear el algoritmo sería:

Para dividir un polinomio entre un monomio, necesitamos usar la ley de


exponentes.

Teorema:

a m  n si m  n
a m

n
 1 si m  n
a  1
 mn si m  n
a

Ahora para dividir polinomios, en los cuales el divisor tiene dos o más términos. El
procedimiento es semejante al de división larga en aritmética.

1.- Arregle el dividendo y el divisor en el orden de las potencias descendentes de


una literal común, dejando un espacio para cualquier potencia faltante de la literal
en el dividendo.

2. Divida el primer término del dividendo entre el primer término del divisor. Esto
da el primer término del cociente.

3. Multiplique el divisor por el primer término del cociente y sustraiga el resultado


del dividendo.

4. Considere el residuo así obtenido como un nuevo dividendo y repita los pasos 2
y 3 para encontrar el segundo término del cociente y el siguiente residuo.

5. Continué este proceso, hasta que se obtenga un residuo que sea cero o de
menor grado en la literal común que el grado del divisor.

Si el residuo es cero entonces la división es exacta y el resultado se puede


expresar como:

Dividendo
= Cociente
Divisor

Si el residuo no es igual a cero entonces expresamos el resultado como


67
Dividendo Residuo
= Cociente +
Divisor Divisor
En cualquier caso se puede comprobar el resultado mediante la relación:

Dividendo = cociente x divisor + residuo

Todo lo anteriormente expuesto, se puede observar en los siguientes ejemplos.

Ejemplos:

1.-Sean los polinomios 𝑓(𝑥) = 2𝑥 4 − 3𝑥 3 + 𝑥 2 − 2𝑥 + 5 𝑦 𝑔(𝑥) = 𝑥 2 + 2𝑥 + 1


para obtener el residuo y el cociente de la división de los dos polinomios anteriores
tendremos:

2 x 2  7 x  13

x 2  2x  1 2 x 4  3x 3  x 2  2 x  5
 2x 4  4x3  2x 2
___________________
 7x3  x 2  2x
7 x 3  14 x 2  7 x
__________________
13x 2  5 x  5
 13x 2  26 x  13
________________
 21x  8

Donde los polinomios buscados serán


𝑟(𝑥) = −21𝑥 − 8
𝑞(𝑥) = 2𝑥 2 − 7𝑥 + 13

2.-Hallar el cociente y residuo, de la división de los polinomios

𝑓(𝑥) = 2𝑥 3 − 3𝑥 2 + 5𝑥 + 1 𝑦 𝑔(𝑥) = 𝑥 − 2

68
Usando el método tradicional obtendremos que los polinomios buscados son:

2x 2  x  7
x  2 2 x  3x 2  5 x  1
3

 2x  4x 2
x 2  5x
 x 2  2x
7x  1
 7 x  14
15

𝑟(𝑥) = 15
𝑞(𝑥) = 2𝑥 2 + 𝑥 + 7

De los dos ejercicios anteriores se puede comprobar que


𝑓(𝑥) = 𝑔(𝑥)𝑞(𝑥) + 𝑟(𝑥) 𝑦 𝑞𝑢𝑒 𝑔𝑟(𝑓) = 𝑔𝑟(𝑞) + 𝑔𝑟(𝑔)

Donde 𝑔𝑟 es grado de…

2.3.2. TEOREMA DEL RESIDUO

Si un polinomio P(x) se divide entre “x-r” hasta que se obtiene un residuo sin “x”,
entonces el residuo resultante es igual a P(r).

Es decir si representamos con Q(x) el cociente obtenido al dividir P(x) entre


“x-r” y representando con R el residuo constante. Entonces

P(x) = (x − r)Q(x) + R

Dividendo = divisor por cociente + residuo

Como esta es una identidad cierta para todos los valores de “x”, podemos sustituir
a “r” por “x” y obtener

P(r) = (r − r)Q(r) + R
= 0⦁ Q(r) + R
=R

69
Esto establece el teorema del residuo.

2.3.3. TEOREMA DEL FACTOR Y SU RECIPROCO

Si “r” es una raíz de la ecuación polinomial P(x)=0, entonces “x-r”


Es un factor de P(x), de manera reciproca si “x-r” es un factor de P(x) entonces
“r” es una raíz de la ecuación P(x)=0

Esto se puede demostrar a partir del teorema del residuo, es decir, si suponemos
la siguiente ecuación:

P(x) = (x − r)Q(x) + R = (x − r)Q(x) + P(r)

Si “r” es una raíz de la ecuación P(x)=0, entonces P(r)=0.


En consecuencia tenemos:
P(x) = (x − r)Q(x)

Para probar el reciproco del teorema solo necesitamos observar que dado “x-r”
como factor, podemos escribir la identidad anterior, que es cierta para todos los
valores de “x” .Cuando remplazamos a “x” por “r” tenemos

P(r) = (r − r)Q(r) = 0 ∗ Q(r) = 0

2.3.4. OTROS TEOREMAS FUNDAMENTALES

TEOREMA I: (Teorema fundamental del algebra).


Toda ecuación polinomial de grado n≥1 tiene al menos una raíz real o imaginaria.

TEOREMA II: Todo polinomio p(x) de grado n≥1 se puede expresar como el
producto de “n” factores lineales.

TEOREMA III: Toda ecuación polinomial P(x)=0 de grado “n” tiene exactamente
“n” raíces.

70
2.3.5. DIVISIÓN SINTETICA.

Definición:

Es un procedimiento abreviado de efectuar una división normal.

La idea de trabajar con la división, es la de inducir a usar la división sintética, que


es una forma más sencilla de hacer la operación entre un polinomio y un binomio.

Otra razón de la utilización de esta forma de división , es que, cuando se quiere


hallar las raíces de algunos polinomios, una forma , es probar con los posibles
factores que resulten al aplicar las reglas que veremos más adelante, y que
muchas veces el número de factores es extenso, y hay que efectuar la división
sintética muchas veces, para demostrar cuales son raíces del polinomio, y además
utilizar la ecuación degradada resultante para continuar hallando las demás raíces
(Donde la ecuación degradada se forma por los coeficientes que aparecen en la
tercera fila de la división sintética, que se describió en párrafos anteriores).

Deducción formal del procedimiento de la división sintética, consideraremos los


polinomios

P(x) = an x n + an−1 x n−1 + an−2 x n−2 + ⋯ + a0 y g(x) = x − c

Si queremos efectuar la operación

P(x) ÷ g(x)

Como en este caso g(x) es de grado uno, entonces el cociente [q(x)] será de
grado “n-1” y podemos escribir:

q(x) = bn−1 x n−1 + bn−2 x n−2 + bn−3 x n−3 + ⋯ + b0 con bn−1 ≠ 0

Como el residuo será cero ó de grado cero, es decir un número “r”. Entonces de
P(x) = g(x)q(x) + r(x)

an x n + an−1 xn−1 + an−2 x n−2 + ⋯ + a0 = (x − c)(bn−1 x n−1 + bn−2 x n−2 … + b0 ) + r → (A)

an x n + an−1 x n−1 + an−2 x n−2 + ⋯ + a0 = bn−1 x n + (bn−2 − cbn−1 )x n−1 + ⋯ + (b0 − cb1 )x + (r − cb0 ) → (B)

71
De la igualdad de polinomios, tendremos (igualando los miembros de los
polinomios A y B).

an = bn−1
an−1 = bn−2 − cbn−1

a1 = b0 − cb1
a0 = r − cb0

Por lo que los coeficientes del cociente y del residuo se obtienen como
an = bn−1
an−1 + cbn−1 = bn−2

a1 + cb1 = b0
a0 + cb0 = r

De tal manera que para tener los cálculos, de forma simplificada utilizando la
notación sintética se puede escribir de la siguiente manera:

𝑎𝑛 𝑎𝑛−1 … … 𝑎1 𝑎0
c 𝑐𝑏𝑛−1 𝑐𝑏1 𝑐𝑏0

𝑏𝑛−1 bn−2 b0 r

Donde:
bn−1 ; bn−2 ; b0 Son los coeficientes del cociente
r Es el residuo

EJEMPLO 1:

Obtener el residuo y el cociente de la operación entre los polinomios siguientes,


usando la división sintética.

P(x) = 2x 3 − 7x 2 + 8x − 15 y g(x) = x − 3

2 −7 8 − 15
3 6 −3 15

72
2 −1 5 0 (residuo)

Cociente
Ejemplo 2:

Efectuar la división sintética entre los polinomios siguientes.

(2𝑥 3 − 3𝑥 2 + 5𝑥 + 1) ÷ (𝑥 − 2)

2 −3 5 1
2 4 2 14

2 1 7 15 (residuo)

Que se puede escribir como


(2𝑥 3 − 3𝑥 2 + 5𝑥 + 1) ÷ (𝑥 − 2) = (2𝑥 2 + 𝑥 + 7)(𝑥 − 2) + 15

73
2.4. RAÍCES DE UN POLINOMIO

2.4.1. DEFINICIÓN DE RAIZ

Se dice que r es raíz de un polinomio si dicho valor (numérico, literal o complejo)


da solución a dicho polinomio, es decir, si al sustituir esa r en el polinomio el
residuo de este, es cero [R(r)] o también se dice que satisface al polinomio, es
decir f(r) = 0

2.4.2. NÚMERO DE RAICES

El Número de raíces de un polinomio está basado en el siguiente teorema:


TEOREMA I: (Teorema fundamental del algebra.)
Toda ecuación polinomial de grado n≥1 tienen al menos una raíz real o imaginaria.

TEOREMA III: Toda ecuación polinomial P(x)=0 de grado “n” tiene exactamente
“n” raíces.

74
2.5. TECNICAS ELEMENTALES PARA BUSCAR RAICES

2.5.1.- POSIBLES RAICES RACIONALES

Obtención de las raíces reales racionales.

Para obtener las raíces reales racionales de un polinomio P(x) se siguen los pasos
que se muestran a continuación:

Sea P(x) = an x n + an−1 x n−1 + an−2x n−2 + ⋯ + a0 = 0

Primer paso:

Tomar los factores de 𝑎0

Segundo paso:

Tomar los factores de 𝑎𝑛

Tercer paso:
a0
Tomamos la razón de
an

Cuarto paso:

Se agrupan todas las posibles raíces, ordenándolos de menor a mayor


simplificando los repetidos y los equivalentes, esto se realiza para probar
primeramente con los valores más pequeños que pueden ser raíces.

Quinto paso:

Se efectúa las divisiones en forma sintética, con todas las raíces posibles,
comenzando de menor a mayor.
Es posible hallar todas las raíces por este método, o bien se puede usar la
ecuación degradada.

75
Ejemplos:

1.-Para el polinomio que a continuación se muestra, aplique los pasos que se


muestran en los párrafos anteriores y halle todas sus raíces.

P(x) = 2x 3 − 7x 2 + 8x − 15 = 0

Primer paso:

Factores de a0 (de 15 son: ± 1, ±3, ±5, ±15)

Paso dos:

Factores de an ( de 2 son ± 1, ±2)

Paso tres:

a0 2 1 3 5 15
(de son ± , ± ± ± , ±1, ±3, ±5, ±15)
an 15 2 2 2 2

Paso cuatro:
1 3 5 15
Agrupando(± , ±1, ± , ± , ±3, ±5, ± , ±15)
2 2 2 2

Paso cinco:

Realizando la división sintética, probando con cada una de las raíces, concluimos
que la única raíz real es 3, tal como se muestra a continuación, ya que con todas
las demás el residuo no es cero y por lo tanto no son raíces del polinomio.

2 −7 8 − 15
3 6 −3 15
2 −1 5 0 (residuo)

2 −7 8 − 15
1 2 −5 3
2 −5 3 − 12 (residuo)

76

Las otras raíces son complejas, las cuales se pueden obtener con la ecuación
degradada de la división, en la cual el residuo dio cero (tercera fila), quedando una
ecuación cuadrática, que al resolverla, obtenemos las otras dos raíces
Como se muestra

2x 2 − x + 5 = 0
1 ± √39 i
Cuyas raices son
4

Otra forma de hallar raíces, es dividiendo el polinomio P(x) = 0 entre 𝑎𝑛 y obtener


𝑎
los factores de 0 , con dichos factores una vez ordenados de mayor a menor se
𝑎𝑛
realiza la división sintética, (que correspondería a el paso cuatro del el método
anterior).

Otra forma para resolver el ejemplo anterior:


Si
P(x) = 2x 3 − 7x 2 + 8x − 15 = 0

Entonces dividiendo entre 2 a P(x) , da como resultado lo siguiente:

7 15
P(x) = x 3 − x 2 + 4x − =0
2 2

Realizando la división sintética tendremos:

7 15
3 1 − 4 −
2 2
3 15
3 −
2 2

1 5
1 − 0 (residuo)
2 2

Resultando una ecuación degradada, de la misma forma que en el ejemplo


anterior, es decir:

2𝑥 2 − 𝑥 + 5 = 0
1 ± √39 i
Cuyas raices son
4

77
También se puede usar el teorema del residuo para hallar las raíces de un
polinomio es decir, al sustituir la raíz 𝑟 en el polinomio 𝑃(𝑥), si resulta que 𝑃(𝑟) es
cero, entonces 𝑟 es una raíz del polinomio. En el caso del ejercicio anterior
resultaría lo siguiente:

Si

P(x) = 2x 3 − 7x 2 + 8x − 15 = 0 y r=3
→ P(r) = P(3) = 2(3)3 − 7(3)2 + 8(3) − 15 = 54 − 63 + 24 − 15 = 0

Por lo tanto se puede ver qué r = 3, es raíz del polinomio.

De esta manera, se pueden probar todas las raíces, para conocer cuales cumplen.
Pero esto no es definitivo ya que si algunas no cumplen no podemos hacer más
para hallarlos.
Mientras que utilizando la división sintética tenemos el recurso de usar la ecuación
degradada.
Es por esto, que es más adecuado usar la división sintética.

2.5.2. RELACIÓN ENTRE LAS RAÍCES Y LOS


COEFICIENTES

En una ecuación, escrita de forma que el coeficiente de mayor potencia de “x” sea
1, como se muestra a continuación:

P(x) = x n + an−1 x n−1 + an−2 x n−2 + ⋯ + a0 = 0


Existen las siguientes relaciones entren los coeficientes y las raíces.

−an−1 = Suma de las raices.


an−2 = Suma de los productos de las raices tomadas de dos en dos.
−an−3 = Suma de los productos de las raices tomadas de tres en tres.
(−1)a0 = Producto de todas las raices.

Ejemplo:

1.- Determinar los coeficientes de 𝑎, 𝑏, 𝑦 𝑐 de tal forma que sean además raíces
del polinomio: 𝑥 3 − 𝑎𝑥 2 + 𝑏𝑥 − 𝑐 = 0

Solución:
Usando el teorema del factor, e igualando los factores al polinomio dado
escribimos.

78
(𝑥 − 𝑎)(𝑥 − 𝑏)(𝑥 − 𝑐) = 𝑥 3 − 𝑎𝑥 2 + 𝑏𝑥 − 𝑐
Desarrollando el producto y agrupando términos obtenemos la siguiente expresión:

𝑥 3 − (𝑎 + 𝑏 + 𝑐)𝑥 2 + (𝑎𝑏 + 𝑏𝑐 + 𝑎𝑐)𝑥 − 𝑎𝑏𝑐 = 𝑥 3 − 𝑎𝑥 2 + 𝑏𝑥 − 𝑐

Utilizando la relaciones entre las raíces y los coeficiente, se estable por la igualdad
entre polinomios las siguientes ecuaciones.

𝑎+𝑏+𝑐 =𝑎 (−𝑎𝑛−1 ) 𝑒𝑐𝑢𝑎𝑐𝑖ó𝑛 1


𝑎𝑏 + 𝑏𝑐 + 𝑎𝑐 = 𝑏 (𝑎𝑛−2 ) 𝑒𝑐𝑢𝑎𝑐𝑖ó𝑛 2
𝑎𝑏𝑐 = 𝑐 [(−1)𝑛 𝑎0 ] 𝑒𝑐𝑢𝑎𝑐𝑖ó𝑛 3

Resolviendo estas 3 ecuaciones, se obtienen los valores de a, b, y c

𝑎 = −1, 𝑏 = −1 𝑦 𝑐 = 1

Otro método.

Este método, se puede llevar a cabo efectuando la división sintética entre cada
una de las raíces, en este caso a, b, y c que también son raíces del polinomio
como a continuación se muestra.

𝑎 1 −𝑎 𝑏 𝑐
𝑎 0 𝑎𝑏
1 0 𝑏 𝑎𝑏 − 𝑐
𝑏 𝑏 𝑏2
1 𝑏 𝑏2 + 𝑏
𝑐 𝑐
1 𝑏+𝑐

Con los residuos de la división anterior, se escriben las ecuaciones que se


muestran, ya que como lo indica el problema son raíces del polinomio, por lo tanto
dichos residuos deben valer cero.

𝑎𝑏 − 𝑐 = 0 → 𝑒𝑐. 1
𝑏 2 + 𝑏 = 0 → 𝑒𝑐. 2
𝑏 + 𝑐 = 0 → 𝑒𝑐. 3

Despejando “b” de la ecuación 3 y sustituyendo en la 1, obtenemos que 𝑎 = −1


De la ecuación 2 se obtiene que 𝑏 = −1 𝑦 𝑏 = 0 (este valor no se considera ya
que indetermina el sistema).
79
Puesto que 𝑏 = −𝑐 entonces 𝑐 = 1
Como se puede observar son los mismos valores que se obtuvieron con el método
anterior, es decir 𝑎 = −1 𝑏 = −1 𝑦 𝑐 = 1

La comprobación se puede efectuar, sustituyendo los valores de a, b, y c en el


polinomio original del problema, y efectuando la división sintética entre cada uno
de estos mismos, dando todos los residuos cero, tal como se muestra a
continuación.

1 1 −1 −1
1 1 2 1
1 2 1 0
−1 −1 −1
1 1 0
−1 −1
1 0

2.5.3. COTAS DE RAÍCES REALES

TEOREMA IV: Sea P(x), con elementos dentro de los reales, si “a” y “b” son dos
números reales tales que “a< b” y P(a) y P (b), tienen signos contrarios entonces
P(x) tiene al menos una raíz “r” en el intervalo a< r< b.

TEOREMA: Toda ecuación de grado impar tiene por lo menos una raíz real cuyo
signo es opuesto al de su último término.
Toda ecuación polinomial de grado par, cuyo último término es negativo tiene
como mínimo dos raíces reales una positiva y una negativa

Otro teorema referente a los dos anteriores se puede enunciar de la siguiente


forma:

TEOREMA: Si el polinomio

F(x) = a0 x n + a1 x n−1 + ⋯ + an−1 x + an

Tiene coeficientes reales con a0 positivo y se aplica la división sintética a f(x)


donde (x-r) es el divisor, entonces.

1). Si r>0 todos los números del tercer renglón son positivos, r es una cota
superior de las raíces reales de f(x)=0.

80
2). Si r< 0 y todos los números del tercer renglón son, alternativamente, positivos
y negativos, r es una cota inferior de las raíces reales de f(x)=0.

Un cero del tercer renglón se puede sustituir con un signo más o un signo menos.

Prueba: En el caso 1, para cualquier valor mayor que r todos los números
del tercer renglón, después del primero serán mayores. En consecuencia
ningún número mayor que r puede ser una raíz. En el caso 2, para cualquier
número menor que r, todos los números del tercer renglón, después del
primero, serán mayores en valor absoluto. En consecuencia, ningún número
menor que r puede ser una raíz.

2.5.4 TEOREMA: REGLA DE LOS SIGNOS DE DESCARTES

El número de raíces reales positivas de una ecuación polinomial P(x)=0 con


coeficientes reales, es igual al número de variaciones de signo en P(x), ó es menor
que ese número, menos un numero par entero.
El número de raíces reales negativas de P(x)=0, es igual al número de variaciones
de signo de P (-x), ó es menor que este número, menos un número par entero.

Ejemplo:
1.- Para el polinomio 𝑥 5 − 6𝑥 3 + 6𝑥 2 − 7𝑥 + 6 = 0

a) Construir una tabla de las posibles raíces, utilizando la regla de los signos de
Descartes.

b) hallar todas las raíces del polinomio si sabemos que i es una de sus raíces.

Solución:

a)
Posibilidades
1 2 3
Raíces reales(+) 4 4-2=2 4-4=0
Raíces reales(-) 1 1 1
Raíces complejas 0 2 4
Total de raíces 5 5 5

81
b) Si 𝑖 es raíz, entonces −𝑖 también lo es (por el teorema de las raíces
imaginarias)
De esta manera, ya tenemos dos raíces, pero nos falta hallar las otras tres, esto lo
haremos utilizando la división sintética, y posteriormente utilizando la ecuación
degradada resultante de dividir sucesivamente entre 𝑖 𝑦 − 𝑖 tal como se muestra
a continuación:

10 −6 6 −7 6
𝑖 𝑖 −1 − 7𝑖 7 + 6𝑖 −6
1 𝑖 −7 6 − 7𝑖 6𝑖 0
−𝑖 −𝑖 0 − 7𝑖 − 6𝑖
1 0 −7 6 0

Estableciendo la ecuación degradada tomando la última fila de la división sintética


anterior tendremos:
𝑥 3 − 7𝑥 + 6 = 0

Resolviéndolo, utilizando los pasos dados con anterioridad, tendremos que:


Los factores serán ±1, ±2, ±3 𝑦 ± 6 y que al hacer la división sintética entre cada
uno de ellos, resulta que las raíces son: 1, 2 𝑦 − 3

Por lo tanto las 5 raíces son: 1,2 𝑦 − 3 así como 𝑖 𝑦 − 𝑖

82
2.6 TEOREMAS SOBRE RAÍCES

2.6.1 TEOREMA DE LAS RAICES IRRACIONALES CONJUGADAS.

Suponga que los coeficientes de una ecuación polinomial, P(x)=0, son números
racionales. Si a + √b con “a” y “b” racionales y raíz de “b” irracional, es una raíz
de la ecuación, entonces a − √b también es una raíz de la ecuación. De manera
reciproca si a − √b es una raíz, entonces a + √b también es una raíz.

Ejemplo 1:

𝑎. −Hallar las raíces del polinomio siguiente x 4 − 2x 3 − 5x 2 + 10x − 3 = 0 si una


3−√5
de las raíces es
2
𝑏. −Escribir el polinomio como n - factores lineales

Solución:

Esta ecuación no se puede resolver utilizando el proceso del ejemplo anterior


puesto que las únicas raíces de dicho polinomio son solamente ±1 𝑦 ± 3, al
probar con la división sintética ninguno de estas cuatro posibilidades son raíces,
por lo que procede a la utilización del teorema de las raíces irracionales.

Utilizando el teorema anterior y tomando cada una de las raíces dadas como
factores se podrá escribir.

3 − √5 3 + √5 3 √5 3 √5
[𝑥 − ( )] [𝑥 − ( )] = (𝑥 − + ) (𝑥 − − ) = 𝑥 2 − 3𝑥 + 1
2 2 2 2 2 2

Ahora efectuamos la división normal entre el polinomio original y el polinomio


resultante obtenido en el paso anterior

83
x2  x  3
x  3x  1
2
x  2 x  5 x  10 x  3
4 3 2

 x 4  3x 3  x 2
x 3  6 x 2  10 x
 x 3  3x 2  x
 3x 2  9 x  3
3x 2  9 x  3
0 0 0

Y ahora resolviendo la ecuación del cociente x 2 + x − 3 = 0 obtenemos las otras


−1+√13 −1−√13
raíces las cuales son: y por lo que las cuatro raíces del polinomio
2 2
serán:

3 − √5 3 − √5 −1 + √13 −1 − √13
r1 = , r2 = , r3 = y r4 =
2 2 2 2

𝑏. −Entonces el polinomio escrito como factores lineales es:

3 − √5 3 + √5 −1 + √13 −1 − √13
[𝑥 − ( )] [𝑥 − ( )] [𝑥 − ( )] [𝑥 − ( )] =
2 2 2 2

= (𝐱 𝟐 − 𝟑𝐱 + 𝟏)(𝐱 𝟐 + 𝐱 − 𝟑) = 𝐱 𝟒 − 𝟐𝐱𝟑 − 𝟓𝐱𝟐 + 𝟏𝟎𝐱 − 𝟑 𝐋. 𝐐. 𝐐. 𝐃.

Otro método.

Utilizando la división sintética, entre cada una de las raíces, y tomando la ecuación
degradada resultante para obtener las otras raíces como se muestra a
continuación:

1 −2 −5 10 −3
3 √5 3 √5 1 √5 11 3√5
− − − − + 3
2 2 2 2 2 2 2 2

1 √5 9 √5 9 3√5
1 − − − − + 0
2 2 2 2 2 2
3 √5 3 √5 3 √5 9 3√5
+ + + − −
2 2 2 2 2 2 2 2

1 1 −3 0

84
La ecuación degradada resultante es x 2 + x − 3 = 0 que es la misma que se obtuvo
en líneas arriba y con la cual se obtienen las dos raíces que faltan.

2.6.2 C0NJUGADAS COMPLEJAS O IMAGINARIAS.

Si el número complejo “a+bi”; b≠0, es una raíz de una ecuación polinomial con
coeficientes reales, entonces el número complejo “a-bi” (el conjugado), también es
una raíz de dicho polinomio.

Ejemplo:

1.- Resuelva la ecuación 𝑥 4 − 4𝑥 3 + 10𝑥 2 + 12𝑥 − 39 = 0 , si 2 − 3𝑖 es una de


las raíces y escriba el polinomio como un producto de factores lineales

Solución:

Como se pide que se resuelva la ecuación, hay que hallar todas las raíces, por lo
tanto efectuamos la división sintética como se muestra a continuación:

1 −4 10 12 − 39
2 − 3𝑖 2 − 3𝑖 − 13 − 6 + 9𝑖 39
1 − 2 − 3𝑖 − 3 6 + 9𝑖 0
2 + 3𝑖 2 + 3𝑖 0 − 6 − 9𝑖
1 0 −3 0

Con la ecuación degradada que se plantea de la última fila de la división sintética


se pude hallar las otras raíces.

x2 − 3 = 0
resolviendo x = ±√3

Por lo tanto las raíces del polinomio serán:


r1 = 2 − 3i r2 = 2 + 3i r3 = √3 y r4 = −√3

Por lo tanto el polinomio se puede escribir como un producto de factores lineales


de la siguiente manera

P(x) = (x − r1 )(x − r2 )(x − r3 )(x − r4 ) = [x − (2 − 3i)][x − (2 + 3i)][x − √3][x − (−√3 )]

P(x) = x 4 − 4x 3 + 10x 2 + 12x − 39 = 0

85
Este mismo problema, se puede resolver de otra forma o por otro método
Se puede efectuar primeramente, el producto de los factores de las dos primeras
raíces, considerando el teorema de las raíces imaginarias, y posteriormente
efectuar una división entre el polinomio resultante y el original, tal como se
muestra a continuación.

[x − (2 − 3i)][x − (2 + 3i)] = (x − 2 + 3i)(x − 2 − 3i) = x 2 − 4x + 13


Efectuando la división

x2  3
x  4 x  13
2
x  4 x  10 x  12 x  39
4 3 2

 x 4  4 x 3  13x 2
 3x 2  12 x  39
3 x  12 x  39
0 0 0
Como el residuo es cero, usamos el cociente de la división, 𝑥 2 − 3 y con esta
hallamos las otras dos raíces, tal como se hizo en método anterior.

2.- Determinar si los números 0,-2,3/2, e i son raíces del polinomio

2𝑤 4 − 𝑤 3 − 𝑤 2 − 𝑤 − 3
Utilizando la división sintética. Si los cuatro números anteriores no son todas raíces
halle las que faltan.

2 −1 −1 −1 −3
0 0 0 0 0

2 −1 −1 − 1 − 3 (residuo)

Como el residuo es diferente de cero, entonces no es raíz

2 −1 −1 −1 −3
−2 −4 10 − 18 38

2 −5 9 − 19 35 (residuo)

Como el residuo es diferente de cero, entonces no es raíz

2 −1 −1 −1 −3
3
3 3 3 3
2
2 2 2 2 0

86
Como el residuo es cero, entonces es raíz

2 −1 −1 −1 −3
i 2i −2−i 1 − 3i 3
2 − 1 + 2i −3−i − 3i 0

Como el residuo es cero, entonces es raíz.

Se ve que solamente dos valores cumplieron con ser raíz, pero según el problema
también pide que se hallen las otras raíces. Entonces utilizando la división sintética
con las dos raíces halladas y aplicando el teorema para las raíces complejas
podemos hallar todas las raíces como se muestra a continuación.

Primeramente utilizamos el teorema de las raíces complejas, que establece que, si


0 + 𝑖 es raíz entonces 0 − 𝑖 también va ser raíz, lo que se demuestra utilizando la
ecuación degradada de la última división sintética con la raíz 𝑖.

2 −1 −1 −1 −3
i 2i −2−i 1 − 3i 3
2 − 1 + 2i −3−i − 3i 0
−𝑖 − 2𝑖 𝑖 3𝑖
2 −1 −3 0

De la quinta fila de la división sintética, se plantea una ecuación cuadrática como a


continuación se muestra:

2𝑥 2 − 𝑥 − 3 = 0

3
Resolviendo esta ecuación tendremos las otras dos raíces, que serán −1 y .
2

También se puede usar todo el proceso anterior y continuar la división con el valor
3
de que ya se probó en la primera parte del problema como se muestra en las
2
siguientes operaciones.

2 −1 −1 −1 −3
i 2i −2−i 1 − 3i 3
2 − 1 + 2i −3−i − 3i 0
−𝑖 − 2𝑖 𝑖 3𝑖
2 −1 −3 0
3
3 3
2
2 2 0
87
De la última fila se plantea la ecuación 2𝑥 + 2 = 0, que se resuelve para hallar la
última raíz que será −1

Lo que da como resultado que las raíces son:

3
𝑟1 = 𝑖 𝑟2 = −𝑖 𝑟3 = 𝑦 𝑟4 = −1
2

3.- Dado el polinomio 𝑥 4 + 6𝑥 3 + 17𝑥 2 + 36𝑥 + 66 = 0, si −3 + √2 𝑖 es una de las


raíces y hacer una tabla de las posibles raíces según el criterio de la regla de los
signos de Descartes.

Solución.

Método 1.

Aplicando el teorema del factor y el de las raíces imaginarias tendremos.

[𝑥 − (−3 − √2 𝑖][𝑥 − (−3 + √2 𝑖)] = (𝑥 + 3 + √2 𝑖)(𝑥 + 3 − √2 𝑖) = 𝑥 2 + 6𝑥 + 11 = 0

Efectuando la división normal entre el polinomio original y este último hallado se


obtiene.

x2  6
x 2  6 x  11 x 4  6 x 3  17 x 2  36 x  66
 x 4  6 x 3  11x 2
6 x 2  36 x  66
 6 x 2  36 x  66
0 0 0

Resolviendo la ecuación que resulta en el cociente de la división anterior


x2 + 6 = 0

Las raíces son: √6 𝑖 𝑦 − √6 𝑖

Por lo que las cuatro raíces del polinomio son:


𝑟1 = −3 + √2 𝑖 𝑟2 = −3 + √2 𝑖 𝑟3 = √6 𝑖 𝑦 𝑟4 = −√6 𝑖

Ahora para armar la tabla, se puede observar que hay cuatro cambios de signo en
el polinomio P(x), lo cual nos indica que hay 4 posibles raíces reales positivas, y no
hay ninguno en el de P (-x), indicando con esto que no hay ninguna raíz real
negativa, por lo tanto la tabla es:
88
Raíces reales 4 4-2=2 4-4=0
positivas
Raíces reales 0 O 0
negativas
Raíces complejas 4 2 4
Total de raíces 4 4 4

Método 2.

En este método como ya lo hemos venido haciendo, con otros problemas, se


efectúa la división sintética con la raíz que se proporciona, y se aplica el teorema
de las raíces imaginarias, para establecer la otra raíz, y se vuelve a hacer la
división.

1 6 17 36 66
−3 + √2 𝑖 − 3 + √2 𝑖 − 11 − 18 + 6√2𝑖 − 66

1 3 + √2 i 6 18 + 6√2i 0
−3 − √2𝑖 − 3 − √2𝑖 0 − 18 − 6√2𝑖

1 0 6 0

Se puede observar que, de la ecuación degradada obtenida en la última división


sintética 𝑥 2 + 6 = 0 se pueden obtener las otras raíces tal como se hace en el
método anterior.

89
2.6.3 SOLUCIÓN DE ECUACIONES CÚBICAS.

La forma general de una ecuación cúbica es:

𝑥 3 + 𝑝𝑥 2 + 𝑞𝑥 + 𝑟 = 0

Que en forma más simple se puede escribir:

𝑥 3 + 𝑞𝑥 + 𝑟 = 0

Que se tomara como una ecuación tipo de las cúbicas.


Para resolver este tipo de ecuaciones puede aplicarse los siguientes pasos.

Paso 1.
Suponemos a 𝑥 = 𝑦 + 𝑧 el cual se sustituye en la ecuación tipo y se simplifica.
Obteniéndose:
𝑦 3 + 𝑧 3 + (3𝑦𝑧 + 𝑞)𝑥 + 𝑟 = 0

Paso 2.
Se resuelve la ecuación anterior, la cual se debe cumplir para cuando

−𝑞3
𝑦 3 + 𝑧 3 = −𝑟 𝑦 3𝑦𝑧 = −𝑞 → 𝑦 3 𝑧 3 =
27

Paso 3.
Se supone que los valores anteriores son raíces de la ecuación cuadrática.

𝑡 2 + 𝑟𝑡 − 𝑞3
=0
27

La cual se le llama la resolvente de dicha ecuación.

Paso 4.
Se resuelve esta ecuación cuadrática y obtenemos que:

𝑟 𝑟 2 𝑞3 𝑟 𝑟 2 𝑞3
𝑦3 = − + √ + → 𝑒𝑐. (𝐴) 𝑦 𝑧 3 = − − √ + → 𝑒𝑐. (𝐵)
2 4 27 2 4 27

90
Paso 5.

A partir de la relación 𝑥 = 𝑦 + 𝑧 se puede obtener que:

𝑟 𝑟2 𝑞3 1 𝑟 𝑟2 𝑞3 1
𝑥 = [− + √ + ]3 + [− − √ + ]3
2 4 27 2 4 27

La ecuación anterior es conocida como fórmula de CARDANO, ya que él fue el


primero en publicarla en la obra de Ars Magna, en 1545. Cardano obtuvo la
solución de Tartaglia; pero la solución de la ecuación cúbica parece deberse
originalmente a Scipio Ferro, alrededor de 1505. Una interesante nota histórica
sobre este punto que aparece al final de la obra de Burnside y Paton, Theory of
Equations.

Cada una de las cantidades del segundo miembro de las ecuaciones A Y B del paso
4 tiene tres raíces cúbicas, por tanto parece que x tiene nueve valores, esto sin
−𝑞
embargo no es el caso, porque como 𝑦 = , las raíces cúbicas deben ser
3
tomadas a pares, de manera que el producto de cada una sea un número racional.
Por lo tanto, si 𝑦, 𝑧 son los valores de cualquier par de raíces cúbicas que
satisfacen esta condición, los demás únicos pares admisibles serán 𝑤 y 𝑤 2 𝑧, 𝑤𝑧,
en donde 𝑤, 𝑤 2 son las raíces cúbicas imaginarias de la unidad. En resumen, las
raíces cúbicas de la ecuación son:

𝑦+𝑧 𝑤𝑦 + 𝑤 2 𝑧 𝑤𝑦 + 𝑤𝑧

Ejemplo:

1.- Resolver la ecuación 𝑥 3 − 15𝑥 − 126 = 0

Solución:

Como 𝑥 = 𝑦 + 𝑧 resulta, que según la ecuación tipo, 𝑞 = −15 por lo tanto la


ecuación dará:
𝑦 3 + 𝑧 3 + (3𝑦𝑧 − 15)𝑥 = 126

De aquí que, esta ecuación se verifica para los valores de y, z; que cumplan las
condiciones.

3𝑦𝑧 − 15 = 0 𝑜 𝑠𝑒𝑎 𝑦 3 𝑧 3 = 125 𝑦 𝑦 3 + 𝑧 3 = 126

91
Por lo tanto 𝑦 3 , 𝑧 3 son las raíces de la ecuación

𝑡 2 − 126𝑡 + 125 = 0

Resolviendo resulta que:


𝑦 3 = 125 → 𝑦 = 5
𝑧3 = 1 → 𝑧 = 1

Por lo tanto 𝑦 + 𝑧 = 5 + 1 = 6

Entonces

−1 + √3 𝑖 −1 + √3 𝑖
𝑤𝑦 + 𝑤 2 𝑧 = 1 + (5 + ) = −3 + 2√3 𝑖
2 2

𝑤 2 𝑦 + 𝑤𝑧 = −3 − 2√3 𝑖

Por lo tanto las raíces son:

6, − 3 + 2√3 i y − 3 − 2√3 i L. Q. Q. D.

92
2.7 MÉTODOS ESPECIALES PARA LA SOLUCIÓN DE
ECUACIONES CUARTICAS

En esta parte se hará un breve estudio de algunos de los métodos que se emplean
para obtener la solución general de una ecuación de cuarto grado. Se verá que en
cada uno de los métodos tenemos que resolver una ecuación auxiliar de tercer
grado, y por lo tanto, se verá que, como en el caso de las cúbicas la solución g
general no se adapta para escribir inmediatamente la solución de una ecuación
numérica dada.

2.7.1. CAMBIO DE VARIABLE.


En muchas ocasiones se encuentran ecuaciones de cuarto grado las cuales no se
pueden resolver tan fácilmente, en ese caso se puede usar este método para
encontrar la solución.
+
Si la ecuación es de la forma 𝒙𝟒 + 𝒙𝟐 𝒄 = 𝟎

Se puede hacer el siguiente cambio de variable 𝑥 4 = 𝑡 2 𝑝𝑜𝑟 𝑙𝑜 𝑡𝑎𝑛𝑡𝑜 𝑥 2 = 𝑡
+ +
y de esta manera la ecuación original queda como 𝑡 2 𝑎𝑡 𝑐 = 0
− −

La cual como se puede ver es una ecuación cuadrática se puede resolver más
fácilmente, aunque el valor hallado de t no es la solución de la ecuación de cuarto
grado, y aquí es donde se puede usar el cambio establecido para poder hallar los
valores de la variable de la ecuación original () por lo general es más fácil usar
𝑥 2 = 𝑡 y por lo tanto obtendremos dos valores que son:

x = ±√ t

𝑆𝑖 ± √𝑡 Es puramente real, es sencillo hallar los valores de x, pero si son


imaginarias hay que hacer uso de la forma polar de un número complejo.
Esto lo podemos aclarar en los siguientes ejercicios.

Ejemplos:

1.-Resolver la siguiente ecuación:


x4 − x2 − 1

Solución:

𝑥4 = 𝑡2
𝑥2 = 𝑡
𝑡2 − 𝑡 − 1 = 0

93
+
−(−1) ± √(−1)2 − 4(1)(−1) 1 ± √1 + 4 1 − √5
𝑡= = =
2 2 2
1 + √5
𝑡1 =
2

1 − √5
𝑡2 =
2

Raíces

1 + √5
x1 = √
2

x2 = −√1 + √5
2

x3 = √1 − √5
2

x4 = −√1 − √5
2

2.- Resolver la siguiente ecuación: 𝐱 𝟒 + 𝟑𝐱𝟐 + 𝟐 = 𝟎


𝐬𝐢 𝐱 𝟒 = 𝐭 𝟐 𝐲 𝐱 𝟐 = 𝐭
Entonces
La ecuación se escribe:
t 2 + 3t + 2 = 0 Resolviendo tendremos t1 = −1 y t 2 = −2. Por lo que
x 2 = ±√t1 Entonces x = ±√−1 = ±i y por lo tanto x1 = i y x2 = −i
Ahora haciendo
x 2 = ±√t 2 entonces x3,4 = ±√2i por lo que x3 = √2i y x4 = −√2i
Que son las soluciones de las ecuaciones.

OTRA FORMA:
De otra forma se puede factorizar y tendremos:
x 4 + 3x 2 + 2 = (x 2 + 2)(x 2 + 1) = 0
Igualando a cero ambos factores se obtiene la solución del párrafo anterior.

94
3.- Resolver la siguiente ecuación: 𝐱 𝟒 + 𝐱 𝟐 + 𝟏 = 𝟎

Haciendo el cambio de variable que se muestra en el ejemplo 2 se obtiene la


ecuación:
𝐭𝟐 + 𝐭 + 𝟏 = 𝟎
Esta no se puede factorizar, por lo tanto haciendo el cambio de variable se tendrá
que
−𝟏 + √𝟑𝐢 −𝟏 − √𝟑𝒊
𝐭𝟏 = 𝐲 𝐭𝟐 =
𝟐 𝟐

Por lo que usando los principios de números complejos para obtener raíces con la
ecuación general en forma polar que dice:

n n θ + 2πk
√r cis θ = √r cis ( )
n

Haciendo la igualación que indica el cambio de variable obtendremos las Siguientes


ecuaciones:
para x1,2 = √t1 y x3,4 = √t 2
Por lo tanto haciendo el cálculo de estos

√3
x1 = cis 60 = 0.5 + i para k = 0
2

√3
x2 = cis 240 = −0.5 − i para k = 1
2

Tomando ahora el ángulo negativo.

√3
x1 = cis − 60 = 0.5 − i para k = 0
2
√3
x2 = cis 120 = −0.5 + i para k = 1
2
Como se puede observar, las raíces son conjugadas la 1 con la 3 y la 2 con la 4

95
2.7.2. MÉTODO DE FERRARI

La solución de la ecuación cuarta lo obtuvo por primera vez FERRARI, discípulo de


CARDANO de la siguiente manera.

Para ecuaciones cuarticas de la forma:

𝑥 4 + 2𝑝𝑥 3 + 𝑞𝑥 2 + 2𝑟𝑥 + 𝑠 = 0 − − − − − −(1)

Paso 1. Sumar a ambos miembros la expresión (𝑎𝑥 + 𝑏)2 , de tal manera que el
primer miembro sea un cuadrado perfecto, entonces desarrollando y agrupando
términos se obtiene la ecuación:

𝑥 4 + 2𝑝𝑥 3 + (𝑞 + 𝑎2 )𝑥 2 + 2(𝑟 + 𝑎𝑏)𝑥 + 𝑠 + 𝑏 2 = (𝑎𝑥 + 𝑏)2

Paso 2. Suponemos que el primer miembro de la ecuación, es igual a (𝑥 2 +


𝑝𝑥 + 𝑘)2

Entonces desarrollando, agrupando y comparando los coeficientes de este trinomio


cuadrado obtendremos:

𝑝2 + 2𝑘 = 𝑞 + 𝑎2
𝑝𝑘 = 𝑟 + 𝑎𝑏
𝑘2 = 𝑠 + 𝑏2

Despejando de las ecuaciones anteriores tendremos las ecuaciones siguientes:

𝑎2 = 𝑝2 + 2𝑘 − 𝑞 (3)
𝑎𝑏 = 𝑝𝑘 − 𝑟
𝑏2 = 𝑘 2 − 𝑠 (4)

Paso 3. Eliminamos a 𝑎 𝑦 𝑏 de ambas ecuaciones, utilizando el argumento


siguiente:
(𝑎𝑏)2 = 𝑎2 𝑏 2
Entonces
(𝑝𝑘 − 𝑟)2 = (2𝑘 + 𝑝2 − 𝑞)(𝑘 2 − 𝑠)

96
Desarrollando este producto, obtenemos la siguiente ecuación cúbica

2𝑘 3 − 𝑞𝑘 2 + 2(𝑝𝑟 − 𝑠)𝑘 − 𝑝2 𝑠 + 𝑞𝑠 − 𝑟 2 = 0 (2)

Paso 4. De la ecuación cúbica anterior, se haya siempre un valor real de k, luego


𝑎 𝑦 𝑏 están determinados por las ecuaciones 3 y 4 del paso 2

Los valores de las raíces, se obtienen de las siguientes ecuaciones cuadráticas.

(𝑥 2 + 𝑝𝑥 + 𝑘)2 = (𝑎𝑥 + 𝑏)2


Aplicando la raíz cuadrada ambos miembros
𝑥 2 + 𝑝𝑥 + 𝑘 = ±(𝑎𝑥 + 𝑏)
𝑥 2 + (𝑝 − 𝑎)𝑥 + (𝑘 − 𝑏) = 0 (5)
𝑥 2 + (𝑝 + 𝑎)𝑥 + (𝑘 + 𝑏) = 0 (6)

De esta manera, para resolver una ecuación que tenga la forma de la ecuación (1),
se puede utilizar la ecuación (2) para hallar el valor de 𝑘, puesto que todos los
valores que se necesitan en esta ecuación están dados en la uno, a continuación
se calculan los valores que nos proporcionaran las ecuaciones (3) y (4), para que
finalmente se sustituyan todos estos valores calculados en las ecuaciones (5) y
(6), con las cuales como ya se dijo se encuentran las cuatro raíces del polinomio
que se quiere resolver.

Ejemplo:

1.- Para el polinomio 𝑥 6 − 18𝑥 4 + 16𝑥 3 + 28𝑥 2 − 32𝑥 + 8 = 0, si una de las raíces
es √6 − 2 hallar todas las demás raíces.

Solución:

Aplicando el teorema del factor y el de las raíces racionales escribimos la siguiente


ecuación.
[x − (−2 + √6][x − (−2 − √6)] = x 2 + 4x − 2

97
Teniendo el polinomio anterior, lo usamos para efectuar la división entre el
polinomio original, de la cual se obtendrá el cociente, para continuar con el cálculo
de las otras raíces, debiéndose obtener como residuo cero. Tal como se muestra a
continuación.

x 4 4 x 3  8 x  4
x  4x  2
2
x 6
 18 x  16 x  28 x 2  32 x  8
4 3

 x 6  4x5  2x 4
 4 x 5  16 x 4  16 x 3
4 x 5  16 x 4  8 x 3
8 x 3  28 x 2  32 x
 8 x 3  32 x 2  16 x
 4 x 2  16 x  8
 4 x 2  16 x  8
0 0 0
Método 1.
Con el cociente de la división, aplicamos el método de FERRARI para encontrar las
otras raíces
Comparando la ecuación tipo (1) con el cociente obtenido en la división tendremos
𝑥 4 + 2𝑝𝑥 3 + 𝑞𝑥 2 + 2𝑟𝑥 + 𝑠 = 𝑥 4 − 4𝑥 3 + 8𝑥 − 4

Por la propiedad de la igualdad de polinomios


p = −2 q=0 r = 4 y s = −4

Sustituyendo estos valores en la ecuación cúbica


2k 3 − qk 2 + 2(pr − s)k − p2 s + r 2 = 0 (2)

Se obtiene los valores de k (0, 2 y -2), tomándose el valor absoluto más pequeño,
que será cero (k=0)

98
Ahora calculamos 𝑎 𝑦 𝑏 con las ecuaciones (3) y (4) de tal manera que los
valores son: 𝑎 = ±2 𝑦 𝑏 = ±2
Finalmente se sustituyen los valores de 𝑘, 𝑝, 𝑎 𝑦 𝑏 en las ecuaciones (5) y (6),
presentándose 2 casos. El primero para 𝑎(+) 𝑦 𝑏(−) el segundo caso para
𝑎(−) 𝑦 𝑏(+) dando iguales resultados para ambos casos es decir:

𝑥1 = 2 + √2 𝑥2 = 2 − √2 𝑥3 = √2 𝑦 𝑥4 = −√2 .

De tal manera que las 6 raíces del polinomio son:

𝑥1 = −2 + √6 𝑥2 = −2 − √6 𝑥3 = 2 + √2 𝑥4 = 2 − √2 𝑥5 = √2 𝑦 𝑥6 = −√2

La combinación en donde 𝑎 𝑦 𝑏 son positivas (+) no satisfacen al polinomio


(queda al lector demostrarlo).

METODO 2.
Por factorización:
Este método no siempre es aplicable, pero habría que probarse, como en este caso
de que si es posible aplicarlo para el polinomio 𝑥 4 − 4𝑥 3 + 8𝑥 − 4.
𝑥 4 − 4𝑥 3 + 8𝑥 − 4 = 0
(𝑥 4 − 4) − 4𝑥(𝑥 2 − 2) = 0
(𝑥 2 − 2)(𝑥 2 + 2) − 4𝑥(𝑥 2 − 2) = 0
(𝑥 2 − 2)(𝑥 2 + 2 − 4𝑥) = 0
(𝑥 2 − 2)(𝑥 2 − 4𝑥 + 2) = 0

Estas 2 últimas ecuaciones, son las que se obtuvieron en el método anterior y con
las cuales se hallaron las raíces del polinomio en cuestión.

99
2.7.3. MÉTODO DE DESCARTES.

Este método de solución fue dado por Descartes, en el año de 1637, y es válido
para ecuaciones de la forma siguiente.
𝑥 4 + 𝑞𝑥 2 + 𝑟𝑥 + 𝑠 = 0 (1)

Los pasos que nos permiten deducir las ecuaciones, para poder resolver
ecuaciones de este tipo, son:
Paso 1.- Suponer que el polinomio de la ecuación (1), es factorizable de la
siguiente forma:
(𝑥 2 − 𝑘𝑥 + 𝑚)(𝑥 2 + 𝑘𝑥 + 𝑙)

Paso 2.- Desarrollar el producto de los trinomios, e igualarlos al polinomio de la


ecuación (1)
𝑥 4 + 𝑞𝑥 2 + 𝑟𝑥 + 𝑠 = (𝑥 2 − 𝑘𝑥 + 𝑚)(𝑥 2 + 𝑘𝑥 + 𝑙)
= 𝑥 4 + (𝑚 − 𝑘 2 + 𝑙)𝑥 2 + 𝑘(𝑚 − 𝑙)𝑥 + 𝑙𝑚
Igualando los dos polinomios tendremos.
𝑞 = 𝑚 − 𝑘2 + 𝑙 𝑒𝑐. 3
𝑟 = 𝑘(𝑚 − 𝑙) 𝑒𝑐. 4
𝑠 = 𝑙𝑚 𝑒𝑐. 5

Despejemos a “m”, en la ecuación (3), sustituyéndolo en (4), y despejamos a 𝑙


Posteriormente despejamos a 𝑙 de la ecuación (3), y sustituyéndolo en la ecuación
(4), seguidamente despejamos a 𝑚. De donde 𝑙 𝑦 𝑚 resultan:

𝑘 3 + 𝑞𝑘 − 𝑟 𝑘 3 + 𝑞𝑘 + 𝑟
𝑙= 𝑒𝑐. (6) 𝑦 𝑚= 𝑒𝑐. (7)
2𝑘 2𝑘

Sustituyendo a 𝑚 𝑦 𝑙 en la ecuación (5), obtenemos la ecuación elevada a la


sexta potencia que se muestra:
𝑘 6 + 2𝑞𝑘 4 + (𝑞2 − 4𝑠)𝑘 2 − 𝑟 2 = 0 𝑒𝑐(8)

100
Paso 3.-
Con la ecuación a la sexta potencia calculada en el paso anterior, que no es más
que una ecuación cúbica en 𝑘 2 , que siempre tiene una solución real positiva (la
cual se toma para el valor de “k”), una vez teniendo estos valores (𝑘, 𝑞 𝑦 𝑟) , se
calcula 𝑚 𝑦 𝑙 con la ayuda de las ecuaciones (6) y (7).

Paso 4.-
Finalmente se halla la solución de la ecuación cuartica, resolviendo las ecuaciones
cuadráticas que se plantean en el paso 1, que son las siguientes:

𝑥 2 − 𝑘𝑥 + 𝑚 = 0 𝑥 2 + 𝑘𝑥 + 𝑙 = 0

Obteniéndose de esta manera las raíces.

Ejemplo:

1.- Hallar las raíces del polinomio 𝑥 4 − 2𝑥 2 + 8𝑥 − 3 = 0 y escribirlo como un


producto de n -factores lineales

Solución:

Primeramente igualando el polinomio tipo con el del problema, para identificar las
variables, 𝑞, 𝑟 𝑦 𝑠

𝑥 4 + 𝑞𝑥 2 + 𝑟𝑥 + 𝑠 = 𝑥 4 − 2𝑥 2 + 8𝑥 − 3

Entonces
𝑞 = −2, 𝑟 = 8 𝑦 𝑠 = −3

Ahora sustituyendo estos valores en la ecuación de sexto grado del paso 2


ecuación (8). Obtenemos:
𝑘 + 4𝑘 4 + 16𝑘 2 − 64 = 0

101
Resolviendo esta ecuación, por el método ya conocido, resulta 𝑘 = ±2, que son
las raíces más pequeñas; tomando cualquiera de los dos valores de k, se procede a
calcular a 𝑙 𝑦 𝑚 , obteniéndose lo siguiente:

𝑚 = 3 𝑦 𝑙 = −1

Finalmente, sustituyendo los valores de 𝑘, 𝑙 𝑦 𝑚 en las ecuaciones que se


mostraron en el paso 1, se tienen los resultados siguientes

(x 2 − kx + m)(x 2 + kx + l) = 0
(x 2 − 2x + 3)(x 2 + 2x + −1) = 0
x12 = 1 ± √2 i
x34 = −1 ± √2

De esta manera se hallan las cuatro raíces del polinomio.

Ahora escribiendo el polinomio como un producto de factores lineales escribimos:

[𝑥 − (1 + √2 𝑖)][𝑥 − (1 − √2 𝑖)][𝑥 − (−1 + √2 𝑖)][𝑥 − (−1 − √2 𝑖)] = 0


[𝑥 − (1 − √2 𝑖)][𝑥 − 1 + √2 𝑖][𝑥 + 1 − √2 𝑖][𝑥 + 1 + √2 𝑖] = 𝑥 4 − 2𝑥 2 + 8𝑥 − 3 = 0

102
2.7.4. MÉTODO DE NEWTON

Existen pocos métodos directos para encontrar las raíces de una ecuación

f(x) = 0

Para ecuaciones polinomiales de grado cuatro o menor, puede resolverse siempre


la ecuación por medio de una fórmula que expresa las respuestas en términos de
los coeficientes de f(x). Se sabe, que ax2 + bx + c = 0, a ≠ 0, puede resolverse
por la formula cuadrática. Uno de los más grandes éxitos dentro de las
matemáticas fue la demostración de que las ecuaciones polinomiales de grado
mayor que cuatro no pueden resolverse por medio de alguna fórmula. Así que,
resolver la ecuación algebraica.

𝑥 5 − 3𝑥 2 + 4𝑥 − 6 = 0 … … … … … … … … .2.1

Plantea un serio aprieto, a menos que el polinomio se pueda factorizar. Además, a


menudo se requiere en análisis científico encontrar las raíces de ecuaciones
trascendentes tales como

𝑓(𝑥) = tan 𝑥 … … … … … … … … … … … … … … 2.2

En el caso de problemas como (2.1) y (2.2), es común emplear una técnica que da
lugar a una aproximación o estimación de las raíces. Uno de tales procedimientos,
conocido como el método de Newton, emplea la derivada de una función.

103
Una técnica iterativa

Supóngase que f es diferenciable y que c representa la raíz desconocida de


f(x) =0; esto es, f(c) = 0. Se denota por x0 un numero escogido arbitrariamente
como una primera aproximación a c. si f(x0) ≠ 0, se calcula f’(x0) y, como se
muestra en la figura 3.42, se traza una tangente a la grafica de f en (x0, f(x0)). Si
se denota ahora por x1 a la intersección x de esta recta, debe tenerse que
f(x0 )
Pendiente de la recta = f ′ (x0 ) =
(x1 −x0 )

Despejando x1 se obtiene

f(x0 )
x1 = x0 −
f′ (x0 )

Se repite el procedimiento en (x1, f(x1)) y se denota por x2 la intersección x de la


segunda recta tangente. De
𝑓(𝑥1 )
𝑓(𝑥1 ) =
(𝑥2 − 𝑥1 )

y
y = f(x)

Pendiente = f’(x0)

(x0, f(x0))
Pendiente = f’(x1)

(x1, f(x1))

x
x2 x1 x0

Figura 2.1

Se obtiene

f(x1 )
x2 = x1 −
f′ (x1 )

104
Continuando en esta forma, se determina xn+1 como

𝑓(𝑥𝑛 )
𝑥𝑛+1 = 𝑥𝑛 − … … … … … … … … … . .2.3
𝑓 ′ (𝑥𝑛 )

El uso reiterativo, o iterativo, de (2.3) da lugar a una sucesión x1, x2, x3,…de
aproximaciones que se espera que converja a la raíz c; esto es, xn → c cuando n
aumenta.

2.8. ANÁLISIS GRÁFICO

Antes de aplicar (2.3) tratemos de determinar la existencia y el número de raíces


reales de f(x) =0 de modo grafico; por ejemplo, el número irracional √3 se
puede interpretar ya sea como:

I. Una raíz de la ecuación cuadrática x2 – 3 = 0, y por lo tanto, como un cero


de la función continua f(x) = x2 – 3, o bien
II. La abscisa de un punto de intersección de las graficas de y = x2 y y = 3

Ambas interpretaciones se ilustran en la figura 2.2. Desde luego, otra razón para
emplear una grafica es que permite escoger la aproximación inicial x0 de modo que
este cerca de la raíz c.

Figura 2.2
105
Ejemplos:

1.- Determinar el número de raíces reales de x3 – x + 1 = 0.

Solución. De las graficas de y = x3 y y = x – 1 que se muestran en la Figura 2.4,


se concluye que la ecuación

x3 = x – 1

o bien

x3 - x + 1 = 0

Tiene solamente una raíz real.

y y = x3

y=x-1

c
x

Figura 2.4

Utilizamos el método de Newton para obtener una aproximación a la raíz real de


x3-x +1 = 0.
Solución. Sea f(x)=x3 - x + 1, de modo que f’(x)=3x2 - 1. Por lo tanto, (2.3) es

𝑥𝑛 3 − 𝑥𝑛 + 1 2𝑥𝑛 3 − 1
𝑥𝑛+1 = 𝑥𝑛 − =
3𝑥𝑛 2 − 1 3𝑥𝑛 2 − 1

106
Si nos interesa una precisión de tres y posiblemente cuatro decimales, puede
llevarse a cabo la iteración hasta que dos iterantes consecutivos coincidan en sus
cuatro primeros decimales. También, la Figura 2.4 nos sugiere tomar
x0 = -1.5 como la aproximación inicial. Consecuentemente,
2𝑥0 3 − 1 2(−1.5)3 − 1
𝑥1 = 2 = ≈ −1.3478
3𝑥0 − 1 3(−1.5)2 − 1
2𝑥1 3 − 1
𝑥2 = ≈ −1. −1.3252
3𝑥1 2 − 1
2𝑥2 3 − 1
𝑥3 = ≈ −1.3247
3𝑥2 2 − 1
2𝑥3 3 − 1
𝑥4 = ≈ −1.3247
3𝑥3 2 − 1

Por lo tanto, la raíz de la ecuación dada es aproximadamente -1.3247.

2.- Aproximar √3 por el método de Newton.

Solución. Si se define f(x) = x2 - 3, entonces f’(x) = 2x, y (2.3) se convierte en

𝑥𝑛 2 − 3 1 3
𝑥𝑛+1 = 𝑥𝑛 − = (xn + )
2xn 2 xn

Como 1 < √3 < 2 , parece razonable escoger x0 = 1. Así que,

1 3 1
𝑥1 = (𝑥0 + ) = (1 + 3) = 2
2 𝑥0 2
1 3 1 3
𝑥2 = (𝑥1 + ) = (2 + ) = 1.75
2 𝑥1 2 2
1 3 1 7 12
x3 = (x2 + ) = ( + ) ≅ 1.7321
2 x2 2 4 7
1 3
𝑥4 = (𝑥3 + ) ≈ 1.7321
2 𝑥3

Puesto que no hay diferencia significativa entre x3 y x4, se justifica detener la


iteración. En efecto, puede demostrarse que √3 ≈ 1.73205 es precisa hasta la
quinta cifra decimal.
107
Ejercicios 2.9
“PROBLEMAS PROPUESTOS”

Las respuestas a los problemas de número impar empiezan en la pagina


972. (Checar cuales el libro al que hace referencia)

En donde sea apropiado efectúe la iteración de (2.3) hasta que los iterantes
sucesivos coincidan en las primeras cuatro cifras decimales.

En los problemas 1-4 determine gráficamente si la ecuación tiene algunas raíces


reales.

1.- x 3 = −2 + sen x
2.- x 3 − 3x = x 2 − 1
3.- x 4 + x 2 − 2x + 3 = 0
4.- tan x = Cos x

En los problemas 5-8 use el método de Newton para encontrar aproximación al


número dado.

5.- √10
6.- 1 + √5
3
7.- √4
8.- √2
5

En los problemas 9-14 use el método de Newton, si es necesario, para encontrar


aproximaciones a todas las raíces reales de las ecuaciones dadas.

9.- 𝑥 3 = −𝑥 + 1
10.- 𝑥3 + 𝑥2 + 1 = 0
11.- 𝑥4 + 𝑥2 − 3 = 0
12.- 𝑥 4 = 2𝑥 + 1
13.- 𝑥 2 = 𝑠𝑒𝑛 𝑥
14.- 𝑥 + cos 𝑥 = 0

15.- Encontrar la menor intersección x positiva de la grafica de


𝑓(𝑥) = 3 cos 𝑥 + 4 .

16.- Considere la función f(x) = x5 + x2. Use el método de Newton para aproximar
al menor número positivo para el cual f(x) = 4.

108
17.- Una viga en voladizo (cantiléver) de 20 pies de longitud con una carga de 600
1b en su extremo, sufre una deflexión de magnitud d = (60 x2 – x3)/16, 000, en
donde d se mide en pulgadas y x en pies. Véase la figura 2.7. Use el método de
Newton para aproximar el valor de x que corresponde a una deflexión de 0.01 plg.

600 lb

20 pies

FIGURA 2.7

18.- una columna cilíndrica maciza vertical, de radio fijo r y que soporta su propio
peso, finalmente se pandeara cuando se incremente su altura. Puede demostrarse
que su altura máxima, o critica, de tal columna es hcr = kr2/3, en donde k es una
constante y r se mide en metros. Use el método de Newton para aproximar el
diámetro de una columna para la cual hcr = 10 m y k = 35 m.

19.- Un rayo de luz que emana de un punto P en un medio A, cuyo índice de


refracción es n1, incide en la superficie de un medio B, cuyo índice de refracción es
n2. Puede demostrarse, con base en la ley de Shell (Véase en el problema 45 de
los Ejercicios 4.7), que el rayo se refracta tangencialmente a la superficie cuando
el ángulo tiene el valor critico determinado por sen θc = n2/n1, 0<θ c<90°. Para
ángulos de incidencia mayores que el ángulo critico, toda la luz se refleja hacia el
interior del medio A. Véase la Figura 3.8. Si n2=1 para el aire y n1= 1.5 para el
vidrio, use el método de Newton para aproximar θc en radianes.

Medio A (n1)
P

θc θ>θc

Superficie
θ< θc

Medio B (n2)
Figura 2.8

109
20.- Para un puente colgante, la longitud s de un cable entre dos soportes
verticales, cuya separación es l (distancia horizontal) está relacionada con la
flecha d del cable mediante

8𝑑2 32𝑑4
𝑠=𝑙+ −
3𝑙 5𝑙 3

Véase la figura 2.9. Si s = 404 pie y l = 400 pie, use el método de Newton para
evaluar aproximadamente la flecha. Redondee su respuesta a una cifra decimal.*
(Indicación: la raíz c satisface 20 <c<30.)

Figura 2.9

Problemas diversos

21.- sea f una función diferenciadle. Demuestre que si f(x0) = -f(x1) y f’(x0) =
f’(x1), entonces (2.3) implica que x2= x0.

22.- Dada

𝑓(𝑥) = −√4 − 𝑥 → 𝑠𝑖, 𝑥 < 4


√𝑥 − 4 → 𝑠𝑖, 𝑥 ≥ 4

Obsérvese que f(4) = 0. Demuestre que para cualquier selección de x0 el método


de Newton no convergerá a la raíz. (Sugerencia: Véase el problema 21.)

23.- Demuestre que para encontrar la raíz r- ésima de un numero N, (2.3) se


convierte en

110
2.10. MÉTODO DE NEWTON PARA APROXIMAR LOS CEROS DE
UNA FUNCIÓN

Sea f(c) =0, donde f es derivable en un intervalo abierto que contiene a c.


Entonces, para aproximar c efectuar los siguientes pasos:

1. hacer una estimación x1 <<próxima>> al c.


2. determinar una nueva estimación

𝑓(𝑥𝑛 )
𝑥𝑛+1 = 𝑥𝑛 −
𝑓 ′ (𝑥𝑛 )

3. si |xn – xn+1| es menor que la precisión deseada, tómese xn+1 como


aproximación final. En caso contrario, volver al punto 2 y calcular una nueva
estimación.

Cada aproximación sucesiva se denomina una iteración.

Para muchas funciones, unas pocas iteraciones proporcionaran valores de precisión


sorprendente. Para comprobarlo, comenzamos con un ejemplo en el que
conocemos el valor exacto de cero.

Ejemplo 1: uso del método de Newton

Calcular tres iteraciones del método de Newton para aproximar un cero de


f(x) = x2 – 2. Tomar x0 = 1 como estimación inicial.

Solución: como f(x) = x2 – 2, tenemos f’(x) = 2x y el proceso de iteraciones viene


dado por la formula

𝑓(𝑥𝑛 ) 𝑥𝑛 2 −2
𝑥𝑛+1 = 𝑥𝑛 − = 𝑥𝑛 −
𝑓′ (𝑥𝑛 ) 2𝑥𝑛

La tabla 2.2 muestra los cálculos de tres iteraciones.

n xn f(xn) f'(xn) f(xn) / f'(xn) xn-( f(xn) / f'(xn))


1 1,000000 -1,000000 2,000000 -0,500000 1,500000
2 1,500000 0,250000 3,000000 0,083333 1,416667
3 1,416667 0,006945 2,833334 0,002451 1,414216
4 1,414216

Tabla 2.2
111
Obviamente en este caso sabemos que los dos ceros de f son ±√2 .
Con seis cifras decimales, √2 = 1.4142. Así pues, tras solo tres iteraciones del
método de Newton hemos obtenido una aproximación con un error no superior a
0,000002. La primera iteración de este proceso puede verse en la figura 2.10

x1=1

x2=1,5
x

-1

f(x)=x2-2
-2

Figura 2.10

Nota: De los cálculos del ejemplo 1, puede colegirse(o deducirse) que el método
de Newton es conveniente aplicarlo con ayuda de una calculadora.

Ejemplo 2: aplicando el método de Newton

Para aproximar los ceros de f(x) = 2x3 + x2 - x + 1, usar el método de Newton.


Continuar las iteraciones hasta lograr que dos aproximaciones sucesivas difieran en
menos de 0,0001.

Solución: Un esbozo de la grafica de f muestra que solo hay un cero y que está
próximo a x = -1, 2 (véase figura 2.11). Ahora bien, derivando f se obtiene para
la formula de iteración.

𝑓(𝑥𝑛 )
𝑥𝑛+1 = 𝑥𝑛 −
𝑓 ′ (𝑥𝑛 )

2𝑥𝑛 3 +𝑥𝑛 2 −𝑥𝑛 +1


= 𝑥𝑛 −
6𝑥𝑛 2 +2𝑥𝑛 −1

112
y

x
-1 1
f(x) = 2x3+x2-x+1
Figura 2.11

Los cálculos se muestran en la tabla 2.3

n xn f(xn) f'(xn) f(xn) / f'(xn) xn-( f(xn) / f'(xn))


1 -1,20000 0,18400 5,24000 0,03511 -1,23511
2 -1,23511 -0,00771 5,68276 -0,00136 -1,23375
3 -1,23375 0,00001 5,66533 0,00000 -1,23375
4 -1,23375

Tabla 2.3

Luego estimamos que el cero de f es -1,23375, dado que dos aproximaciones


sucesivas han diferido en menos que la cota prefijada 0,0001.

Cuando las aproximaciones tiendan a un límite, como en los ejemplos 1 y 2,


decimos que la sucesión

x1, x2, x3,…, xn,…

Converge. Además, si el límite es c, puede probarse que c es necesariamente un


cero de f.

Es importante observar que el método de Newton no siempre proporciona una


sucesión convergente. Hay dos dificultades posibles. La primera la sugiere la figura
2.12. Como el método de Newton requiere división por f’ (xn), está claro que el
método fallara si la derivada es cero para cualquier xn de la sucesión. Este
problema puede subsanarse con una elección adecuada de x1.

113
y

f’(x1)=0

x
x1

Figura 2.12

Más serio suele ser el problema que presenta el ejemplo siguiente. La figura 2.13
indica que el método de Newton no converge para ninguna elección del x0 inicial
(salvo si x0 es el cero exacto) para la función f(x) = x1/3.

F
Figura 2.13

114
Ejemplo 3: uso del método de Newton para hallar un punto de
intersección

Estimar el punto de intersección de las graficas de

y = tg x e y = 2x

Véase figura 2.14. Usar el método de Newton y continuar las iteraciones hasta que
dos sucesivas difieran en menos que 0,0001.

y y=tg x

-π/2 π/2

y=2x

Figura 2.14

Solución: poner tg x = 2x equivale a que 2x – tg x = 0, luego hemos de hallar los


ceros de la función
f(x) = 2x – tg x

Para ella, la formula iterativa de Newton da

𝑓(𝑥𝑛 )
𝑥𝑛+1 = 𝑥𝑛 −
𝑓 ′ (𝑥𝑛 )

2xn − tgxn
= xn −
2 − sec 2 xn

Los cálculos aparecen en la tabla 2.5, comenzando con x1 = 1,25.

115
n xn f(xn) f'(xn) f(xn) / f'(xn) xn-( f(xn) / f'(xn))
1 1,25000 -0,50957 -8,05751 0,06324 1,18676
2 1,18676 -0,10110 -5,12374 0,01973 1,16703
3 1,16703 -0,00653 -4,47831 0,00146 1,16557
4 1,16557 -0,00003 -4,43436 0,00001 1,16556

Tabla 2.5
Así pues, obtenemos como aproximación del punto de intersección 𝑥 = 1.1655.

116
FRACCIONES PARCIALES.

Al trabajar con fracción, hasta ahora se ha combinado dos o más de ellas por
medio de las cuatro operaciones fundamentales del álgebra.
Pero muchas veces, especialmente en el cálculo integral, se hace necesario
expresar como la suma de otras dos o más, de forma más sencilla que la fracción
original. Las fracciones que se obtienen de esta manera se llaman fracciones
parciales.

DEFINICIONES.

Una fracción parcial es el cociente de dos polinomios. Aquí trataremos únicamente


las FRACCIONES PROPIAS, es decir a aquellas que en las cuales el numerador es
de grado inferior que el denominador.

Según lo que hemos visto en temas anteriores, sabemos que: Todo polinomio se
puede expresar como producto repetidos o distintos factores de primer grado,
por factores irreductibles de segundo grado, todos ellos con coeficientes reales.

Por lo tanto, toda fracción racional pertenece a los cuatro casos siguientes:

1. Todos los factores del denominador son de primer grado y ninguno de ellos
se encuentran repetidos.
2. Todos los factores del denominador son de primer grado y algunos de ellos
se encuentran repetidos.
3. El denominador contiene factores irreductibles de segundo grado, ninguno
de los cuales se encuentra repetido.
4. El denominador contiene factores irreductibles de segundo grado, algunos
de los cuales se encuentran repetidos.

TEOREMA:

Si una fracción racional propia reducida su mínima expresión, se expresa como la


suma de fracciones parciales, como se describe en los casos siguientes:

a) A todo factor (ax + b) del denominador, que no aparezcan repetidos corresponde


una fracción parcial
𝐴
(𝑎𝑥 + 𝑏)

117
b) A todo factor (ax+b) del denominador corresponden las fracciones
parciales.

𝐀𝟏 𝐀𝟐 𝐀𝐤
+ + ⋯+
(𝐚𝐱 + 𝐛) (𝐚𝐱 + 𝐛)𝟐 (𝐚𝐱 + 𝐛)𝐤

En donde A1 A2……………..Ak. Son constantes.

c) A todo factor irreductible de segundo grado (𝐚𝐱 𝟐 + 𝐛𝐱 + 𝐜) del


denominador que no aparezca repetido corresponde la fracción parcial
(𝑨𝒙+𝑩)
en donde A y B son constantes.
(𝒂𝒙𝟐 +𝒃𝒙+𝒄)

d) Si (𝐚𝐱 𝟐 + 𝐛𝐱 + 𝐜) es irreductible, entonces a todo factor (𝐚𝐱 𝟐 +


𝐛𝐱 + 𝐜)𝐤 del denominador corresponden a las fracciones parciales.

𝑨𝟏 𝒙 + 𝑩 𝟏 𝑨𝟐 𝒙 + 𝑩 𝟐 𝑨𝒌 𝒙 + 𝑩 𝒌
+ + ⋯+
𝒙𝟐 + 𝒃𝒙 + 𝒄 (𝒂𝒙𝟐 + 𝒃𝒙 + 𝒄)𝟐 (𝒂𝒙𝟐 + 𝒃𝒙 + 𝒄)𝒌

Donde A1 A 2………..Ak B1 B2….Bk son constantes.

A continuación se presentaran los ejemplos para los casos de las fracciones


parciales que se planteo en teorema anterior.

𝟐𝐱 𝟐 +𝐱+𝟏
Caso a)
(𝐱+𝟐)(𝟑𝐱+𝟏)(𝐱+𝟑)

Haciendo operaciones y utilizando la igualdad entre polinomios tendremos:

2x 2 + x + 1 = (3A + B + 3C)x 2 + (10A + 5B + 7C) + (3A + 6B + 2C) → (A)

Haciendo las igualdades tendremos las siguientes ecuaciones.

3A + B + 3C = 2
10A + 5B + 7C = 1
3A + 6B + 2C = 1

118
Resolviendo por matrices, determinantes o cualquier otro método obtenemos los
−7 1
siguientes resultados 𝐴 = 𝐵= 𝑦 𝐶 = 2.
5 5
Por lo que la solución de la fracción es:

2x 2 + x + 1 −7 1 2
= + +
(x + 2)(3x + 1)(x + 3) 5(x + 2) 5(3x + 1) (x + 3)

Otro método de obtener los valores de A, B Y C. es que a partir de la ecuación A


se puede hacer que B=2, sustituyéndolo en dicha ecuación observamos que, los
7
dos últimos términos se anulan, por lo que A = .
5
De la misma manera si hacemos que x=-3, en la ecuación A se anulan los dos
primeros términos y por lo tanto C=2.
1 1
Análogamente si hacemos que x = − entonces B =
8 5

𝟑𝐱 𝟐 +𝟓𝐱+𝟏
Caso b).
(𝐱−𝟏)(𝐱+𝟐)𝟐

Siguiendo el teorema para este caso se obtendrá:

3𝑥 2 + 5𝑥 + 1 𝐴 𝐵 𝐶
= + +
(𝑥 − 1)(𝑥 + 2)2 (𝑥 − 1) (𝑥 + 2) (𝑥 + 2)2

Resolviendo e igualando.

3x 2 + 5x + 1 = (A + B)x 2 + (4A + B + C)x + (4A − 2B − C)

Entonces:

A+B= 3
4A + B + C = 5
4A − 2B − C = 1

Cuya solución por cualquiera de los métodos es:


A=1 B=2 y C=-1

119
Por lo que la solución de la fracción es:

3𝑥 2 + 5𝑥 + 1 1 2 −1
= + +
(𝑥 − 1)(𝑥 + 2)2 (𝑥 − 1) (𝑥 + 2) (𝑥 + 2)2

𝟏𝟒𝐱 𝟑 +𝟏𝟒𝐱 𝟐 −𝟒𝐱+𝟑


Caso c).
(𝟑𝐱 𝟐 −𝐱+𝟏)(𝐱−𝟏)(𝐱+𝟐)

Aplicando el teorema para el caso que nos ocupa tendremos.

14x 3 + 14x 2 − 4x + 3 Ax + B C D
= + +
(3x 2 − x + 1)(x − 1)(x + 2) (3x 2 − x + 1) (x − 1) (x + 2)

Resolviendo e igualando términos se obtiene.

𝟏𝟒𝐱𝟑 + 𝟏𝟒𝐱𝟐 − 𝟒𝐱 + 𝟑
= (𝐀 + 𝟑𝐂 + 𝟑𝐃)𝐱 𝟑 + (𝐀 + 𝐁 + 𝟓𝐂 − 𝟒𝐃)𝐱 𝟐 + (−𝟐𝐀 + 𝟐𝐂 − 𝐃)𝐱 + (−𝟐𝐁 + 𝟐𝐂 − 𝐃)

De la ecuación anterior se obtiene las ecuaciones lineales siguientes:


A + 3C + 3D = 14
A + B + 5C − 4D = 14
−2A + B − C + 2D = −4
−2B + 2C − D = 3

La solución de este sistema usando los métodos conocidos son:

A=2 B=1 C=3 yD=1

Por lo que la solución de la fracción es:

14x 3 + 14x 2 − 4x + 3 2x + 1 3 1
2
= 2
+ +
(3x − x + 1)(x − 1)(x + 2) (3x − x + 1) (x − 1) (x + 2)

120
𝟔𝐱 𝟒 +𝟏𝟏𝐱 𝟑 +𝟏𝟖𝐱 𝟐 +𝟏𝟒𝐱+𝟔
Caso d)
(𝐱+𝟏)(𝐱 𝟐 +𝐱+𝟏)𝟐

6x 4 + 11x 3 + 18x 2 + 14x + 6 A Bx + c Dx + E


= + +
(x + 1)(x 2 + x + 1)2 (x + 1) (x 2 + x + 1) (x 2 + x + 1)2

6x 4 + 11x 3 + 18x 2 + 14x + 6


= (A + B)x 4 + (2A + 2B + C)x 3 + (3A + 2B + 2C + D)x 2 + (2A + B + 2C + D + E)x
+ (A + C + E)

El sistema de ecuaciones lineales es:

A+B= 6
2A + 2B + C = 11
3A + 2B + 2C + D = 18
2A + B + 2C + D + E = 14
A+C+E= 6

Cuya solución resulta ser:

A=5 B=1 C = −1 D=3 Y E=2

De aquí que el resultado de la fracción es:

6x 4 + 11x 3 + 18x 2 + 14x + 6 5 x+1 3x + 2


= + + L. Q. Q. D.
(x + 1)(x 2 + x + 1)2 (x + 1) (x 2 + x + 1) (x 2 + x + 1)2

121
EJERCICIO 90: DESCOMPOSICIÓN DE FRACCIONES EN
FRACCIONES PARCIALES; CASO I.
Descompóngase las siguientes fracciones en fracciones parciales.
2 9
1: 2:
(𝑥−1)(3𝑥−2) (2𝑥+3)(𝑥+3)

1 −7
3: 4:
(3𝑥+7)(2𝑥+5) (3𝑥−2)(4𝑥−5)

7𝑥 −8𝑥
5: 6:
(2𝑥+1)(𝑥−3) (3𝑥+5)(𝑥+3)

3𝑥 14𝑥
7: 8:
(5𝑥−8)(3𝑥−4) (4𝑥−9)(𝑥+3)

𝑥−8 6𝑥+9
9: 10:
(𝑥−3)(𝑥−4) (2𝑥−5)(2𝑥−1)

15−7𝑥 𝑥−7
11: 12:
(3𝑥−7)(2𝑥−5) (5𝑥−3)(13𝑥−11)

−4𝑥−23 61𝑥−1
13: 14:
(2𝑥+1)(𝑥+2)(𝑥−3) (3𝑥−2)(𝑥+5)(2𝑥+1)

14𝑥−106 −73𝑥−77
15: 16:
(𝑥+1)(3𝑥−5)(2𝑥+7) (2𝑥+3)(𝑥−5)(3𝑥+2)

6𝑥 2 −4𝑥+31 −2𝑥 2 −22𝑥−24


17: 18:
(2𝑥−3)(𝑥+4)(3𝑥−1) (𝑥+6)(2𝑥+3)(𝑥+2)

−4𝑥 2 −25𝑥+41 𝑥 2 +62𝑥−114


19: 20:
(𝑥+4)(4𝑥−5)(2𝑥−3) (𝑥−5)(2𝑥+7)(3𝑥−2)

3𝑥 2 −9𝑥−20 2𝑥 2 −9𝑥−44
21: 22:
𝑥 2 −2𝑥−3 2𝑥 2 −7𝑥−15

6𝑥 3 −7𝑥 2 −16𝑥+27 45𝑥 3 −42𝑥 2 +20𝑥−1


23: 24:
6𝑥 2 −7𝑥−5 15𝑥 2 −14𝑥+3

122
EJERCICIO 91: DESCOMPOSICIÓN DE FRACCIONES EN
FRACCIONES PARCIALES; CASO II.

Descompóngase las siguientes fracciones en fracciones parciales.

2x+5 −2x+11
1: 2:
(x+1)2 (x−4)2

3x+1 −6x−11
3: 4:
(3x−1)2 (2x+5)2

−4x2 −5x+1 −16x2 +54x−40


5: 6:
(x+1)2 (2x−3)2

27x2 +21x+8 20x2 −144x+265


7: 8:
(3x+2)2 (2x−7)2

−8x2 +35x+9 12x2 −9x+20


9: 10:
(2x−1)(x−4)2 (2x+3)(3x−1)2

−22x2 +32x+138 24x2 −94x+88


11: 12:
(5x+2)(2x−7)2 (2x−3)(3x−5)2

−14x3 +14x2 +x−7 2x3 −5x2 −27x−24


13: 14:
(2x+1)(x−4)(x+1)2 (x+1)(x−1)(x+2)2

4x3 −42x2 +78x−30 −13x3 −11x2 +131x+189


15: 16:
(2x+1)(x−2)(2x−3)2 (2x−3)(3x−2)(x+5)2

2x2 +5x+1 3x2 −16x+20


17: 18:
(x+1)2 (x−3)2

2x3 −10x2 +4x+11 6x3 +33x2 +23x−45


19: 20:
(2x+1)(x−2)2 (2x−1)(x+3)2

123
EJERCICIO 92: DESCOMPOSICIÓN DE FRACCIONES EN
FRACCIONES PARCIALES: CASO III

Descompónganse las siguientes fracciones en fracciones parciales.

−2x+5 5x2 +x+2


1: 2:
(x−1)(x2 +2) (x+1)(x2 +1)

3x2 −10x+16 x2 −4x−3


3: 4:
(x−3)(x2 +x+1) (2x−3)(x2 −x−3)

5x3 −6x2 +12x+13 5x3 +14x2 +71x−14


5: 6:
(x−1)(2x+1)(x2 +3) (x+5)(2x−1)(x2 −3)

x3 +11x2 +13x−5 −x3 −9x2 −4x−18


7: 8:
(x−3)(3x+1)(x2 −x+2) (x+2)(x+3)(3x2 −2x+1)

3x2 −10x2 +9x−6 −x3 +8x2 +2x−19


9: 10:
(x−1)2 (x2 +1) (x−2)2 (x2 +5)

4x3 −11x2 +12x−19 4x3 +19x2 +18x−13


11: 12:
(x−2)2 (x2 +1) (x+3)2 (x2 +2x−1)

2x2 −8x+4 3x3 −10x2 +7x−3


13: 14:
(x2 +2)(x2 −2) (x2 +1)(x2 −3x−1)

3x3 −3x2 −4x−14 −x3 −13x2 +9


15: 16:
(x2 −5)(x2 −x+3) (x2 +3)(x2 −3x−3)

2x3 +2x2 2x+4 6x3 −9x2 +33x−64


17: 18:
(x+1)(x2 +1) (2x−3)(x2 +5)

2x3 −10x2 +11x+19 12x+14x2 +8x+39


19: 20:
(x−5)(2x2 −3x+2) (2x+3)(2x2 −x+3)

124
EJERCICIO 93: DESCOMPOSICIÓN DE FRACCONES
PARCIALES; CASO IV.

Descompónganse las siguientes fracciones en fracciones parciales.

x3 +2x2 −3 2𝑥 3 −3𝑥 2 +3𝑥−10


1: 2:
(x2 −2)2 (𝑥 2 +3)2

2𝑥 3 −𝑥 2 −6𝑥−8 𝑥 3 −7𝑥 2 +7𝑥+8


3: 4:
(𝑥 2 −𝑥−3)2 (𝑥 2 −2𝑥−1)2

𝑥 3 +2𝑥 2 −𝑥+3 𝑥 3 +4𝑥−5


5: 6:
(𝑥 2 +1)2 (𝑥 2 +2)3

𝑥 4 +2𝑥 3 −3𝑥 2 −3𝑥 𝑥 5 +4𝑥 4 +6𝑥 3 +𝑥 2 −3𝑥−8


7: 8:
(𝑥 2 +𝑥−1)3 (𝑥 2 +2𝑥+2)2

2𝑥 4 +13𝑥 3 +13𝑥 2 −12𝑥+2 2𝑥 4 −𝑥 3 +6𝑥 2 −𝑥+8


9: 10:
𝑥 2 (𝑥 2 +3𝑥−1)2 𝑥(𝑥 2 +2)2

−2𝑥 5 −4𝑥 4 +12𝑥 3 +9𝑥 2 −6𝑥+1 2𝑥 6 −𝑥 5 −8𝑥 4 +5𝑥 3 +9𝑥 2 +𝑥−1


11: 12:
𝑥 2 (𝑥 2 +3𝑥−1)2 𝑥 2 (𝑥 2 −𝑥−1)2

4𝑥 4 +𝑥 3 −25𝑥 2 −9𝑥+30 3𝑥 4 +2𝑥 3 +8𝑥 2 +7𝑥+7


13: 14:
(𝑥+2)(𝑥 2 −3)2 (𝑥−1)(𝑥 2 +2)2

4𝑥 4 −7𝑥 3 +5𝑥 2 −𝑥+1 𝑥 4 −5𝑥 3 −10𝑥 2 +26𝑥+33


15: 16:
(2𝑥−1)(𝑥 2 −𝑥+1)2 (𝑥+3)(𝑥 2 −𝑥−3)2

125
126
Ejercicios:

1.- Para cada una de las siguientes expresiones indicar si son polinomios o no lo
son. En este caso afirmativo indicar su grado.

𝑎) 1 + 𝑥 2 = 𝑠𝑖 𝑒𝑠 𝑢𝑛 𝑝𝑜𝑙𝑖𝑛𝑜𝑚𝑖𝑜, 𝑔𝑟𝑎𝑑𝑜 2
1
𝑏) − √3𝑡 2 + 5𝑡 3 + 0𝑡 4 = 𝑠𝑖 𝑒𝑠 𝑝𝑜𝑙𝑖𝑛𝑜𝑚𝑖𝑜, 𝑔𝑟𝑎𝑑𝑜 3
2
1
𝑐) 2𝑍 − 2 + 4𝑍 3 = 𝑠𝑖 𝑒𝑠 𝑝𝑜𝑙𝑖𝑛𝑜𝑚𝑖𝑜, 𝑔𝑟𝑎𝑑𝑜 5
𝑍
𝑑) 3 + 𝑥 𝑠𝑒𝑛 𝑥 − 4𝑥 2 + 𝑠𝑒𝑛 3 𝑥 = 𝑛𝑜 𝑒𝑠 𝑢𝑛 𝑝𝑜𝑙𝑖𝑛𝑜𝑚𝑖𝑜
𝑒) − 2 + 𝑥𝑠𝑒𝑛𝑥 − 5𝑥 3 𝑠𝑒𝑛3 𝑥 = 𝑛𝑜 𝑒𝑠 𝑢𝑛 𝑝𝑜𝑙𝑖𝑛𝑜𝑚𝑖𝑜
𝑓) 𝐿𝜃 −4 + (𝐿𝜃)3 = 𝑠𝑖 𝑒𝑠 𝑢𝑛 𝑝𝑜𝑙𝑖𝑛𝑜𝑚𝑖𝑜, 𝑔𝑟𝑎𝑑𝑜 5 𝑒𝑛 𝜃 𝑦 𝑔𝑟𝑎𝑑𝑜 3 𝑒𝑛 𝐿

2.- 𝑆𝑖 𝑓(𝑥) = 1 + 𝛼 2 x 2 − 2x 3 + βx 4 y g(x) = 1 − γ𝑥 + 3𝑥 2 + 2𝛿 2 𝑥 3


Determinar para que valores de α, β, γ, δ ε C

a) f(x)=g(x)
f(x) = g(x) = 1 + 𝛼 2 x 2 − 2x 3 + βx 4 = 1 − γ𝑥 + 3𝑥 2 + 2𝛿 2 𝑥 3

Igualando término a término tendremos que:


𝛼 2 = 3 𝑒𝑛𝑡𝑜𝑛𝑐𝑒𝑠 𝛼 = √3
−2 = 2𝛿 2 𝑒𝑛𝑡𝑜𝑛𝑐𝑒𝑠 𝛿 = √−1 = 𝑖
𝛽 = 0 𝑝𝑢𝑒𝑠𝑡𝑜 𝑞𝑢𝑒 𝑛𝑜 ℎ𝑎𝑦 𝑡é𝑟𝑚𝑖𝑛𝑜 𝑒𝑛 𝑒𝑙 𝑜𝑡𝑟𝑜 𝑚𝑖𝑒𝑚𝑏𝑟𝑜.
𝛾 = 0 𝑝𝑢𝑒𝑠𝑡𝑜 𝑞𝑢𝑒 𝑛𝑜 ℎ𝑎𝑦 𝑡é𝑟𝑚𝑖𝑛𝑜 𝑒𝑛 𝑒𝑙 𝑜𝑡𝑟𝑜 𝑚𝑖𝑒𝑚𝑏𝑟𝑜.

b) gr(f)=3

Si 𝑓(𝑥) = 1 + 𝛼 2 x 2 − 2x 3 + βx 4

Como f(x) deber ser grado 3 entonces β necesariamente debe ser cero, por lo
tanto 𝑓(𝑥) = 1 + 𝛼 2 x 2 − 2x 3

c) gr(g)=1
Esta condición no la puede cumplir.
Puesto que δ=0 queda de grado 2 y si x=0 queda de grado 0.

127
3.-Demostrar que si f(x), g(x) y h(x) son polinomios en x con coeficientes en C
entonces.

a) 𝑓(𝑥) + [𝑔(𝑥) + ℎ(𝑥)] = [𝑓(𝑥) + 𝑔(𝑥)] + ℎ(𝑥)

𝑓(𝑥) + 𝑔(𝑥) + ℎ(𝑥) = 𝑓(𝑥) + 𝑔(𝑥) + ℎ(𝑥)

𝑆𝑖 𝑓(𝑥) = 𝑥1 + 𝑥2 𝑖
𝑔(𝑥) = 𝑥3 + 𝑥4 𝑖
ℎ(𝑥) = 𝑥5 + 𝑥6 𝑖

(x1 + x2 i) + [(x3 + x4 i) + (x5 + x6 i)] = [(x1 + x2 i) + (x3 + x4 i)] + (x5 + x6 i)


x1 + x2 i + [x3 + x4 i + x5 + x6 i] = [x1 + x2 i + x3 + x4 i] + (x5 + x6 i)
x1 + x2 i + x3 + x4 i + x5 + x6 i = x1 + x2 i + x3 + x4 i + x5 + x6 i
f(x) + g(x) + h(x) = f(x) + g(x) + h(x)

b) Existe un polinomio –f(x) tal que f(x)+[-f(x)]=φ(x)


𝑓(𝑥) − 𝑓(𝑥) = 𝜑(𝑥)
𝜑(𝑥) = 𝜑(𝑥)

4.- Demostrar que sí.

𝑛 𝑛

𝑓(𝑥) = ∑ 𝑎𝑘 𝑥 𝑘 𝑦 𝑔(𝑥) = ∑ 𝑏𝑘 𝑥 𝑘
𝑘=0 𝑘=0
𝑛

𝑓(𝑥) − 𝑔(𝑥) = ∑(𝑎𝑘 − 𝑏𝑘 )𝑥 𝑘


𝑘=0

Entonces:
𝑛

𝑓(𝑥) − 𝑔(𝑥) = ∑(𝑎𝑘 − 𝑏𝑘 )𝑥 𝑘


𝑘=0
𝑛 𝑛 𝑛

∑ 𝑎𝑘 𝑥 𝑘 − ∑ 𝑏𝑘 𝑥 𝑘 = ∑(𝑎𝑘 − 𝑏𝑘 )𝑥 𝑘
𝑘=0 𝑘=0 𝑘=0

n n

∑(ak − bk )x k = ∑(ak − bk )x k
k=0 k=0

128
5.-Para los polinomios

𝑓(𝑥) = −2 + 2𝑥 − 𝑥 2

𝑔(𝑥) = 2𝑥 − 𝑥 2

ℎ(𝑥) = 1 − 3𝑥 2 − 𝑥 3

Hallar el polinomio p(x) tal que:

a) 𝑓(𝑥) + 𝑝(𝑥) = 0
𝑝(𝑥) = −𝑓(𝑥)
𝑝(𝑥) = 2 − 2𝑥 + 𝑥 2

b) 𝑝(𝑥)[3𝑔(𝑥) − ℎ(𝑥)] = −1 + 6𝑥 + 𝑥 3

−1 + 6𝑥 + 𝑥 3
𝑝(𝑥) =
3𝑔(𝑥) − ℎ(𝑥)
−1 + 6𝑥 + 𝑥 3
𝑝(𝑥) =
3(2𝑥 − 𝑥 2 ) − (1 − 3𝑥 2 − 𝑥 3 )
−1 + 6𝑥 + 𝑥 3
𝑝(𝑥) =
6𝑥 − 3𝑥 2 − 1 + 3𝑥 2 + 𝑥 3
−1 + 6𝑥 + 𝑥 3
𝑝(𝑥) =
−1 + 6𝑥 + 𝑥 3
𝑝(𝑥) = 1

c) 𝑝(𝑥)𝑓(𝑥) = 𝑝(𝑥)𝑔(𝑥) + 𝑓(𝑥) ▫ ℎ(𝑥) + 𝑔(𝑥)


𝑝(𝑥)[𝑓(𝑥) − 𝑔(𝑥)] = 𝑓(𝑥)ℎ(𝑥) + 𝑔(𝑥)
𝑓(𝑥)ℎ(𝑥) + 𝑔(𝑥)
𝑝(𝑥) =
𝑓(𝑥) − 𝑔(𝑥)
(−2 + 2𝑥 − 𝑥 2 )(1 − 3𝑥 2 − 𝑥 3 ) + 2𝑥 − 𝑥 2
𝑝(𝑥) =
−2 + 2𝑥 − 𝑥 2 − 2𝑥 + 𝑥 2
−2 + 6𝑥 2 + 2𝑥 3 + 2𝑥 − 6𝑥 3 − 2𝑥 4 − 𝑥 2 + 3𝑥 4 + 𝑥 5 + 2𝑥 − 𝑥 2
𝑝(𝑥) =
−2
−2 + 4𝑥 2 − 4𝑥 3 + 4𝑥 + 𝑥 4 + 𝑥 5
𝑝(𝑥) =
−2
−2 + 4𝑥 + 4𝑥 2 − 4𝑥 3 + 𝑥 4 + 𝑥 5
𝑝(𝑥) =
−2

129
6.- Dados f(x)=x 4 + 2x 3 + x 2 − 1 y g(x) = x 2 + 3

a) Obtener dos polinomios q(x) y r(x) , tales que f(x) = g(x)q(x) + r(x)

f(x)
= q(x) + r(x)
g(x)

x 2  2x  2
x  3 x  2x3  x 2  1
2 4

 x4  3x 2
2x 3  2x 2  1
 2x3  6x
 2x 2  6x  1
2x 2  6
 6x  5

f(x) = (x 2 + 3)(x 2 + 2x − 2) + (−6x + 5)


f(x) = x 4 + 2x 3 − 2x 2 + 3x 2 + 6x − 6 − 6x + 5
f(x) = x 4 + 2x 3 + x 2 − 1 L.Q.Q.D.

7.- Si q(x) = x 2 hallar r(x)tal que f(x) = g(x)q(x) + r(x)

f(x)
= q(x) + r(x)
g(x) 𝑓 𝑥
x2
x 2  2x x 4  2x3  x  1 q(x)

 x(4 )
g(x) 2x3  x  1
 2x3
x 1
r(x)

f(x) = (x 2 + 2x)(x 2 ) + (x − 1)
f(x) = x 4 + 2x 3 + x − 1
x 4 + 2x 3 + x − 1 = 𝐋. 𝐐. 𝐐. 𝐃.

130
8.- Hallar los valores de A y B tales que el polinomio

p(x) = 2x 4 − x 3 + Ax 2 + 2x + B = 0 y sea divisible entre 𝑥 + 1 𝑦 2𝑥 − 3 a la vez.

Despejamos de la ecuación 𝑥 + 1 y 2x − 3
3
𝑥 = −1 𝑥=
2

2 −1 A 2 B
−1 −2 3 −3−A 1+A
2 −3 3+A −1−A 1+A+B
3 9+3𝐴
3 0
2 2
7+A
2 0 3+A 2
=0

1+𝐴+𝐵 =0
7 A 𝐴 7 7
+ =0 =− 𝐴 = − (2)
2 2 2 2 2

𝐴 = −7

1 + (−7) + B = 0 → 1 − 7 + B = 0
𝐵=6 2𝑥 4 − 𝑥 3 − 7𝑥 2 + 2𝑥 + 6 = 0

Comprobación

2 1 7 2 6
1 2 3 4 6
2 3 4 6 0
3
2
3 0 6
2 0 4 0
L.Q.Q.D.

131
9.- Que condiciones debe cumplir a, b y c para que el polinomio

x 3 + ax + b = 0 tenga como factor x 2 + cx − 1 = 0

xc
x 2  cx  1 x 3  ax  b
 x 3  cx 2  x
cx 2  x  ax  b
 cx 2  c 2 x  c
c 2 x  x  ax  b  c

(c 2 + 1 + a)x + b − c
(c 2 + a + 1) = 0 → a = −1 − c 2
b−c=0→b=c

𝑆𝑖 𝑐 = −4
𝑏 = −4
𝑎 = −17

x4
x 2  4 x  1 x 3  17 x  4
 x3  4x 2  x
4 x 2  16 x  4
 4 x 2  16 x  4
0

10.- Si p(x) = −2 + 3x + 4x 2 + x 3

a) Empleando el teorema del residuo. Calcule P (-3)

x2  x
x  3 x  4 x  3x  2
3 2

 x 3  3x 2
x 2  3x  2 Residuo: -2
 x  3x
2

2

132
11.- Empleando el teorema del factor determine si p(x) es divisible entre x+2.
X=-2
P(−2) = −2 + 3x + 4x 2 + x 3 = 0
P(−2) = −2 + 3(−2) + 4(−2)2 + (−2)3 = 0
P(−2) = −2 − 6 + 16 − 8 = 0
P(−2) = −16 + 16 = 0
P(−2) = 0

12.-Utilizando la división sintética divide el polinomio 9𝑥 4 + 8𝑥 2 − 1 entre:

a) x+1

9 0 8 0 1
1 9 9  17 17
9 9 17  17 16

16
9x 3 − 9x 2 + 17x − 17 + =0
x+1

b) x+i

9 0 8 0 1
i  9i 9 i 1
9  9i 1 i 0

9x 3 − 9i x 2 − x + i = 0

1
c) x − 3

9 0 8 0 1
1
3 1 3 1
3
9 3 9 3 0

9𝑥 3 + 3𝑥 2 + 9𝑥 + 3 = 0
133
13.- Utilizando la división sintética resolver el ejercicio.

𝑥 3 + 4𝑥 2 + 3𝑥 − 2

a) x=-3

1 4 3 2
3 3 3 0
1 1 0 2

2
x2 + x − =0
x+3

b) x=-2

1 4 3 2
2 2 4 2
1 2 1 0

x 2 + 2x − 1 = 0

14.- Determinar si los números 0,-2, 3/2, i son raíces del polinomio.

𝑝(𝑤) = 2𝑤 4 − 𝑤 3 − 𝑤 2 − 𝑤 − 3

2 1 1 1 3
0 0 0 0 0
2 1 1 1 3
No es raíz

134
2 1 1 1 3
2 4 10  18 38
2 5 9  19 35 No es raíz

2 1 1 1 3
3
3 3 3 3
2
2 2 2 2 0 Si es raíz

2 1 1 1 3
i 2i  2  i 1  3i 3
2 2i  1  3  i  3i 0
Si es raíz

15.- Hallar los valores de K ε Z tales que el número 1 sea una raíz del polinomio.

2𝑥 4 − 𝑘𝑥 3 + 𝑘𝑥 2 − 𝑘𝑥 + 3

2 k k k 3
1 2 2k 2 2k
2 2k 2 2k 5k  0
K=5

Como el residuo es 5-k este debe igualarse a cero (5-k=0) de donde se concluye
que k=5.

135
Comprobación para cuando k=5

2 5 5 5 3
1 2 3 2 3
2 3 2 3 0
L.Q.Q.D.

16.- Determinar los valores de A, B, C.de tal manera que se cumpla la siguiente
ecuación

A(2x + 1) + (Bx + C)(2x − 1) = (x + 1)(4x + 3)

Solución:
2Ax + A + 2Bx 2 − Bx + 2Cx − C = 4x 2 + 7x + 3
Agrupando
x 2 (2B) + x(2A − B + 2C) + A − C = 4x 2 + 7x + 3
Igualando términos semejantes de ambos miembros.
2B=4 B=2
2A − B + 2C = 7 2A − 2 + 2C = 7
A−C= 3 2A − 2C = 6

4A=15

15
𝐴=
4

C= A−3
15
C= −3
4
3
C=
4

COMPROBANDO

30 6 15 3
x 2 (4) + [x ( ) − 2 + ( )] + − = 4x 2 + 7x + 3
4 4 4 4

4x 2 + 7x + 3 = 4x 2 + 7x + 3

136
Resolviendo por determinantes el problema anterior tendremos

2B=4
2A − B + 2C = 7
A−C= 3

0 2 0 0 2
∆= 2  1 2 2 1 4+4=8
1 0 1 1 0

4 2 0 4 2
∆A= 7  1 2 7 1 16+14=30
3 0 1 3 0

0 4 0 0 4
∆B= 2 7 2 2 7 8+8=16
1 3 1 1 3

0 2 4 0 2
∆C= 2  1 7 2  1 14+4-12=6
1 0 3 1 0

Entonces

30 15 16 6 3
𝐴= = 𝐵= =2 𝑦 𝐶= =
8 4 8 8 4

17.- Determinar los valores de A, B, C. si se cumple la igualdad siguiente:

A(3x − 5) + B(2x − 1) + C(x 2 ) = 6 − 5x

Solución

3Ax − 5A + 2Bx − B + Cx 2 = 6 − 5x

𝑥 2 (𝐶) + 𝑥(3𝐴 + 2𝐵) − 5𝐴 − 𝐵 = 6 − 5𝑥

𝐶=0

137
3A + 2B = −5 3A + 2B = −5

−5A − B = 6 −10A − 2B = 12

−7A =7

A = −1 → −5(−1) − B = 6
5−B=6
−B = 6 − 5 → −B = 1 → B = −1

Resolver por determinantes

3 2 −5 2
∆= | |=7 ∆A = | | = −7
−5 −1 6 −1

3 −5
∆𝐵 = | | = −7
−5 6

∆A −7
= = −1
∆ 7

∆B −7
= = −1
∆ 7

A =-1; B=-1; C=0

18.- Encuentre la cota superior e inferior para las raíces reales de la ecuación:
2x 3 − 5x 2 − 7x + 4 = 0

Solución:

138
Como el -2 hace que los números del tercer renglón se alternen en signo, este
número es una cota inferior y con el 4 no hay cambio de signo entonces es una
cota superior. En consecuencia todas las raíces de la ecuación dada están entre -2
y 4.

19.-Para el polinomio P(x) dado por:

P(x) = x 6 − 2x 5 + 4x 4 − 14x 3 + 29x 2 − 20x + 50

a) Hallar una tabla de las posibles raíces, usando la regla de los signos de
Descartes.
b) Escribir el polinomio como un producto de n-factores lineales, sabiendo que
2 + 𝑖 es una raíz.

Solución:
a.-

𝑃(−𝑥) = (−𝑥)6 − 2(−𝑥)5 + 4(−𝑥)4 − 14(−𝑥)3 + 29(−𝑥)2 − 20(−𝑥) + 50 = 0


P(−x) = x 6 + 2x 5 + 4x 4 + 14x 3 + 29x 2 + 20x + 50 = 0

RR (+) 6 4 2 0
RR (-) 0 0 0 0
RC 0 2 4 6
RT 6 6 6 6

b) Escribir el polinomio como un producto de n-factores lineales, sabiendo que 2 +


𝑖 es una raíz.

r1 = 2 + i
r2 = 2 − i

(x − 2 − i)(x − 2 + i)
[(𝑥 − 2) − 𝑖][(𝑥 − 2) + 𝑖]
𝑥 2 − 4𝑥 + 4 − 𝑖 2
𝑥 2 − 4𝑥 + 5

139
X 6  2 X 5  4 X 4  14 X 3  29 X 2  20 X  50  0
usando el teorema de las raices complejas tendremos :
X  2  i X  2  i    X  2  i  X  2  i   x 2  4 x  5
efectuando la división normal entre este polinomio tendremos
X 6  2 X 5  4 X 4  14 X 3  29 X 2  20 X  50
 X 4  2 X 3  7 X 2  4 X  10
X 2  4X  5
como se muestra a continuación
X 4  2 X 3  7 X 2  4 X  10
X 2  4X  5 
X  2 X  4 X 4  14 X 3  29 X 2  20 X  50  0
6 5

 X 6  4X 5  5X 4

2 X 5  X 4  14 X 3
 2 X 5  8 X 4  10 X 3
7 X 4  24 X 3  29 X 2
 7 X 4  28 X 3  35 X 2

4 X 3  6 X 2  20 X
 4 X 3  16 X 2  20 X
10 X 2  40 X  50
 10 X 2  40 X  50
0

Aplicando el método de Ferrari para el cociente de la división anterior.

x 4 + 2x 3 + 7x 2 + 4x + 10 = 0
x 4 + 2px 3 + qx 2 + 2rx + s = 0
2p = 2 𝑝=1
q=7
2𝑟 = 4 𝑟=2
𝑠 = 10

Sustituyendo el valor de p, q, r, s.
2k 3 − qk 2 + 2(pr − s)k + s(q − p2 ) − r 2 = 0
2k 3 − (7)k 2 + 2[(1▫2) − 10]k + 10[7 − (1)2 ] − (2)2 = 0
2k 3 − 7k 2 − 16k + 60 − 4 = 0
2k 3 − 7k 2 − 16k + 56 = 0

140
Para hallar el valor de k
Buscamos los factores:
𝐹(2) ± 1, ±2
𝐹(56) = ±1, ±2, ±4, ±7, ±8, ±14, ±28, ±56
56 1 7
𝐹 ( ) = ±1, ± , ±2, ±4, ±7, ± , ±8, ±14, ±28, ±56
2 2 2

2k 3 − 7k 2 − 16k + 56 = 0

2 7  16 56
7
7 0  56
2
2 0  16 0

7
k=
2
𝑎2 = 𝑝2 + 2𝑘 − 𝑞
7
a2 = 1 + 2 ( ) − 7
2
𝑎2 = 1 + 7 − 7
𝑎 = ±√1 𝑎 = ±1

b2 = k 2 − s
7
𝑏 2 = ( )2 − 10
2
2
49
b = − 10
4
9
𝑏 = ±√
4
3
𝑏=±
2

x 2 + (p − a)x + (k − b) = 0
3 7
Para a = 1, b = − , p = 1, k=
2 2

7 3
x 2 + (1 − 1)x + [ − (− )] = 0
2 2
x2 + 5 = 0

141
3
Para a = −1 , b =
2
2
7 3
x + (1 + 1)x + [ − ( )] = 0
2 2
𝑥 2 + 2𝑥 + 2 = 0

3
Para a = −1, b = −
2
7 3
x 2 + [(1 − (−1)]x + [ − (− )] = 0
2 2
x 2 + 2x + 5 = 0

3
Para a = 1, b =
2
2
7 3
x + (1 − 1)x + [ − ( )] = 0
2 2
x2 + 2 = 0

(x 2 + 2x + 5)(x 2 + 2) 𝑥 4 + 2𝑥 3 + 7𝑥 2 + 4𝑥 + 10 = 0
x 4 + 2x 2 + 2x 3 + 4x + 5x 2 + 10 → x 4 + 2x 3 + 7x 2 + 4x + 10

Raíces:

x 2 + 2x + 5 = 0

−2 ± √(2)2 − 4(1)(5)
𝑥=
2

−2 ± √4 − 20
𝑥=
2

−2 ± √−16
𝑥=
2

−2 ± 4i
𝑥=
2

x3 = −1 + 2i
x4 = −1 − 2i

x2 + 2 = 0
𝑥 2 = −2
𝑥 = ±√−2
𝑥 = ±√2 𝑖

142
𝑥5 = √2 𝑖
𝑥6 = −√2 𝑖
Por lo tanto las raíces buscadas son:

r1 = 2 + i
𝑟2 = 2 − 𝑖
𝑟3 = −1 + 2𝑖
𝑟4 = −1 − 2𝑖
r5 = √2i
r6 = −√2i

Comprobación de n- factores:

𝑃(𝑥) = 𝑥 6 − 2𝑥 5 + 4𝑥 4 − 14𝑥 3 + 29𝑥 2 − 20𝑥 + 50 = 0


𝑃(𝑥) = (𝑥 − 𝑟1 )(𝑥 − 𝑟2 )(𝑥 − 𝑟3 )(𝑥 − 𝑟4 )(𝑥 − 𝑟5 )(𝑥 − 𝑟6 )

𝑃(𝑥) = (𝑥 − 2 − 𝑖)(𝑥 − 2 + 𝑖)(𝑥 + 1 − 2𝑖)(𝑥 + 1 + 2𝑖)(𝑥 − √2𝑖)(𝑥 + √2 𝑖)


𝑃(𝑥) = [(𝑥 − 2) − 𝑖][(𝑥 − 2) + 𝑖][(𝑥 + 1) − 2𝑖][(𝑥 + 1) + 2𝑖](𝑥 2 + 2)
𝑃(𝑥) = (𝑥 2 − 4𝑥 + 4 − 𝑖 2 )(𝑥 2 + 2𝑥 + 1 − 4𝑖 2 )(𝑥 2 + 2)
𝑃(𝑥) = (𝑥 2 − 4𝑥 + 5)(𝑥 2 + 2𝑥 + 5)(𝑥 2 + 2)
𝑃(𝑥) = (𝑥 4 + 2𝑥 3 + 5𝑥 2 − 4𝑥 3 − 8𝑥 2 − 20𝑥 + 5𝑥 2 + 10𝑥 + 25)(𝑥 2 + 2)
𝑃(𝑥) = (𝑥 4 − 2𝑥 3 + 2𝑥 2 − 10𝑥 + 25)(𝑥 2 + 2)
𝑃(𝑥) = 𝑥 6 + 2𝑥 4 − 2𝑥 5 − 4𝑥 3 + 2𝑥 4 + 4𝑥 2 − 10𝑥 3 − 20𝑥 + 25𝑥 2 + 50
𝑃(𝑥) = 𝑥 6 − 2𝑥 5 + 4𝑥 4 − 14𝑥 3 + 29𝑥 2 − 20𝑥 + 50

20.- Dado el siguiente polinomio.


𝒙𝟔 − 𝟐𝒙𝟓 − 𝟒𝒙𝟒 + 𝟏𝟐𝒙𝟑 − 𝟑𝒙𝟐 − 𝟏𝟖𝒙 + 𝟏𝟖 = 𝟎

Hallar:
a) Tabla de las posibles raíces, usando el teorema de los signos de Descartes.
b) Todas las raíces, si 1 + 𝑖 es una de las raíces.
c) Escribirlo como un producto de n-factor lineal y compruébelo.

Se resuelve el polinomio:

𝒙𝟔 − 𝟐𝒙𝟓 − 𝟒𝒙𝟒 + 𝟏𝟐𝒙𝟑 − 𝟑𝒙𝟐 − 𝟏𝟖𝒙 + 𝟏𝟖 = 𝟎

143
Hallar las posibles raíces usando el teorema de los signos de descartes

P(−x) = (−x)6 − 2(−x)5 − 4(−x)4 + 12(−x)3 − 3(−x)2 − 18(−x) + 18


P(−x) = x 6 + 2x 5 − 4x 4 − 12x 3 − 3x 2 + 18x + 18

RR (+) 4 4-2=2 4-4=0 4 4-2=2


RR (-) 2 2 2 2-2=0 2-2=0
RC 0 2 4 2 4
RT 6 6 6 6 6

Sabiendo que 1+i es una raíz

b) 1+i

x 6 − 2x 5 − 4x 4 + 12x 3 − 3x 2 − 18x + 18 = 0

1 -2 -4 12 -3 -18 18

1+i 1+i -2 -6-6i 12 9+9i -18

1 -1+i -6 6-6i 9 -9+9i 0

1 -1+i -6 6-6i 9 -9+9i 0

1-i 1-i 0 -6+6i 0 9-9i

1 0 -6 0 9 0

El polinomio queda de la siguiente forma utilizando la ecuación degradada

x 4 − 6x 2 + 9

Se factoriza dicho polinomio que dando así

(x 2 − 3)(x 2 − 3)

x2 = 3 𝑥2 = 3

𝑥3,4=±√3 𝑥5,6 = ±√3

raices encontradas r1 = 1 + i, r2 = 1 − i, r3 = √3, r4 = −√3, r5 = √3, r6 = −√3

144
c)

Comprobación por n − factores (x − r1 )(x − r2 )(x − r3 )(x − r4 )(x − r5 )(x − r6 )

(x − 1 − i)(x − 1 + i)(x − √3)(x + √3)(x − √3)(x + √3) = P(x)


(x 2 − x + xi − x + 1 − i − xi + i − i2 )(x 2 + √3x − √3x − 3)(x 2 + √3x − √3x − 3) = P(x)
(x 2 − 2x + 2)(x 2 − 3)(x 2 − 3) = P(x)
(𝑥 2 − 2𝑥 + 2)(𝑥 4 − 6𝑥 2 + 9) = 𝑃(𝑥)
x 6 − 6x 4 + 9x 2 − 2x 5 + 12x 3 − 18x + 2x 4 − 12x 2 + 18 = P(x)
x 6 − 2x 5 − 4x 4 + 12x 3 − 3x 2 − 18x + 18 = P(x)

21.-Dado el siguiente polinomio.


𝒙𝟔 − 𝟐𝒙𝟓 − 𝟒𝒙𝟒 + 𝟏𝟐𝒙𝟑 − 𝟑𝒙𝟐 − 𝟏𝟖𝒙 + 𝟏𝟖 = 𝟎
Hallar:
a) Tabla de las posibles raíces, usando el teorema de los signos de Descartes.
b) Todas las raíces, si √3 es una de las raíces.
c) Escribirlo como un producto de n-factor lineal y compruébelo.

𝒙𝟔 − 𝟐𝒙𝟓 − 𝟒𝒙𝟒 + 𝟏𝟐𝒙𝟑 − 𝟑𝒙𝟐 − 𝟏𝟖𝒙 + 𝟏𝟖 = 𝟎


Usando el teorema de los signos de descartes:
P(−x) = (−x)6 − 2(−x)5 − 4(−x)4 + 12(−x)3 − 3(−x)2 − 18(−x) + 18
P(−x) = x 6 + 2x 5 − 4x 4 − 12x 3 − 3x 2 + 18x + 18 r1 = √3

RR (+) 4 4-2=2 4-4=0 4 4-2=2


RR (-) 2 2 2 2-2=0 2-2=0
RC 0 2 4 2 4
RT 6 6 6 6 6

Hallar todas las raíces sabiendo que √3 es una raíz.

r1 = √3
r2 = −√3
[x − (√3)][x − (−√3)]
(x − √3)(x + √3) = x 2 + √3x − √3x − 3
x2 − 3

145
X4-2X3-X2+6X-6

X2-3 X6-2X5-4X4+12X3-3X2-18X+18
-X6 +3X4

-2X5-X4
2X5 -6X3

–X4 +6X3
X4 -3X2

6X3-6X2 - 18x
-6X3 +18X
-6X2 +18
6X2 -18
0 X4-2X3-X2+6X-6=0
Aplicando el método de Ferrari para el cociente de la división anterior.

x 4 − 2x 3 − x 2 + 6x − 6 = 0

2p = −2 𝑝 = −1
𝑞 = −1
2r = 6 𝑟=3
𝑠 = −6

2𝑘 3 + 𝑘 2 + 2[−3 + 6]𝑘 + [(−1 − 1)(−6) − 9] = 0


2k 3 + k 2 + 6k + 12 − 9 = 0
2𝑘 3 + 𝑘 2 + 6𝑘 + 3 = 0

1 1
factores: ± , ± , ±1, ±2, ±3
2 3

2 1 6 3
1
 1 0 3
2
2 0 6 0

146
El valor de k es:
1
k=−
2

a2 = 2k + p2 − q
1
a2 = 2 (− ) + (−1)2 − (−1)
2
𝑎2 = −1 + 1 + 1
a = ±√1
a = ±1

b2 = k 2 − s
1
b2 = ( )2 − (−6)
2
2
1
b = +6
4
25
b = ±√
4

5
𝑏=±
2

a=1
5
b=−
2

x 2 + (p − a)x + (k − b) = 0
x 2 + (p − a)x + (k + b) = 0

1 5
1. − x 2 + (−1 − 1)x + [(− ) + ] = 0
2 2
2
1 5
2. − x + (−1 − 1)x + [(− ) − ] = 0
2 2

1. −𝑥 2 − 2𝑥 + 2 = 0 → 𝑒𝑐. 1
2. −𝑥 2 − 2𝑥 − 3 = 0 → 𝑒𝑐. 2

147
Ecuación 1:

Formula general:

−b ± √b 2 − 4ac
x=
2a

2 ± √4 − 4(1)(2)
x=
2
+
2 ± √−4 2 2i +
x= = − =1 i
2 2 −

r3 = 1 + i
r4 = 1 − i

Ecuación 2:

𝑥 2 − 2𝑥 − 3 = 0

2 ± √4 − 4(1)(−3)
𝑥=
2
+
2 ± √4 + 12 2 √16
𝑥= = −
2 2

2+4 6
X1 = = =3
2 2

2 − 4 −2
X2 = = = −1
2 2

Estas raíces no son las correctas. Por lo que se deberán utilizar los valores de “a” y
“b” con signo contrario al utilizado en la ecuación anterior, es decir:

5
a = −1 y b=+
2

148
Por lo que la ecuación 2 queda
1 5
x 2 + [−1 − (−1)]x + + = 0
2 2
x2 + 3 = 0
𝑥 2 = −3
𝑥 = ±√−3
𝑥 = ±√3 𝑖

𝑟5 = √3 𝑖
𝑟6 = −√3 𝑖

Gráfica del polinomio;

𝑦 = 𝑥 6 − 2𝑥 5 − 4𝑥 4 + 12𝑥 3 − 3𝑥 2 − 18𝑥 + 18 = 0

149
22.- Resolver el siguiente polinomio:

Obtener las raíces del polinomio.

+ + + +
𝑥 5 − 5𝑥 4 + 9𝑥 3 − 6𝑥 2 Factores de 6: 1, 2, 3, 6
− − − −

𝑥 2 (𝑥 3 − 5𝑥 2 + 9𝑥 − 6) = 0

RR(+) 3 1
RR(-) 0 0
RC 0 2
RT 3 3

1 5 9 6
2 2 6 6
1 3 3 0

(x 2 − 3x + 3)(x − 2)

−(−3) ± √(−3)2 − 4(1)(3) 3 ± √9 − 12


= →=
2(1) 2

3 ± √−3 3 ± 3i

2 2

3 + √3𝑖
𝑟1 =
2
3 − √3𝑖
𝑟2 =
2
𝑟3 = 2

150
23.- Obtener las raíces de un polinomio.

P(x) = x 5 − 6x 3 + 6x 2 − 7x + 6

Si una de sus raíces es r1=i

RR(+) 4 2 0
RR(-) 1 1 1
RC 0 2 4
RT 5 5 5

(x − r)(x − r)
(x − i)(x + i)
(x 2 + ix − ix − i2 )
x2 + 1
x 3 7 x  6
x  1 x  6x  6x 2  7x  6
2 5 3

 x5  x3
 7x3  6x 2  7x  6
7x3  7x
6x  6
2

6x 2  6
0

1 0 7 6
1 1 1 6
1 1 6 0

𝑥 3 − 7𝑥 + 6
+ + + +
Factores de 6: 1, 2, 3, 6
− − − −

(x − 1)(x 2 + x − 6)

(x − 1)(x − 2)(x + 3)

𝐫𝟏 = 𝐢, 𝐫𝟐 = −𝐢, 𝐫𝟑 = 𝟏, 𝐫𝟒 = 𝟐, 𝐫𝟓 = −𝟑

151
24.- Obtener las raíces del polinomio P(x) = 2x 4 − x 3 + x 2 − x − 1 = 0

RR(+) 3 1
RR(-) 1 1
RC 0 2
RT 4 4

2 1 1 1 1
1 2 1 2 1
2 1 2 1 0

(x − 1)(2𝑥 3 + 𝑥 2 + 2𝑥 + 1)

2𝑥 3 + 𝑥 2 + 2𝑥 + 1 = 0

Factores de 1: factores de 2:

±1 ±1, ±2

2 1 2 1
1
 1 0 1
2
2 0 2 0

2x 2 + 2 = 0
1
(x − 1) (x + ) (2x 2 + 2)
2

−0 ± √(0)2 − 4(2)(2) ±√−16 ±4𝑖


→ →
2(2) 4 4
1
±i → (x − 1) (x + ) (x − i)(x + i)
2

𝟏
𝐫𝟏 = 𝟏; 𝐫𝟐 = − ; 𝐫𝟑 = 𝐢, 𝐫𝟒 = −𝐢
𝟐

152
25.- Obtener las raíces del siguiente polinomio:

x 3 − 6x 2 + 16x − 16 = 0

factores de 16: ± 1, ±2, ±4, ±8, ±16

RR(+) 3 1
RR(-) 0 0
RC 0 2
RT 3 3

1 6 16  16
2 2 8 16
1 4 8 0

x 2 − 4x + 8 = 0

4 ± √(−4)2 − 4(1)(8) 4 ± √16 − 32 4 ± √−16 4 ± 4𝑖


= →= →= →=
2(1) 2 2 2
= 2 ± 2𝑖

𝐫𝟏 = 𝟐; 𝐫𝟐 = 𝟐 + 𝟐𝐢; 𝐫𝟑 = 𝟐 − 𝟐𝐢

26.-Encontrar las raíces del siguiente polinomio

P(x) = 2t 5 − 3t 4 + t 2

t 2 (2t 3 − 3t 2 + 1)

Factores de 1=±1

RR(+) 2 0
RR(-) 1 1
RC 0 2
RT 3 3

153
2 3 0 1
1 2 1 1
2 1 1 0

2 1 1
1
 1 1
2
2 2 0

2
2x − 2 = 0 → x = →x=1
2
𝟏
𝐫𝟏 = 𝟏; 𝐫𝟐 = − ; 𝐫𝟑 = 𝟏
𝟐

27.- Encontrar las raíces del siguiente polinomio

f(x) = 2x 3 − x 2 − 6x + 3 = 0

RR(+) 2 0
RR(-) 1 1
RC 0 2
RT 3 3

+ +
Factores de 3= 1; 3
− −

3 +1 + +3 + +
Factores de = ; 1; ; 2; 3
2 −2 − −2 − −

154
2 1 6 3
1
1 0 3
2
2 0 6 0

2x 2 − 6 = 0

0 ± √(0)2 − 4(2)(−6) ±√48 ±4√3


= = = = ±√3
2(2) 4 4

𝟏
𝐫𝟏 = ; 𝐫𝟐 = √𝟑; 𝐫𝟑 = −√𝟑
𝟐

28.- Encontrar las raíces del siguiente polinomio

f(x) = x 3 − 4x 2 − x + 4

factores de 4 = ±1, ±2, ±4

RR(+) 2 0
RR(-) 1 1
RC 0 2
RT 3 3

1 4 1 4
1 1 3 4
1 3 4 0

x 2 − 3x − 4 = 0

(x − 1)(x − 4)(x + 1)
𝐫𝟏 = 𝟏; 𝐫𝟐 = 𝟒; 𝐫𝟑 = −𝟏

155
29.- Encontrar las raíces del siguiente polinomio
f(x) = x 3 − 3x 2 − 4x + 12

RR(+) 2 0
RR(-) 1 1
RC 0 2
RT 3 3

factores de 12 = ±1, ±2, ±3, ±4

1 3 4 12
2 2 2  12
1 1 6 0

x2 − x − 6 = 0
(x − 2)(x 2 − x − 6)
(x − 2)(x − 3)(x + 2)

𝐫𝟏 = 𝟐; 𝐫𝟐 = 𝟑; 𝐫𝟑 = −𝟐

30.- Encontrar las raíces del siguiente polinomio.

f(x) = x 4 + 2x 3 − 3x 2 − 4x + 4

factores de 4: ± 1, ±2, ±4

RR(+) 2 0 0
RR(-) 2 2 0
RC 0 2 4
RT 4 4 4

156
1 2 3 4 4
1 1 3 0 4
1 3 0 4 0
2 2 2 4
1 1 2 0

(x 2 + x − 2) = 0
(x − 1)(x + 2)(x 2 + x − 2)
(x − 1)(x + 2)(x − 1)(x + 2)

𝐫𝟏 = 𝟏; 𝐫𝟐 = −𝟐; 𝐫𝟑 = 𝟏; 𝐫𝟒 = −𝟐

31.-Resolver el siguiente polinomio por el método de Ferrari

𝑥 4 + 2𝑥 3 − 3𝑥 2 − 4𝑥 + 4 = 0
𝑥 4 + 2𝑝𝑥 3 + 𝑞𝑥 2 + 2𝑟𝑥 + 𝑠 = 0
2𝑝 = 2 → 𝑝 = 1
𝑞 = −3
2𝑟 = −4 → 𝑟 = −2
𝑠=4

Sustituyendo en la formula de Ferrari:

2k 3 − qk 2 + 2(pr − s)k − (p2 − q)s − r 2


2𝑘 3 − (−3𝑘 2 ) + 2[(1)(−2) − 4]𝑘 − [(12 ) − (−3)](4) − (−22 )
2𝑘 3 + 3𝑘 2 + 2(−2 − 4)𝑘 − [(1 + 3)4] − 4
2k 3 + 3k 2 − 12k − 20 = 0

factores: ±1, ±2, ±4, ±5, ±10, ±20

2 3  12  20
2 4 2 20
2 1  10 0

k = −2

157
Buscamos las constantes “ a” y “b” a través de las formulas

𝑎2 = 2𝑘 + 𝑝2 − 𝑞
𝑎2 = 2(−2) + (1)2 − (−3)
𝑎2 = −4 + 1 + 3
𝑎 = ±√0
𝑎=0

𝑏2 = 𝑘 2 − 𝑠
𝑏 2 = (−2)2 − 4
𝑏2 = 4 − 4
𝑏2 = 0
𝑏 = ±√0
𝑏=0

Sustituimos en las siguientes formulas para hallar sus raíces

Tomamos los valores positivos de “a” y “b”

x 2 + (p − a)x + (k − b) = 0
𝑥 2 + (1 − 0)𝑥 + (−2 − 0) = 0
𝑥2 + 𝑥 − 2 = 0

x 2 + (p + a)x + (k + b) = 0
x 2 + (1 + 0)x + (−2 + 0) = 0
𝑥2 + 𝑥 − 2 = 0

(𝑥 + 2)(𝑥 − 1) = 0 (𝑥 + 2)(𝑥 − 1) = 0
𝑥 = −2 𝑥 = 1 𝑥 = −2 𝑥 = 1

𝑟1 = −2
𝑟2 = 1
𝑟3 = −2
𝑟4 = 1

158
CAPITULO III

MATRICES Y DETERMINANTES

INTRODUCCION

Las matrices aparecen por primera vez hacia el año 1850, introducidas por J.J.
Sylvester. El desarrollo inicial de la teoría se debe al matemático W.R. Hamilton en
1853. En 1858, A. Cayley introduce la notación matricial como una forma
abreviada de escribir un sistema de m ecuaciones lineales con n incógnitas.

Las matrices se utilizan en el cálculo numérico, en la resolución de sistemas de


ecuaciones lineales, de las ecuaciones diferenciales y de las derivadas parciales.
Además de su utilidad para el estudio de sistemas de ecuaciones lineales, las
matrices aparecen de forma natural en geometría, estadística, economía,
informática, física, etc...
Los modelos matemáticos han jugado un papel un papel muy importante en el
desarrollo de las ciencias, tanto exactas como sociales, ya que permiten describir
de una manera situaciones reales.
Ningún modelo es una descripción exacta de la realidad, sin embargo, los modelos
son útiles para hacer predicciones acerca de ella. En particular, los modelos
lineales proporcionan una buena aproximación de la realidad y tienen la ventaja de
ser fácilmente manejable y de que muy frecuentemente se puedan encontrar las
soluciones analíticas o numéricas de las cantidades de interés.
Las técnicas desarrolladas en el álgebra lineal se pueden usar en todos los
modelos lineales, y, en ocasiones, se pueden usar para amplificar las operaciones
en modelos no lineales.
El propósito de este capítulo es proporcionar al estudiante conceptos y problemas
que le ayuden en el manejo del álgebra haciendo énfasis en la comprensión de
los conceptos más relevantes y en su aplicación a problemas tomados de diversas
disciplinas.

159
El capitulo está diseñado como un complemento para un curso convencional de
esta materia. Se incluyen los elementos más importantes de la teoría para que
pueda usarse independientemente del curso, pero se hace mayor énfasis en los
problemas. Es a través de la solución de éstos que se adquiere un conocimiento
más sólido.
3.1. Definición de matriz.

Esta se define como: un arreglo cuadrado o no cuadrado de filas (m), columnas(n)


(es decir 𝑚 = 𝑛, 𝑚 > 𝑛, 𝑚 < 𝑛), las cuales pueden estar formadas de Números
reales, complejos o de ambos, así como, literales, expresiones Algebraica o una
combinación de estas, las cuales están cerradas en Paréntesis circulares (), o
corchetes [].

Por lo general las matrices se representan con letras mayúsculas (A, B, C,……) Y
los elementos de estas con letras minúsculas (a, b, c,…….)

Una matriz se puede escribir en general de la siguiente forma:


Se llama matriz de orden m × n a todo conjunto rectangular de elementos aij
dispuestos en m líneas horizontales (filas) y n verticales (columnas) de la forma:
 a11 a12 a13 ..... a1n 
 
 a 21 a 22 a 23 ..... a 2 n 
a a32 a33 ..... a3n 
Sea A  aij   31 
 .................................... 
 
 .................................... 
a 
 m1 a m 2 a m3 ..... a mn 

Como se podrá comprobar más adelante, esta forma tabular de escribirlo, nos
servirá para tratar a los sistemas de ecuaciones lineales; y facilitar con ello el
empleo de transformaciones elementales.
En esta unidad trataremos a estos, como entes matemáticos, con existencia
propia, aunque posteriormente lo asociaremos con los sistemas de ecuaciones
lineales.

160
i =1, 2, 3,4,..., m (Número de filas o renglones)

j=1, 2, 3,4,..., n (Números de columnas)

𝐴 = 𝑚 ∗ 𝑛 (𝑂𝑟𝑑𝑒𝑛 𝑑𝑒 𝑙𝑎 𝑚𝑎𝑡𝑟𝑖𝑧)

Lo anterior se debe de interpretar de la siguiente manera: primer subíndice del


orden de una matriz nos indica el número de filas que esta contiene, y el segundo
subíndice el numero de columnas correspondiente.

3.1.1. Tipos de Matrices. Atendiendo a la forma:

Matriz fila: Es una matriz que solo tiene una fila, es decir m =1 y por tanto es de
orden 1 x n.

A 1x3 = (7 2 −5 )

Matriz columna: Es una matriz que solo tiene una columna, es decir, n =1 y por
tanto es de orden m x 1.

−7
𝐴3𝑥1 = 1
6

Matriz cuadrada: Es aquella que tiene el mismo número de filas que de


columnas, es decir m = n. En estos casos se dice que la matriz cuadrada es de
orden n.

Diagonal principal: son los elementos de a11 , a22,..., ann ,

161
Matriz transpuesta (AT): Dada una matriz A, se llama transpuesta de A, y se
representa por AT, a la matriz que se obtiene cambiando las filas por columnas. La
primera fila de A es la primera columna de AT, la segunda fila de A es la segunda
columna de AT, etc. De la definición se deduce que, si A es de orden m x n,
entonces AT es de orden m x n.

𝑆𝑖 𝑒𝑠 𝐴 = (𝑎𝑖𝑗 )𝑚 𝑥 𝑛

su transpuesta es AT = (aji )n x m

1 3
1 2 5
A= ; 𝐴𝑇 = 2 −4
3 −4 7 5 7

Ejemplo:

1.- Indique el orden de las matrices que se muestran a continuación:

2 −𝑖 4
𝐴= Orden 2 x 3
5 2 1

𝐵 = (1 − 𝑖 3 4𝑎 − 2𝑏) Orden 1 x 3

4 −2
𝐶= 1+𝑖 3 Orden 3 x 2
4𝑎 + 𝑏 5

3
 
  1
D  orden 4 x1
4
 
2
 

162
3.1.2. Operaciones y propiedades

3.1.2.1.-La Igualdad de Matrices


Se dice que dos matrices son iguales, cuando tienen los mismos elementos y éstos
se encuentran dispuestos de la misma manera en ambos arreglos.

Esta idea se pude expresar en términos más precisos, con ayuda del símbolo a i j ,
que representa el término que se encuentra en la posición correspondiente al
renglón “i” y a la columna “j” de la matriz.

Definición:

Sean A = (aij ) y B = (bij ) dos matrices de mxn; se dice que A y B son iguales si

𝑎𝑖𝑗 = 𝑏𝑖𝑗 Para i=1,2,… m y j=1,2,…n

Ejemplo:
 1 2  1  2
   
 3  i 3 i 
Sean las matrices A   y B A es diferente de B, ya que
4 2  2 1
   
 i  1  i  1 
  

𝑎12 ≠ 𝑏12 𝑎31 ≠ 𝑏31 𝑦 𝑎32 ≠ 𝑏32 ; aun cuando todos los demás elementos
son iguales.

163
3.1.2.2. Suma, diferencia y propiedades

Para poder sumar o restar matrices, éstas deben tener el mismo número de
filas y de columnas. Es decir, si una matriz es de orden 3x2 y otra de 3x3, no
se pueden sumar ni restar, tanto para la suma como para la resta, se suman o
se restan los términos que ocupan el mismo lugar en las matrices.

Para sumar o restar más de dos matrices, se procede de la misma manera. No


necesariamente para poder sumar o restar matrices, éstas tienen que ser
cuadradas.
Dos matrices se pueden sumar y restar, si y solo si, son del mismo orden, y la
operación se hace termino a termino de los elementos que tengan la misma
posición en ambas matrices.
En general se define de la siguiente manera:

Si A = (aij ) y B = (bij )

Dos matrices de mxn

→ A ± B = (aij ) ± (bij )

Para i = 1, 2, … m y j = 1, 2, … n

si m = 3 y n=2

i = 1,2,3 j = 1,2

𝑎11 𝑎12 𝑏11 𝑏12


𝑎
𝑎𝑖𝑗 = 21 𝑎22 ; 𝑏𝑖𝑗 = 𝑏21 𝑏22
𝑎31 𝑎32 𝑏31 𝑏32

𝑎11 + 𝑏11 𝑎12 + 𝑏12


𝐴 + 𝐵 = 𝑎𝑖𝑗 = 𝑎21 + 𝑏21 𝑎22 + 𝑏22
𝑎31 + 𝑏31 𝑎32 + 𝑏32

164
PROPIEDADES
1) A + (B + C) = (A + B) + C

2) 𝐴 (𝐵 + 𝐶) = 𝐴𝐵 + 𝐴𝐶 Commented [G1]: Ya no lleva ese signo..

3) ∃ − A| A + (−A) = 0

4) ∃ 0| 𝐴 + 0 = 𝐴

5) A + B = B + A

Donde la matriz cero es aquella cuyos elementos son todos ceros.

Ejemplo:
Realizar las siguientes operaciones
3 1 2 −1 2 4
1.- Sean las matrices A = 0 5 −3 y B= 2 5 8 .
7 0 4 0 1 −2
Entonces:

3 1 2 −1 2 4 2 3 6
A+B= 0 5 −3 + 2 5 8 = 2 10 5
7 0 4 0 1 −2 7 1 2

3 1 2 −1 2 4 4 −1 −2
A−B= 0 5 −3 − 2 5 8 = −2 0 −11
7 0 4 0 1 −2 7 −1 6

−1 2 4 3 2 0 5 −1 3
2.- Sean A = , B= y C=
2 7 6 0 −3 −1 1 1 2

−1 2 4 3 2 0 5 −1 3 7 3 7
A+B+C= + + =
2 7 6 0 −3 −1 1 1 2 3 5 7

−1 2 4 3 2 0 5 −1 3 1 −1 7
A−B+C= − + =
2 7 6 0 −3 −1 1 1 2 3 11 9

165
3.1.2.3. Producto Matricial y sus propiedades.
Se puede realizar un producto entre dos matrices A de orden (𝑚𝑥𝑛) y una matriz B
de (𝑟𝑥𝑝), si y solamente si, cumplen con el criterio de conformabilidad, esto quiere
decir que dos matrices son conformables, si el número de columnas (n) de la
primera matriz (la que premultiplica), tiene el mismo número de filas (r) de la
segunda matriz (la que posmultiplica). Y el orden resultante C (𝑚𝑥𝑝), de la matriz,
es el número de filas (m) de la primera matriz (la que premultiplica) y el número
de columnas (p) de la segunda matriz (la que posmultiplica).

Otra manera de plantear este producto es la siguiente:

Sean A, B, y C matricesde orden que se indican

A  B = C

m*n r* p m*p

Iguales

Orden resultante

En general el producto matricial se puede definir:

Sean 𝐴 = (𝑎𝑖𝑗 ) 𝑦 𝐵 = (𝑏𝑖𝑗 ) dos matrices con elementos en C de mxn y nxq


respectivamente. El producto AB es una matriz 𝑃 = (𝑃𝑖𝑗 ) de nxq definida por

𝑃𝑖𝑗 = ∑ 𝑎𝑖𝑘 𝑏𝑘𝑗 ; 𝑝𝑎𝑟𝑎 𝑖 = 1 … 𝑛 𝑦 𝑗 = 1 … . 𝑞


𝑘=1

Ejemplo:

1.- Realizar el producto matricial indicado.

3 2 −1 2 5
𝐴 = (4 5 2 ) ° 𝐵 = (−1 2) = 3 ∗ 2 (𝑂𝑟𝑑𝑒𝑛 𝑑𝑒 𝑙𝑎 𝑚𝑎𝑡𝑟𝑖𝑧 𝑟𝑒𝑠𝑢𝑙𝑡𝑎𝑛𝑡𝑒)
0 1 0 4 3

3∗3 3∗2

166
 2
 
5
  
  1   2 
 4
 
 3
  
 3 2  1 3 2  1 
 2
 
5
 

  1   2 
3 2 −1 2 5  4
 
 3
  
4 5 2 ° −1 2 =  4 5 2  4 5 2 
0 1 0 4 3  2
 
5
 

  1   2 
 4
 
 3
  
 0 1 0 0 1 0 
 
 
 

0 16
= A°B = ( 11 36)
−1 2

PROPIEDADES

1) AB ≠ BA → Conmutativa
2) A(BC) = (AB)C → Asociativa

3) A(B + C) = AB + AC → Distribucion por la derecha

4) (B + C)A = BA + CA → Distribucion por la izquierda

5) kA = Ak → Donde k es un escalar

Cuando se multiplica un escalar por una matriz, se multiplica cada uno de los
elementos de la matriz por el escalar.

De esta manera, cuando se multiplica un escalar por la matriz unitaria, los


elementos de la diagonal principal, toman los valores del escalar, generándose así
lo que se le llama matriz escalar, que más adelante se retomara, y se pudiera decir
que el escalar se convierte en una matriz.

167
Es decir:
ka11 ka12 ka13
kA = ( … … … )
kam1 kam2 kamn

Ejemplo:

1 −2 3
Sea A = ( )
4 5 −2

Entonces:

3▫1 3▫ (−2) 3 ▫3 3 −6 9
3𝐴 = ( )=( )
3▫4 3▫5 3▫(−2) 12 15 −6

Producto de una matriz por un escalar

1. Propiedad distributiva 1 k (A + B) = k A + k B

2. Propiedad distributiva 2 (k + h)A = k A + h A


3. Propiedad asociativa mixta k [h A] = (k h) A
4. Elemento unidad 1·A = A

168
Ejemplos:

1.- Sean

1 3 −1 5 2
A=[ ] y B=[ ]
−1 0 0 4 7

Calcule, de ser posible, los productos AB y BA.

Solución: Ya que la dimensión de A es de 2 x 2 y la de B es 2 x 3, el producto


AB está definido, y su dimensión es 2 x 3. Por consiguiente, podemos escribir

1 3 −1 5 2 ? ? ?
AB = [ ][ ]=[ ]
−1 0 0 4 7 ? ? ?

donde los signos de interrogación se sustituyen aplicando la regla que define los
elementos de un producto de matrices.

El elemento (1,1) será el producto interior del primer renglón de A por la primera
columna de B:

1(−1) + 3(0) = −1

Para el elemento (1,2) será el producto del primer renglón de A por la segunda
columna de B:

1(5) + 3(4) = 17

Y así se llega entonces a:

−1 17 23
𝐴𝐵 = [ ]
1 −5 −2

Sin embargo, el producto BA no está definido, porque no coinciden el número de


columnas de la matriz B con el número de renglones de A.

169
2.- Para las siguientes matrices:

2 1−i 1 2i 1 + i
1 −1 3
A = (i 0 3 );B = (1 0 ); C = ( )
1 2 −1
0 1 + i −i 0 2−i

Calcular, de ser posible AB, BA, BC, CB, ABC, CBA, AB+C, BC+A.
Solución:
AB:
2 1−i 1 2i 1 + i 4i + 1 − i 2 + 2i + 2 − i 1 + 3i 4+i
AB = ( i 0 3 )(1 0 ) = ( 2i2 i + i2 + 6 − 3i) = ( −2 5 − 2i )
0 1 + i −i 0 2−i 1+i −2i + i2 1 + i −1 − 2i

BA:
B3*2 · A3*3
 No se puede realizar

BC:

2i 1 + i
1 −1 3
BC = ( 1 0 )( ) → B3∗2 ⦁C2∗3 = D3∗3
1 2 −1
0 2−i

2𝑖 + 1 + 𝑖 −2𝑖 + 2 + 2𝑖 6𝑖 − 1 − 𝑖 1 + 3i 2 −1 + 5𝑖
=( 1 −1 3 )=( 1 −1 3 )
2−𝑖 4 − 2𝑖 −2 + 𝑖 2 − 𝑖 4 − 2𝑖 −2 + 𝑖

CB:
2𝑖 1 + 𝑖
1
−1 3
𝐶𝐵 = ( )(1 0 ) → 𝐶2∗3 ⦁𝐵3∗2 = 𝐴2∗2
12 −1
0 2−𝑖
2𝑖 − 1 1 + 𝑖 + 6 − 3𝑖 2𝑖 − 1 7 − 2𝑖
=( )=( )
2𝑖 + 2 1 + 𝑖 − 2 + 𝑖 2𝑖 + 2 −1 + 2𝑖

170
ACB:
𝐀𝐂𝐁 = 𝐀⦁𝐁⦁𝐂 = (𝐀𝟑∗𝟑 ⦁ 𝐁𝟑∗𝟐 )𝐂𝟐∗𝟑 = 𝐗 𝟑∗𝟐 ⦁𝐂𝟐∗𝟑 = 𝐘𝟑∗𝟑

3i + 1 4+i
1 −1 3
( −2 −2i + 5) ( )
1 2 −1
1+i −2i − 1

AB C
3i + 1 + 4 + i −3i − 1 + 8 + 2i 9i + 3 − 4 − i
= ( −2 − 2i + 5 2 − 4i + 10 −6 + 2i − 5 )
1 + i − 2i − 1 −1 − i − 4i − 2 3 + 3i + 2i + 1
5 + 4i 7−i −1 + 8i
= (3 − 2i 12 − 4i −11 + 2i )
−i −3 − 5i 4 + 5i

CBA:
𝐂𝐁𝐀 = 𝐂⦁𝐁⦁𝐀 = (𝐂𝟐∗𝟑 ⦁𝐁𝟑∗𝟐 )𝐀𝟑∗𝟑

 No se puede realizar, ya que el producto por “A” no es conformable

BCA:
BCA = B⦁C⦁A = (B3∗2 ⦁C2∗3 )A3∗3 = X 3∗2 • A3∗3 = Y3∗3

1 + 3i 2 −1 + 5𝑖 2 1 − i 1
( 1 −1 3 )(i 0 3)
2 − 𝑖 4 − 2𝑖 −2 + 𝑖 0 1 + i −i
2 + 6𝑖 + 2𝑖 1 − 𝑖 + 3𝑖 − 3𝑖 2 − 1 − 𝑖 + 5𝑖 + 5𝑖 2 1 + 3𝑖 + 6 + 𝑖 − 5𝑖 2
=( 2−𝑖 1 − 𝑖 + 3 + 3𝑖 1 − 3 − 3𝑖 )
4 − 2𝑖 + 4𝑖 − 2𝑖 2 2 − 2𝑖 − 𝑖 + 𝑖 2 − 2 − 2𝑖 + 𝑖 + 𝑖 2 2 − 𝑖 + 12 − 6𝑖 + 2𝑖 − 𝑖 2

2 + 8𝑖 −2 + 6𝑖 12 + 4𝑖
( 2−𝑖 4 + 2𝑖 −2 − 3𝑖 )
6 + 2𝑖 −2 − 4𝑖 15 − 5𝑖

171
3.1.2.4. Ejercicios Propuestos

1-12. Las matrices A, B, C, D, E, F Y G se definen como siguen:

5 1 0 0 1
2 −5
A = [2 −
2
0] B = [0 1 0 ] C=[ ] D = 7 3] E = [2 ]
0 7
0 2 −3 0 0 1 0

1 5 −3 10
F = [3 2 5 ] G=[ 6 1 0]
1 −1 3 −5 2 2

Efectúe la operación algebraica indicada, o explique por qué no se puede


hacer.

1. - B+C 2.- B+F 3.- C-B 4.- 5A

5.- 3B+2C 6.- C-5A 7.- 2C-6B 8.- DA

9.- AD 10.- BC 11.- BF 12.- GF

13 – 18. Sean

 1
0 6 1  0
1
A B  1 7 9 2 C 
 2 1/ 2 4 0   1
 
 2 

Indique cuáles de los siguientes productos están definidos, y calcúlelos.

13.- ABC 14.- ACB 15.- BAC 16.- BCA

17.- CAB 18.- CBA

172
3.2. Transformaciones elementales
Una propiedad importante que se usa a menudo en los sistemas matriciales es la
llamada “TRANSFORMACIONES ELEMENTALES”, las cuales no alteran la solución
de los sistemas, es decir transformaciones que al aplicarse a un sistema dan como
resultado un sistema equivalente. Estas transformaciones pueden ser de tres tipos.

1. Intercambio de filas.

2. Multiplicación de una fila por cualquier escalar.

3. Multiplicación de una fila por un escalar, y el resultado agregarlo a otra fila,


remplazando esta ultima por el resultado obtenido.

Nota: Lo anterior, se aplicará a algunos problemas más adelante.

3.3. Matriz identidad o unitaria (I) y propiedades


La matriz unitaria debe ser una matriz cuadrada (m=n) que debe de cumplir las
siguientes condiciones

s i j  1
 si i j
I  ( si j )  
s i j  0
 si i j

Es decir si

 s11 s12 s13 ........ s1n 


 
 s 21 s 22 s 23 ....... s 2 n 
s s33 ...... s3n 
I  S ij    31 32
s

 ................................... 
 
 ................................... 
s 
 m1 s m 2 s m 3 .... s mn 
Para una matriz de orden 2x2 y de 3x3 tendremos:

 s11 s12 s13   1 0 0 


s s12   1 0     
I   11   I   s 21 s 22 s 23    0 1 0 
 s 21 s 22   0 1  s
 31 s32 s33   0 0 1 

173
En general tendríamos.

 s11 s12 s13 ........ s1n   1 0 0 0


0 0
   
 s 21 s 22 s 23 ....... s 2 n   0 1 0 0
0 0
s s33 ...... s3n   0 0
I  S ij    31 32
s 0 1 0 0
 
 ...................................   0 0 0 0
1 0
   
 ...................................   
s   0 0 0 0 1 
 m1 s m 2 s m3 .... s mn   0

Propiedades:

Si se multiplica la matriz (A) por la matriz unitaria (I), tanto por la derecha o por la
izquierda, da por resultado la misma matriz (A), siempre y cuando el producto se
pueda efectuar.

Lo anterior se puede plantear de la siguiente manera:

IA=AI=A (Siempre que el producto se pueda efectuar).

Ejemplo: Comprobar la propiedad conmutativa de la matriz un unitaria


Sea:

2 1 3
A = (3 0 4 )
1 −2 2
2 1 3 1 0 0 2 1 3
AI = (3 0 4) (0 1 0) = (3 0 4)
1 −2 2 0 0 1 1 −2 2
1 0 0 2 1 3 2 1 3
IA = (0 1 0) (3 0 4) = (3 0 4)
0 0 1 1 −2 2 1 −2 2

174
3.4. M A T R I Z I N V E R S A (A-1)

Es aquella matriz que al multiplicarse por su original, tanto por el lado derecho
como por el izquierdo, el resultado debe ser la matriz unitaria (I), es decir que
A • A−1 = I = A−1 A

En general se puede definir de la siguiente manera


Sea A una matriz de nxn con elementos en C. una matriz X se dice que es inversa
de A (A-1) si

XA = In = AX

Nota:

Si A-1 existe, entonces A es no singular.


Si A-1 no existe, entonces A es singular.

175
3.4.1. Método por la fórmula para matriz de 2x2.

Para hallar la inversa de una matriz de orden dos, se siguen los pasos siguientes:

1. Se calcula el determinante de la matriz.

2. Se arregla la matriz original de la siguiente manera, los elementos de la


diagonal principal de la matriz, se intercambian de posición, y a los
elementos de la diagonal secundaria se le cambian de signo.

3. Se multiplica el inverso del determinante calculado en paso 1, por la


matriz obtenida en el paso 2, de esta manera se obtiene la fórmula
general para calcular la inversa de cualquier matriz de orden 2
únicamente, (las de otro orden debe calcularse de otra forma, que
veremos posteriormente).

Esto se muestra a continuación

𝑎 𝑐
Si 𝐴 = ( ) diagonal principal diagonal secundaria
𝑏 𝑑

Paso 1.

𝑎 𝑐
det 𝐴 = | | = 𝑎𝑑 − 𝑐𝑏
𝑏 𝑑

Paso 2 y 3

1 d −c
A−1 = ( )
Det A −b a

1 d −c
A−1 = ( )
ad − bc −b a

176
Ejemplos:

1.-Comprobar que A • A−1 = I = A−1 A , calcular la inversa por la formula deducida

Sea:

4 5
A=( ) A−1 =?
−2 3
4 5 1 3 −5
det A = ( ) = 12 + 10 = 22 Entonces A−1 = ( )
−2 3 22 2 4

Ahora comprobando.

4 5 1 3 −5
A • A−1 = ( )• ( )
−2 3 22 2 4

1 4 5 3 −5
A • A−1 = ( )( )
22 −2 3 2 4

Haciendo el producto tenemos:

1 22 0
A • A−1 = ( )
22 0 22

1 0
A • A−1 = ( )
0 1
A • A−1 = I

2.-Determinación de la inversa de una matriz de 2 x 2

Sea

4 5
𝐴=[ ]
2 3

Determine 𝐴−1 y comprobar que AA−1 = A−1 A = I2


Solución: Al aplicar la regla se obtiene

1 1 3 −5 3 5
3 −5
𝐴−1 = [ ]= [ ] = [ 2 − 2]
4 • 3 − 5 • 2 −2 4 2 −2 4
−1 2

177
Para comprobar que en realidad es la inversa de A, se calcula AA−1 y A−1 A

4 5 3/2 −5/2 1 0
𝐴𝐴−1 = [ ][ ]=[ ]
2 3 −1 2 0 1

3/2 −5/2 4 5 1 0
A−1 A = [ ][ ]=[ ]
−1 2 2 3 0 1

La cantidad ad-bc que aparece en la regla para calcular la inversa se llama

Determinante de la matriz. Si el determinante es 0, la matriz no tiene inversa.

178
3.4.2.- Método por operaciones elementales para Matrices de
nxn

En este método, se usa las operaciones elementales que se le pueden aplicar a las
filas de las matrices, sin que se alteren, estas son como ya sabemos suma, resta,
multiplicación por un escalar y división.
Este método es aplicable a matrices de cualquier orden.
La forma de realizarlo es la siguiente:
1. A la matriz original, se le extiende con la matriz unitaria del mismo orden.

2. Se le aplican las operaciones elementales a las filas que así lo requieran,


para que al finalizar, la matriz unitaria que se escribió por el lado derecho
en el paso uno, al extenderlo, quede ahora en el lado izquierdo.

3. Una vez obtenido el intercambio de las matrices como se indica en el paso


dos, la matriz que queda al lado derecho, es la matriz inversa buscada.

4. Si la reducción de A conduce a un renglón de ceros a la izquierda de la


barra vertical, entonces A no es invertible.

A continuación, tomaremos el ejemplo que se resolvió con el método anterior, para


comprobar este método.

Ejemplos:

1.-Sea A, la matriz del ejemplo anterior

4 5
A=( )
−2 3

Paso 1, primeramente se extiende.

(A|I) = ( 4 5|1 0)
−2 3 0 1

179
Paso 2. Se aplica una primera operación elemental, para hacer uno el primer
elemento (el 4) de la primera fila y primera columna

A I   
4 5 1 0 1

 4 F1

 2 3 0 1
A continuación, se hace cero el elemento de la segunda fila y segunda columna (-
2), utilizando el uno (1) de la primera fila, con la operación que se indica a la
derecha (2F1+F2)

2 10 2 0
 1 5 1 0 4 4
 4 4  2F1  F2  F2 2 3 0 1
  2 3 0 1
  0 11 2 0
2 4
Resultado de la operación anterior.
1 5 1 0
 4 4 
 0 11 2 1 
 2 4 
Ahora hacemos uno, el elemento de la segunda fila y segunda columna ( 11 )
2
realizando la operación que se muestra a la derecha.
1 5 1 0
 4 4 2 F F
 0 11 2 1  11
2 1

 2 4 

Resultado de la operación
1 5 1 0 
 4 4
0 1 1 2 
 11 11

Ahora, con el uno obtenido, hacemos cero el elemento de la primera fila segunda
columna ( 5 4 ).
0 5 5 5
1 5 1  4 44 22
 0  5 F F F 5 1
4 4 1 0
0 1 1 2  4 2 1 1 4 4
 11 11 3 5
1 0
22 22
Resultado de la operación anterior
1 0 3 5 
 22 22 
0 1 1 2 
 11 11 

180
Como se puede observar, la matriz unitaria ya está del lado izquierdo, por lo tanto,
la del lado derecho de la línea dentro de la matriz, es la inversa buscada.

Arreglando dicha matriz, obtenemos la misma matriz inversa obtenida con el otro
método

1 3 −5
A−1 = ( )
22 2 4

Comprobación:

4 5 1 3 −5 1 4 5 3 −5
AA−1 = ( ) ( ) → A • A−1 = ( )( )
−2 3 22 2 4 22 −2 3 2 4

1 22 0
A • A−1 = ( )
22 0 22

Finalmente

1 0
A • A−1 = I = ( )
0 1

2.-Calcule la inversa de la matriz siguiente:

3 5
A=( ) A−1 =?
−2 1

 3 5 1 0 1

  2 1 0 1  3 F1
 

2 10 2 0
 1 5 1 0 3 3
 3 3  2F1  F2  F2 2 1 0 1
  2 1 0 1
  13 2
0 1
3 3

1 5 1 0
 3 3  3 F
 0 13 2 1  13
2

 3 3 

181
0 5  10  15
1 5 1 0  3 39 39
 3 3  5 F2  F1  F1 1 5 1 0
0 1 13 13 
2 3 3 3 3

1 0 1 5
13 13

 1 0 113  513  1 1 −5
 𝐴−1 = ( )
0 1 2 3  13 2 3
 13 13 

Comprobación

3 5 1 1 −5
AA−1 = ( ) ( )
−2 1 13 2 3

1 3 5 1 −5
AA−1 = ( )( )
13 −2 1 2 3

1 13 0 1 0
AA−1 = ( )=I=( )
13 0 13 0 1

3.-Determinación de la inversa de una matriz de 3 x 3.


Calcule la inversa de la matriz A y verificar que AA−1 = A−1 A = I3

1 −2 −4
𝐴=[ 2 −3 −6]
−3 6 15

Solución: Primero se comienza con la matriz extendida de 3 x 6 cuya mitad


izquierda es A y la mitad derecha es la matriz identidad.

 1 2 4 1 0 0 
 
 2 3 6 0 1 0 
 3 6 15 0 0 1

182
A continuación, se transforma la mitad izquierda de esta nueva matriz, en la matriz
identidad, con la siguiente secuencia de operaciones elementales sobre renglones
en toda la nueva matriz.
 1 2 4 1 0 0   1 2 4 1 0 0 
  2 F  F  
 2 3 6 0 1 0 
 3F  F  0
1
1
1 2 2 1 0  
2
3
1/ 3 F
2F F
 3
3
2

 3 6 15 0 0 1 0 0 3 3 0 1

 1 2 4 1 0 0  1 0 0 3 2 0
  4F F  
 0 1 0 4 1 2 / 3 
2F F
 3
2
1
1  0 1 0 4 1 2 / 3
0 0 1 1 0 1/ 3 0 0 1 1 0 1/ 3

Ya se ha transformado la mitad izquierda, en la matriz identidad, esto quiere decir


que se ha puesto la matriz en forma escalonada reducida, y con esto hemos
obtenido en la mitad derecha 𝐴−1 .

−3 2 0
2
−4 1 −
𝐴−1 = 3
1
[1 0
3 ]

4.-Determine la inversa de la siguiente matriz.

2 −3 −7
[1 2 7]
1 1 4

Solución. Se procede como sigue:

 2 3 7 1 0 0  1 2 7 0 1 0
  F F   2 F  F
1 2 7 0 1 0    1 2
 2 3 7 1 0 0  
F F
1
1
 2
3

 1 1 4 0 0 1  1 1 4 0 0 1

183
1 2 70 1 0 1 2 7 0 1 0
  1/ 7 F2  
0 7 21 1 2 0   0 1 3 1/ 7 2 / 7 0  
F F 2
2 F  F
2
3
1

0 1 3 0 1 1 0 1 3 0 1 1

1 0 1 2 / 7 3 / 7 0
 
 0 1 3 1/ 7 2 / 7 0
0 0 0 1/ 7 5 / 7 1

Si hay un renglón de ceros en el lado izquierdo, cuando se trata de determinar


una inversa, como en el ejemplo anterior, la matriz original no tiene inversa.

El ejemplo anterior, indica que no toda matriz cuadrada tiene inversa.

5.-Otro ejemplo, pero ahora se trata de una matriz de cuarto orden y hay que
hallar el valor de “m”.

4 0 m 1
 
 2 1 3 0 
Si A   , hallar 𝐴−1 como función de “m”, así como el valor o los
0 2 1  1
 
 1 0 1 2 
 
valores de “m” para los cuales A-1 existe y comprobarlo.

Solución:
4 0 m 1 1 0 0 0
 
 2 1 3 0 0 1 0 0
0 2 1 1 0 0 1 0
 
1 0  1 2 0 0 0 1 

En el caso de este tipo de problemas, es conveniente que la literal que se tiene


como incógnita se coloque en las últimas filas, para facilitar el manejo de esta,
hasta el final, tal como se verá en este problema.
Véase, que se utilizó el método de las operaciones elementales para resolverlo, y
en estas, todos los movimientos u operaciones que son permitidos se deberá de
hacer solamente cuando la matriz ya este extendida nunca antes, y además,
nótese que estas operaciones solamente se hacen con las filas de la matriz.

184
Nota: (−2𝐹1 + 𝐹2 = 𝐹2 ) esta nomenclatura que se usa, indica lo siguiente, se
multiplica por menos dos a la fila uno, y posteriormente se le agrega la fila dos ,
dando por resultado una nueva fila dos, que es la que ocupara el lugar de la fila
dos inicial
 1 0 1 2 0 0 0 1
 
 2 1 3 0 0 1 0 0
 0 2 1 1 0 0 1 0
 
 4 0 m 1 1 0 0 0
 

 2 F1  F2  F2
2 0 2 4 0 0 0 2
2 1 3 0 0 1 0 0
0 1 5 4 0 1 0 2
 4 F1  F4  F4

4 0 4 8 0 0 0  4
4 0 m 1 1 0 0 0
0 0 m4 7 1 0 0 4

 1 0 1 2 0 0 0 1 
 
 0 1 5 4 0 1 0  2
  1F2
0 2 1 1 0 0 1 0 
 
 m4 7  4 
 0 0 1 0 0

 1 0 1 2 0 0 0 1 
 
 0 1 5 4 0 1 0 2 
 0 2 1 1 0 0 1 0   2 F2  F3  F3
 
 m4 7  4 
 0 0 1 0 0

 2 F2 0  2 10  8 0 2 0  4
F 3 0 2 1 1 0 0 1 0
0 11  9 0 2 1 4
"
F3 0

185
 1 0 1 2 0 0 0 1 
 
 0 1 5 4 0 1 0 2 
 0 0 11 9 0 2 1  4
 
 m4 7  4 
 0 0 1 0 0

1
F3 0 0 1 9 0 2 1 4
11 11 11 11 11
 F1 1 0 1 2 0 0 0 1
1 0 11 13 0 2 1 7
11 11 11 11

 1 0 1 2 0 0 0 1 
 
 0 1 5 4 0 1 0 2  1
 0 0 11 9 0 2 1  4  11
F3
 
 m4 7  4 
 0 0 1 0 0

0 0 1 9 0 2 1 4
11 11 11 11
1 0 1 2 0 0 0 1
1 0 0 13 0 2 1 7
11 11 11 11

5  45 10 5  20
F3 0 0 5 0
11 11 11 11 11
 F2 0 1 5 4 0 1 0 2
0 1 0 1 0 1 5 2
11 11 11 11

(m  4) 9m  4 - 2m  4  m  4 4m  4


 F3 0 0  m  4 0
11 11 11 11 11
 F4 0 0 m  4 7 1 0 0 4
9m  41  2m  4  m  4 4m - 28
0 0 0 1
11 11 11 11

9m  4 9m  4  77 9m  36  77 9m  41
7   
11 11 11 11
4m  4 4m  4  44 4m  16  44 4m  28
4   
11 11 11 11
 18m  4 2  18m  4  29m  41  18m  72  18m  82  154
   
119m  4 11 119m  41 119m  41 119m  41

186
14

9m  41
9 9 - 18m  4  9m  4 9m  4 
0 0 0
11 9m  41 119m - 41 119m  41 119m  41
0 0 1 9 0 2 1 4
11 11 11 11
9 14 7 8
0 0 1 0  - -
9m  41 9m  41 9m - 41 9m - 41

9m  4 1 9m  4  9m  41  9m  36  9m  41  77


    
119m  41 11 119m  41 119m  41 119m  41
7

9m  41

1 1 - 2m  4  m  4 4m  4 
0 0 0 -
11 9m  41 119m - 41 119m  41 119m  41
0 1 0 1 0 -1 5 1
11 11 11 11
1 m3 4m  19 2m - 10
0 1 0 0 
9m  41 9m  41 9m - 41 9m - 41

  13 2(2m  1) (2m  1) ( m  7) 
1 0 0 0 
 9m  41 9m  41 9m  41 9m  41 
0 1 (m  3) (4m  19) 2(m  5) 
1 0 0
 9m  41 9m  41 9m  41 9m  41 
 9  14 7 8 
0 0 1 0 
 9m  41 9m  41 9m  41 9m  41 
 0 11  2(m  4)  (m  4) 4(m  7) 
0 0 1 
 9m  41 9m  41 9m  41 9m  41 

Finalmente arreglando el lado derecho de la línea de la matriz anterior, obtenemos


la inversa que se busca.

  13 2(2m  1) (2m  1) (m  7) 
 
1  1 (m  3) (4m  19) 2(m  5) 
-1
A =
9m  41  9  14 7 8 
 
 11  2(m  4)  (m  4) 4(m  7) 
 

187
De aquí que podemos decir que para que esta matriz inversa exista, igualamos a
cero 9m-41, y despejamos a “m”.
Es decir:

41
9𝑚 − 41 = 0 → 𝑚 =
9

Por lo tanto.

41
Si m = entonces A−1 no existe
9

41
Si m ≠ entonces A−1 existe
9

3.4.2.1. Propiedades

1.- 𝐀−𝟏 𝐞𝐬 ú𝐧𝐢𝐜𝐚

2.- (𝐀−𝟏 )−𝟏 = 𝐀

3.-(𝐀𝐁)−𝟏 = 𝐁 −𝟏 𝐀−𝟏
𝟏
4.-(𝜶𝑨)−𝟏 = 𝜶−𝟏 • 𝑨−𝟏 = 𝑨−𝟏
𝜶

Para la matriz identidad se cumple también que:

Si A es una matriz m * n

1.- Im A = A
2.- AIn = A

188
Ejemplo:

1.-Sea

3 2 −1
𝐴=( )
4 3 5
2*3
m*n

Para la matriz A, debe de multiplicarse por una matriz unitaria de orden “n”(3x3)
por el lado derecho, y nos dará como resultado la misma matriz A, como se ve a
continuación.

1 0 0
3 2 −1 3 2 −1
( ) (0 1 0) = ( ) = AIn
4 3 5 4 3 5
0 0 1

Por otro lado, debe multiplicarse por una matriz de orden “m” (2x2) por el lado
izquierdo, y nos dará como resultado la misma matriz.

1 0 3 2 −1 3 2 −1
( )( )=( ) = Im A
0 1 4 3 5 4 3 5

Lo anterior es lo que indica las dos propiedades anteriores.

189
EJERCICIOS PROPUESTOS

19 – 20. Determine la inversa de la matriz y compruebe que A−1 A = AA−1 = I2


y que B −1 B = BB −1 = I3 .

1 3 2
7 4
19.- 𝐴 = [ ] 20.-𝐵 = [ 0 2 2]
3 2
−2 −1 0

21 – 32. Determine la inversa de la matriz, en caso que exista.

5 3 3 4 2 5
21.-[ ] 22.-[ ] 23.-[ ]
3 2 7 9 −5 −13
1 1
−7 4 6 −3
24.-[ ] 25.-[ ] 26.-[ 2 3 ]
8 −5 −8 4 5 4

4 2 3 2 4 1
0.4 −1.2
27.-[ ] 28.-[3 3 2] 29.- [−1 1 −1]
0.3 0.6
1 0 1 1 4 0

5 7 4 1 2 3 2 1 0
30.-[3 −1 3] 31.-[4 5 −1 ] 32.-[1 1 4]
6 7 5 1 −1 −10 2 1 2

190
3.5. Algebra Matricial.

Algunas operaciones del algebra matricial, son parecidas a las del algebra
elemental, sobre todo con respecto a la suma y resta de matrices, puesto que
como ya sabemos para hacer estas operaciones solamente deben ser del mismo
orden y se pueden trasladar las matrices de un miembro a otro solamente
cambiando de signo.
La diferencia se presenta cuando queremos despejar alguna matriz que este como
incógnita la ecuación matricial, sobre todo si ésta está multiplicando a otra matriz
u a otra expresión matricial ya sea por la izquierda o por la derecha, puesto que
aquí es donde el algebra elemental no es aplicable, debido principalmente porque
entre las matrices no existe la división, y por lo tanto la operación de despeje debe
hacerse siguiendo las reglas del algebra matricial.

La regla que debe de seguirse es que: por el lado que este la matriz afectando a la
incógnita, debe de multiplicarse por la inversa de esta para hacerlo la unidad,
aprovechando la propiedad de qué A • A−1 = I = A−1 A. Esto puede hacerse tantas
veces como sea posible para dejar a la matriz incógnita sola, como se muestra en
los ejemplos siguientes.

191
3.6. SOLUCIÓN DE ECUACIONES MATRICIALES

Ejemplo:
Para la ecuación matricial que se indican. Hallar a) El valor de X como función de
las variables que intervienen en la ecuación b) calcule el valor de X para las
matrices que se indican.
B(XA + B) = C − 3XA

Solución:
a)

Primeramente desarrollamos la ecuación y agrupamos del lado izquierdo los


términos que contienen la incógnita.

BXA + BB = C − 3XA
BXA + 3XA = C − BB = C − B 2
(B + 3)XA = C − B 2

En el siguiente paso, se convierte el escalar 3 en una matriz multiplicándolo por la


matriz unitaria ya que de esta manera se puede efectuar la resta entre la matriz B
y el 3, y también aplicamos la regala para el despeje de la incógnita, es decir
multiplicar por la inversa del término que está afectando a la incógnita por el lado
izquierdo en el miembro izquierdo, pero lo mismo hay que hacer con el miembro
del lado derecho por el mismo lado para aprovechar la propiedad
A • A−1 = I = A−1 A tal como se muestra en la ecuación siguiente.

(B − 3I)−1 (B − 3I)XA = (B − 3I)−1 (C − B 2 )


I XA = (B − 3I)−1 (C − B2 )
XA = (B − 3I)−1 (C − B 2 )
XAA−1 = (B − 3I)−1 (C − B 2 )A−1
X I = (B − 3I)−1 (C − B 2 )A−1
X = (B − 3I)−1 (C − B2 )A−1

192
Solución b).

Sean las matrices siguientes:

1 0 −2 2 1 −2
A=( ) B=( ) y C=( )
−1 2 0 −1 −1 1

Entonces como

1 1
  2 2   1 0   1  2    2 2    1 0  
2

x     3          


 0  1  0 1    1 1   0 1     1 2  
1
  5 2   1  2    2 2   2 2  1  2 0 
x           
 0  4    1 1   0 1  0 1  2  1 1 
1   4  2  1  2   4  2  1  2 0 
X       
20  0  5   1 1   0 1  2  1 1 
1   4  2   3 0  1  2 0
X      
20  0  5   1 0  2  1 1 
1 14 0  2 0  1  28 0  1 14 0 
X        
40  5 0  1 1  40  10 0  20  5 0 
1 14 0 
X    L.Q.Q.D.
20  5 0 

193
3.7. Tipos especiales de matrices de nxn.

En una matriz cuadrada se distinguen tres regiones, como se muestra a


continuación.

Triangulo superior
 a11 a12 a13 ..... a1n 
 
 a 21 a 22 a 23 ..... a 2 n 
a a32 a33 ..... a3n 
A  a ij    31 
 .................................... 
 
 .................................... 
a 
 m1 a m 2 a m3 ..... a mn  Triangulo inferior

Diagonal principal

La diagonal principal, está constituida por los elementos 𝑎𝑖𝑗 tales que 𝑖 = 𝑗, es
decir por los elementos de la forma 𝑎𝑖𝑖 .

Dichos elementos se encuentra ubicados en lo que geométricamente seria una de


las diagonales del cuadrado formado por la matriz.

3.7.1. Triangulares y sus propiedades

El triangulo superior: Está constituido por los elementos por encima de la diagonal
principal, es decir los elementos 𝑎𝑖𝑗 tales que 𝑖 < 𝑗

El triangulo inferior: Esta constituido todos los elementos por abajo de la diagonal
principal, es decir los elementos 𝑎𝑖𝑗 tales que 𝑖 > 𝑗.

Definición:

Sea 𝐴 = (𝑎𝑖𝑗 ) una matriz con nxn con elementos en C. Se dice que:

a. A es triangular superior si 𝑎𝑖𝑗 = 0 𝑝𝑎𝑟𝑎 𝑖 > 𝑗

194
b. A es triangular inferior si 𝑎𝑖𝑗 = 0 𝑝𝑎𝑟𝑎 𝑖 < 𝑗

1 3 5 1 0 0
𝐴 = (0 4 −1) 𝐴 = (5 4 0)
0 0 9 2 8 7

T. Superior T.Inferior

TEOREMA

Si A y B son dos matrices triangulares superiores (inferiores) del mismo orden y


𝛼 es elemento de C. Entonces.

1. A+B es triangular superior (inferior).

2. 𝛼𝐴 es triangular superior (inferior).

3. AB es triangular superior (inferior).

3.7.2. Diagonales

La diagonal principal es aquella que está formada por todos los elementos aii
tales que i = j

Definición:

Sea A(aij ) una matriz con nxn con elementos en C. Se dice que A es una matriz
diagonal si: aij = 0 para toda i ≠ j y se representa con

Diag(a11 , a22 , a33 , … … ann )

195
Ejemplo:

7 0 0
𝐴 = (0 5 0)
0 0 −2

Diag(7, 5, −2)

TEOREMA:

Si A y B son dos matrices diagonales tales que

A = diag (a11 , a22 , … . , ann ) y B = diag(b11 , b22 , … , bnn ) y α ε C,

Entonces:

a) A+B=diag (a11+b11, a22+b22, …,ann+bnn)


b) αA = diag (αa11 , αa22 , … , αann )
c) AB=diag (a11b11, a22b22, …,annbnn)
1 1 1
d) A−1 = diag (a , a , … , a ) , Si A es no singular.
11 22 nn

196
3.7.3. Matriz Escalar

Un caso especial de estas matrices es la conocida como matriz escalar, la cual


tiene la particularidad de que todos los elementos de la diagonal principal son
iguales.

Es decir una matriz 𝐴 = (𝑆𝑖𝑗 ) de nxn con elementos en C, se dice que es una matriz
Sij = 0 ∀ i ≠ j
escalar si {
Sij = k, ∀ i = j

Donde k es cualquier constante

Por la definición anterior, una matriz escalar es de la forma siguiente:

 s11 s12 s13 ........ s1n   k 0 0 0


0 0
   
 s 21 s 22 s 23 ....... s 2 n   0 k 0 0
0 0
s s s ...... s3n   0 0 k 0
0 0
  31 32 33    kI
 ...................................   0 0 0 0
k 0
   
 ...................................   
s   0 0 0 0 k 
 m1 s m 2 s m3 .... s mn   0

Donde I es la matriz unitaria de orden “n”, en consecuencia, premultiplicar una


matriz A por una matriz 𝑘𝐼, es equivalente a multiplicar por el escalar 𝑘. A esta
propiedad se le da el nombre de matriz escalar.

197
3.7.4. Traza de una matriz y sus propiedades.

Se conoce como traza de una matriz cuadrada al número que se obtiene sumando
los elementos de su diagonal principal.
Es decir, se define la traza de una matriz de la siguiente manera:

Definición:

Sea A = [aij ] una matriz de nxn con elementos en C.

Se llama traza de A, y se representa como Tr A, al número


𝑛

∑ 𝑎𝑖𝑖
𝑖=1

Ejemplo:

𝟏 𝟐 𝟑
(−𝟏 𝟎 𝟒)
𝟏 𝟓 𝟓

Tr A=1+0+5=6.
En general

𝒂𝟏𝟏 𝒂𝟏𝟐….. 𝒂𝟏𝒏


𝑨 = 𝒂𝟐𝟏 𝒂𝟐𝟐 𝒂𝟐𝒏
⋮ ⋮ ⋮
𝒂𝒎𝟏 𝒂𝒎𝟐 𝒂𝒎𝒏

Tr A=a 11+a 22+a 33++……..amn

198
TEOREMA:

Si A y B son dos matrices de nxn con elementos en C y 𝛼 elemento también de C.


entonces:

1.- Tr(A + B) = (Tr A) + (Tr B)


2.- Tr(αA) = α(Tr A)
3.- Tr(AB) = (Tr BA)

3.8. Otros tipos de matrices (m x n)

3.8.1. Transpuesta y sus propiedades.

Transpuesta: Sea A = (aij ) una matriz de m x n. Entonces la transpuesta de A, que


se escribe A′ , es la matriz de n x m obtenida al intercambiar los renglones por las
columnas de A. De manera breve, se puede escribir A′ = (aji ). En otras palabras.

Definición:

Sea A = (aij ) una matriz de “mxn” con elementasen C. se llama transpuesta de A, a


la matriz “nxm”.

AT = (aij ) tal que aij = aji

𝐚𝟏𝟏 𝐚𝟏𝟐….. 𝐚𝟏𝐧 𝐚𝟏𝟏 𝐚𝟏𝟐….. 𝐚𝟏𝐧


Si 𝐀 = 𝐚𝟐𝟏 𝐚𝟐𝟐 𝐚𝟐𝐧 , entonces 𝐀𝐓 = 𝐚𝟐𝟏 𝐚𝟐𝟐 𝐚𝟐𝐧
⋮ ⋮ ⋮ ⋮ ⋮ ⋮
𝒂𝒎𝟏 𝒂𝒎𝟐 𝒂𝒎𝒏 𝒂𝟏𝒏 𝒂𝟐𝒏 𝒂𝒎𝒏

Simplemente se coloca el renglón “i” de A como la columna “i” de AT , y la columna


j de A como el renglón j de AT .

199
TEOREMA:

Si A Y B son dos matrices con elementos en C y 𝛼 𝜀 𝐶, entonces:

1) (AT )T = A
2) (𝛼𝐴)𝑇 = 𝛼𝐴𝑇
3) (A + B)T = AT + B T , si A + B puede obtenerse
4) (AB)T = B T AT , si AB puede obtenerse
Ejemplo:

1.-Encuentre la transpuesta de la siguiente matriz.

1 2  6
 
2  3 4
C 
0 1 2
 
2 1 5 

Solución: Al intercambiar los renglones y las columnas de cada matriz se obtiene

 1 2 0 2
 
C   2  3 1  1
T

 6 4 2 5 
 

Las propiedades de la transposición se presentan en el siguiente teorema.

200
3.8.1.2.”PROBLEMAS PROPUESTOS”

39 – 44. En los siguientes problemas encuentre la transpuesta de las matrices


dadas.

2 3
−1 4 3 0
39.-( ) 40.-( ) 41.-(−1 2)
6 5 1 2
1 4

1 2 3 𝑎 𝑏 𝑐
2 −1 0
42.-(−1 0 4) 43.-(𝑑 𝑒 𝑓) 44.-( )
1 5 6
1 5 5 𝑔 ℎ 𝑖

201
3.8.2. Simétrica.

Matriz simétrica. Una matriz cuadrada A, se dice que es simétrica, si


T T
A = A(o´A = A )
Es decir 𝑎𝑖𝑗 = 𝑎𝑗𝑖

Definición:

Sea A una matriz de nxn con elementos en C. Se dice que:

1) A es simétrica si AT=A

Ejemplo: Las matrices siguientes son simétricas

−𝟐 𝟏 𝟕
𝟏 𝟐
[ ], [𝟏 𝟑 𝟒]
𝟐 𝟑
𝟕 𝟒 𝟓

De manera análoga pueden definirse las matrices antisimétricas.

3.8.3. Antisimétricas y propiedades.

Matriz antisimétrica. Una matriz A se dice que es antisimétrica si A = −AT es decir


que 𝑎𝑖𝑗 = −𝑎𝑗𝑖 . Para toda 𝑖, 𝑗.

En general se puede definir lo siguiente:

Sea A una matriz de nxn con elementos en C. Se dice que:


 Es simetrica si A  A
T

A 

 Es antisimetrica si A   A
T

Las características que deben de tener estas, son.

202
Para la simétrica

(𝑎𝑖𝑗 ) = (𝑎𝑗𝑖 ) 𝑦 𝑎𝑖𝑗 = 𝑎𝑗𝑖

Y para la Antisimétrica,

(𝑎𝑖𝑗 ) = (−𝑎𝑗𝑖 ) 𝑦 𝑎𝑖𝑗 = −𝑎𝑗𝑖 ;  i. j

Lo que quiere decir, que los elementos simétricos con respecto a la diagonal
principal debe ser uno el negativo del otro, además debe de cumplirse que para
toda 𝑖 = 𝑗 𝑞𝑢𝑒 𝑎𝑖𝑖 = −𝑎𝑖𝑖 𝑒𝑠 𝑑𝑒𝑐𝑖𝑟 𝑞𝑢𝑒 𝑎𝑖𝑖 = 0 por lo que se puede deducir, que los
elementos de la diagonal principal deben valer cero.

Ejemplo de matrices antisimétricas.

 0 1  2i  2  2i 
 0 2 1  i  1
 2 0 i   0 5  4 
   2i 5 0 i 
 1  i i 0   
2  2i 4 i 0 

0 2 3 0 −1 3
[−2 0 −4] , [1 0 2].
−3 4 0 −3 −2 0

Propiedades.

Si A y B son dos matrices simétricas (antisimétricas) de nxn y α ∈ C, entonces


1. A+B es simétrica (Antisimétrica).
2. 𝑘𝐴 es Simétrica (Antisimétrica)
3. A+AT es Simétrica.
4. A-AT es Antisimétrica

203
3.8.4. Conjugadas y propiedades.

La conjugación transforma una matriz en otra llamada su conjugado, cuyos


elementos son los conjugados de los elementos correspondientes en la matriz
original.

Definición:

Sea A = [aij ] una matriz “mxn” con elementos en C. se llama conjugada de A a la


matriz de “mxn” A ̅ = [cij ] tal que:

cij = a̅ij

Ejemplo:

3 3𝑖
Para la matriz. 𝐴 = [ 2𝑖 1 ]
−𝑖 −2 − 𝑖
𝑐11 𝑐12 𝑎11 𝑎21 3 −3𝑖
𝐴̅ = [𝑐21 𝑐22 ] = [𝑎12 𝑎22 ] = [−2𝑖 1 ]
𝑐31 𝑐32 𝑎13 𝑎23 +𝑖 −2 + 𝑖

Propiedades.

Sea 𝐴 = (𝑎𝑖𝑗 ) una matriz “mxn” con elementos en C y 𝛼 elemento de C. Entonces


1. −(𝐴̿) = A
2.- ̅̅̅̅
𝛼𝐴 = 𝛼̅𝐴̅
3.- ̅̅̅̅̅̅̅
A+B=A ̅+B ̅ Si A + B puede obtenerse
4.- AB
̅̅̅̅ = A̅ B
̅ , Si AB puede obtenerse

Las siguientes matrices, son casos especiales, en las cuales se utiliza el arreglo
conjugada y transpuesta, que lo denotaremos como A∗ = (A̅)T = (A̅̅̅̅
T ).

204
3.8.5. Matriz nula, Hermitiana y antihermitiana y propiedades.

Si todos sus elementos son cero. También se denomina matriz cero y se denota
por m × n.

Hermitiana y Antihermitiana.

Definición:

Sea A una matriz de “nxn” con elementos en C. Se dice que:

Es hermitiana si A∗ = A
A={
𝐸𝑠 𝑎𝑛𝑡𝑖ℎ𝑒𝑟𝑚𝑖𝑡𝑖𝑎𝑛𝑎 𝑠𝑖 A∗ = −A

De acuerdo a la definición se deduce que para una matriz hermitiana, sus


elementos deben de cumplir lo siguiente:
1. 𝑎𝑖𝑗 = 𝑎̅𝑗𝑖 ∀ i, j
2. 𝑎𝑖𝑖 = 𝑎̅𝑖𝑖 → I(aii ) = 0

Estas dos condiciones se pueden interpretar de la siguiente manera, que los


elementos simétricos respecto a la diagonal principal deben ser conjugados, y
segundo, que la parte imaginaria de la diagonal principal deben ser cero, lo que
indica que los elementos de la diagonal principal deben ser puramente reales.

205
Antihermitiana

Para una matriz antihermitiana, debe de cumplirse las condiciones siguientes:

1. 𝑎𝑖𝑗 = −𝑎𝑗𝑖  i, j
̅̅̅
2. 𝑎𝑖𝑖 = 𝑎 𝑖𝑖 → R(a ii ) = 0
̅̅̅

Es decir que: los elementos simétricos respecto a la diagonal principal, deben ser
tales, que sus partes reales solo difieran en signo, y sus partes imaginarias sean
iguales.

Por otra parte que los elementos de la diagonal principal deben ser números
imaginarios.

A continuación se muestran algunos ejemplos: De matrices hermitianas

a1 2  a2 1  2  i
−3 2−𝑖 3𝑖 
2 −1 + 𝑖 a1 3  a3 1  3i
[2 + 𝑖 0 −1 + 𝑖] [ ] ya que 
a2 3  a3 2  1  i
−1 − 𝑖 −3
−3𝑖 −1 − 𝑖 1
 I (a )  I (a )  I (a )  0
 11 22 33

 i 2 1 i 3 
  2  2i   1 
 i   2i 2  2i 
1  i  i  6  i  2  2i i 
matrices antihermitianas
0 
 
 3 1 6  i 5i 

a1 2  a 2 1  2

a1 3  a3 1  1  i

a 2 3  a3 2  i

ya que a1 4  a 4 1  3

a 2 4  a 4 2  1
a  a  6  i
 34 43

 R(a1 1 )  R(a 2 2 )  R(a3 3 )  R(a 4 4 )  0


206
TEOREMA:

Si A y B son dos matrices hermitianas (Antihermitianas) cuadradas, entonces A+B


es hermitiana (antihermitiana).

Demostración:
Sean A y B matrices antihermitianas, del mismo orden, por la propiedad de las
conjugadas transpuestas sabemos que

(A+B)* =A*+ B*

Si A y B son antihermitianas por la definición

A* =-A y B* =-B

Por lo que se puede escribir lo siguiente

(A+B)* = -A+ (-B)= -(A+B)

Por lo que se concluye que (A+B) es antihermitiana.

Propiedades.

Sea 𝐴 = (𝑎𝑖𝑗 ) una matriz “mxn” con elementos en C. por lo tanto

1. AA* Es hermitiana
2. A*A Es hermitiana
3. A+A* Es hermitiana, si A es cuadrada
4. A-A* Es antihermitiana si A es cuadrada

207
3.9. Potenciación.

De la misma manera que en el caso de los números, se conoce como potencia


enésima de una matriz cuadrada cualquier producto

AAA…………An

Definición:

Sea A una matriz de “nxn” con elementos en C. y sean 𝑛 ∈ 𝑁 .Se llama potencia
enésima de A, y se representa como An, a la matriz definida por

A0 = Im

An = AAn−1 , para n ≥ 1

La potenciación cumple con el siguiente:

TEOREMA:

Sea A una matriz de “nxn” con elementos en C. y m, n ∈ N entonces.

Am An = Am+n y (Am )n = Amn

Demostración:

Demostrar que Am An = Am+n


Demostraremos primero que
Am A = AAm  m ∈N

En efecto para m=1 la proposición establece que

A1 A = AA1
Pero como A1 =A A0 =A I=A

Se concluye que A A=A A por lo que la proposición es válida para m=1

Ahora planteamos para una m=k

Ak A=A Ak

208
Y Premultiplicando por A se tiene

A (Ak A)=A(A Ak)

Reasociando

(A Ak)A=A(A Ak)

Ak+1 A=A Ak+1

Lo que se demuestra que es válido para m=k

Como también sabemos


Am Ak=Am+k

Premultiplicando por A se obtiene

A (Am Ak)=A Am+k

Reagrupando y haciendo operaciones tendremos

(A Am) Ak =A Am+k propiedad asociativa

(Am A) Ak =A Am+k propiedad conmutativa

Am (A Ak ) =A Am+k propiedad asociativa

Am Ak+1 =Am+k+1 por definición de potenciación

Por lo que se demuestra la primera propiedad.

209
3.10. Partición de Matrices.

En mucha ocasiones se considera que una matriz está formada por otras matrices
más pequeñas. Esto ocurre, por ejemplo, en aplicaciones en las que aparecen
grandes matrices con un elevado número de ceros.

Debido a esto, se introduce lo que llamaremos partición de matrices


Una matriz, se puede subdividir o partir en matrices más pequeñas insertando
rectas horizontales y verticales entre renglones y columnas, a las cuales lo
llamaremos submatrices y a la matrices mas grandes le llamaremos hipermatrices,
esto es para facilitar la comprensión del concepto de partición.

Ejemplo:
Dadas las matrices:

 2 3
1 0 2 1 3  4 5
0 1 5 1 0   
A y B   1 1
0 0 1 2 1  
   2 1
0 0 2 3 5
 1 1

Calcular el producto AB.

Solución. Antes de hacer el producto observemos que la matriz A (que lo


llamaremos hipermatriz) tiene como submatrices a la matriz identidad de tamaño 2
x 2 y a la matriz nula del mismo tamaño (véase el ejemplo).

Por lo que, trazando las rectas en la hipermatriz, se puede observar lo que sigue:

1 0 2 1 3
0 1 5 1 0  2 −1 3 1 2 −1
A y llamando P = [ ] y Q=[ ],
0 0 1 2 1 5 1 0 2 3 5
 
0 0 2 3 5

210
Podemos considerar que la matriz A está formada por cuatro submatrices, como se
ve en el siguiente arreglo.

I P
A=[ 2 ]
O2X2 Q

El producto de I2 Y O2X2 por otras matrices de tamaño adecuado es muy sencillo.


Para realizar el producto AB sólo queda, partir la hipermatriz B, obteniéndose las
submatrices R y S, tal como se ve abajo, las cuales debemos multiplicar por
I2 Y O2X2 y por P y Q.

De hecho consideremos lo que se muestra abajo.

2 3 
4 5 
  R
B   1 1   
  S
 2 1
 1 1
 

1 1
2 3
Siendo 𝑅 = [ ] 𝑦 𝑆 = [2 −1]
4 5
1 1

Tendremos

5 9 
 
 I P   R   R  QS
P  11 9 
AB        
O Q   S   QS   4 2 
 
13 4 

Nótese que se realizan productos de matrices de tamaño inferior al de las


originales.

211
3.11. Introducción a los determinantes

La idea de los determinantes, es en realidad más antigua que la de las matrices.


Descubierta por Cramer durante sus trabajos orientados en la solución de
ecuaciones lineales, fue expuesta por primera vez en 1750, cien años antes que
Silvestre y Carley empezaran a hablar de matrices.

En la actualidad, sin embargo, el concepto de determinante suele presentarse


como consecuencia de la teoría de matrices, y ha sufrido, incluso el proceso de
“axiomatización”, de la cual surge una definición integrada por cuatro postulados.

En este caso, estableceremos una definición para el concepto de determinante a


partir de un razonamiento similar al que históricamente le dio origen, aunque
emplearemos en ella el concepto de matriz. En la deducción de las propiedades y
en la presentación de los métodos para el cálculo de determinantes, se manejara
algunos elementos de la teoría de matrices

Si una matriz es cuadrada, es decir, si tiene la misma cantidad de renglones que


de columnas, se le puede asignar un número, llamado su determinante.
Una tabla ordenada n x n de escalares, situada entre dos líneas verticales, se le
conoce como determinante de orden n.

212
3.11.1. Definición

En forma generalizada, la definición de un determinante es:


Sea A = [aij ] una matriz de nxn con elementos en C, y sea Pk = (αk1 , αk2 , αk3 … α)
una permutación del conjunto (1,2, … … … n). Se llama determinante de A al número
𝑛! 𝑛

𝐷𝑒𝑡. 𝐴 = ∑ 𝛿(𝑃𝑘 ) ∐ 𝑎𝑖 𝑎𝑘𝑖


𝑘=1 1

; si Pk es de clase par


donde  ( Pk )  
; si Pk es de clase impar

Veremos, que los determinantes, es una herramienta indispensable en el estudio y


solución de sistemas de ecuaciones lineales.

213
3.11.2 .Cálculo de determinantes de orden 1 y 2.

Se representará el determinante de una matriz cuadrada A, por el símbolo |𝐴| y


comenzaremos definiendo |𝐴| para el caso más sencillo. Si A es una matriz de 1 x
1, tiene sólo un elemento, y su determinante se define como el valor de ese
elemento; esto es,

𝑆𝑖 𝐴 = [𝑎], 𝑒𝑛𝑡𝑜𝑛𝑐𝑒𝑠 |𝐴| = 𝑎

Si A es una matriz de 2 x 2,
𝑎11 𝑎12
𝐴 = (𝑎
21 𝑎22 ) Se llama determinante de A al número real
𝑎11 𝑎12
det(𝐴) = |𝑎 𝑎22 | = 𝑎11 • 𝑎22 − 𝑎12 • 𝑎21
21

Ejemplo:

1) Dado que el determinante de orden uno es el mismo escalar, tenemos:

a) Si A = [24], entonces det. A = det. [24] = |24| = 24


b) Si B = [−3], entonces det. A = det. [−3] = |−3| = 3
3 5 3 5 3 5
c) Si C = [ ] → det. C = det. [ ]=| | = 3(1) − 2(5) = −7
2 1 2 1 2 1
2 −3 2 −3 2 −3
d) Si D = [ ] → det. D = det. [ ]=| | = 2(−4) − 1(−3) = −5
1 −4 1 −4 1 −4

214
3.11.3 Determinantes de orden 3.
3.11.3.1. Método de Sarrus.

Esta regla, nos indica que debemos de extender el determinante, esto consiste
en aumentar las dos primeras columnas a la derecha de la tercera (o las dos
primeras filas debajo de la tercera), y multiplicar los elementos en forma
diagonal, de la parte superior izquierda a la parte inferior derecha del arreglo
(o viceversa), todas estas afectadas de un signo positivo, y posteriormente,
multiplicar los elemento de la parte inferior izquierda a la parte superior
derecha(o viceversa), de los elementos correspondientes, estas afectadas de
un signo negativo, tal como se muestra en ejemplo de abajo.

Ejemplo:

Dada una matriz cuadrada de orden tres, su determinante se calculará mediante la regla
de Sarrus.

a11 a12 a13 a11 a12 -


a21 a22 a23 a21 a22
a31 a32 a33 a31 a32
+

a11 a12 a13


a 21 a 22 a 23  (a11  a 22  a33  a12  a 23  a31  a 21  a32  a13 ) 
a31 a32 a33
(a13  a 22  a31  a12  a 21  a33  a 23  a32  a11 )
O bien

a11 a12 a13


a 21 a 22 a 23
a 31 a 32 a 33 (a11  a 22  a33  a12  a 23  a31  a 21  a32  a13 ) 
=
a11 a12 a13 (a13  a 22  a31  a12  a 21  a33  a 23  a32  a11 )
a 21 a 22 a 23

215
Obsérvese que hay seis productos, cada uno formado por tres elementos de la
matriz. Tres de los productos aparecen con signo positivo (conservan su signo)
y tres con signo negativo (cambian su signo).

NOTA: Debe tenerse cuidado, porque este método solo es válido para calcular
determinantes de orden 3 (3x 3).
Para calcular determinantes de orden 4, o de mayor orden, debe usarse otro
método, que se le conoce, como:” por menores o por cofactores”, el cual es
válido para cualquier orden de determinante.
Calcular el valor del determinante por el método de Sarrus:

3 2 1
| 0 2 −5|
−2 1 4

= (3)(2)(4) + (2)(−5)(−2) + (0)(1)(1) − (−2)(2)(1) − (0)(2)(4) − (1)(−5)(3)


= 24 + 20 + 0 − (−4) − 0 − (−15) = 63

Este método no funciona para determinantes de n x n si n > 3.

216
3.11.3.2 Desarrollo por cofactores.

Sea A, una matriz cuadrada, y a ij uno cualquiera de sus elementos. Si se suprime


la fila i y la columna j de la matriz A, se obtiene una submatriz M i j que recibe el
nombre de matriz complementaria del elemento a i j. en el ejemplo siguiente se
puede observar lo anterior.

Ejemplo:

Dada la matriz

La matriz complementaria del elemento a11 (el menor) es la matriz que resulta de
suprimir en la matriz A, la fila 1 y la columna 1; es decir:

𝑎22 … 𝑎2𝑛
𝑀11 = ( … … … )
𝑎𝑛2 … 𝑎𝑛𝑛

Definición.

El menor de un elemento es el determinante de la matriz que queda al eliminar la


fila y la columna en que aparece el citado elemento |M11 |.

Sea A una matriz de n x n. Entonces en general se puede calcular su


determinante por el método de los menores o cofactores siguiendo los
pasos.

217
1.- El menor Mij del elemento 𝑎𝑖𝑗 ; es el determinante de la matriz obtenida,
;
eliminando de la matriz A, el i-ésimo renglón y la j-ésima columna.
2.- El cofactor 𝐴𝑖𝑗 del elemento aij es Aij = (−1)i+j Mij .

Los signos se van alternando, según la posición que ocupen las entradas del
determinante(o van obteniéndose según la fórmula (−1)i+j. Es decir:

El determinante de una matriz cuadrada, es igual a la suma de los productos de


los elementos de una fila (o columna) cualquiera, multiplicados por sus respectivos
cofactores.

Es decir, en general el determinante de una matriz de orden nx n se puede hallar


de la siguiente forma.

Sea la matriz

Det. A = a11 (−1)i+j M11 + a21 (−1)i+j M21 + ⋯ + an1 (−1)i+j Mn1

Esta regla disminuye en una unidad, el orden del determinante que se pretende
calcular, es por esto que recibe el nombre de “por menores”.

Para evitar el cálculo de muchos determinantes, conviene elegir líneas o columnas


que tengan el mayor número de ceros, y si no es posible hay que calcular, todos

218
los que sean necesarios; esto es porque como el menor queda multiplicado por el
cofactor, y si el cofactor es cero, entonces todo el determinante se vuelve cero.

El cálculo del determinante anterior, se puede realizar utilizando cualquier renglón


o columna del determinante, sin que el valor de este cambie.

Ejemplo:

Si A es la matriz

3 1 0
[−2 −4 3 ]
5 4 −2

Evaluar det (A) por desarrollo de cofactores a lo largo de la primera columna de A.

Solución.

−4 3 1 0 1 0
Det. (A) = 3 | | − (−2) | | + 5| | = 3(−4) − (−2)(−2) + 5(3) = −1
4 −2 4 −2 −4 3

Nota: queda al lector realizar el cálculo por cualquier otra fila o columna para
comprobar.

219
3.11.3.3. Método por condensación.

El desarrollo por cofactores aunque de de aplicación general, no es un método


eficiente dada la gran cantidad de determinantes de orden menor que se
requiere calcular. Por ejemplo para un determinante de quinto orden, se
tendrán cinco determinantes de cuarto orden, y cada uno de ellos cuatro de
tercer orden por lo que se requiere calcular un total de 20 determinantes de
tercer orden.
Como se ha visto en algunos casos especiales, puede evitarse el calculo
de algunos cofactores cuando los elementos correspondientes son nulos; en
especial, si algunas de las líneas tuviese todos los elementos excepto una
iguales a cero, sin duda escogeríamos dicha línea para efectuar el desarrollo
por cofactores, ya que solo requeriríamos calcular uno de ellos (el
correspondiente al elemento distinto de cero). Esta situación claramente
deseable, puede lograrse con ayuda de las propiedades que establece el
teorema #M #, en particular de la ultima propiedad.
El siguiente método es conocido, como ”METODO DE CONDENSACIÓN”, se
basa precisamente en esta idea y consiste en lo siguiente:
1) Elegir una línea que contenga el mayor número de ceros posible.
2) Elegir un elemento no nulo de dicha línea (de preferencia un 1 ó -1) y
aplicar reiteradamente la ultima propiedad del teorema, hasta reducir a cero
todos los demás elementos de la línea.
3) Desarrollar por cofactores según dicha línea.
4) repetir los cuatro pasos anteriores hasta obtener un determinante de tercer
orden (o de segundo si se prefiere) y obtener su valor mediante la regla de
Sarrus.

220
Ejemplo:
1) Calcular por condensación el determinante de la siguiente matriz.
 1 1  3  2 3 
3 2 1 0  1

A   1 1 2 1 0
 
0 2 1 3  1
 1 2 4 0 1 

Primeramente elegimos la cuarta columna, ya que contiene 2 ceros, y


tomamos como pivote el tercer elemento de dicha columna, por tratarse de un
uno.
La operación es: 2f3+f1 y (-3) f3+f4 obteniendo:
1 −1 1 0 3
3 2 1 0 −1
1 −1 2 1 0
−3 5 −5 0 −1
1 2 4 0 1
De aquí podemos observar que desarrollando este por cofactores según la
cuarta columna resulta.
1 1 1 3 Commented [G2]: Corrección determinada.

3 2 1  1
Det A  1(1) 7
 3 5  5 1
1 2 4 1
Ahora haciendo lo siguiente
1 0 0 0
3 5  2  10
c1  c 2 ; c1  c3 ; y (3)c1  c 4 obtenemos Det A  (1)
3 2 2 8
1 3 3 2

Y desarrollando por cofactores según el primer renglón

5 −2 −10
𝐷𝑒𝑡 𝐴 = (−1)(1)(−1)2 |2 −2 8 |
3 3 −2

221
Que resolviendo por sarrus o cualquier otro método se obtiene que:
𝐷𝑒𝑡 𝐴 = −348
Como puede verse, el método de condensación ofrece en cada ciclo un gran
número de posibilidades para la selección de la línea y el elemento pivote. Una
buena selección en cada caso puede contribuir notablemente a simplificar los
cálculos correspondientes.

3.12. Propiedades de los determinantes

Las propiedades básicas del determinante son las siguientes:


1. El determinante de una matriz A y el de su transpuesta (AT) son iguales, es
decir,
|𝐴| = |𝐴𝑇 |

2. Sea A una matriz cuadrada,


Si A posee dos filas (columnas) iguales, necesariamente |𝐴| = 0.
Si A es triangular, esto es, A sólo tiene ceros por encima o por debajo de la
diagonal principal, entonces |𝐴|, es igual al producto de los elementos de la
diagonal.

3. Supongamos, que B se ha obtenido de A, mediante una operación


elemental entre filas o columnas,
Si se han intercambiado dos filas (columnas) de A, |B| = - |A|.
Si se ha sumado un múltiplo de una fila (columna) a otra, entonces |B| =|A|.
Si se ha multiplicado una fila (columna) de A por un escalar k, |B| = k| A|.

4. Sea A cualquier matriz n-cuadrada, son equivalentes los siguientes


principios:
A es invertible, es decir, A tiene inversa A-1.
AX = 0 tiene solamente la solución trivial.
El determinante de A, no es nulo: |A| ≠ 0.

222
5. El determinante es una función multiplicativa. Es decir, el determinante del
producto de matrices A y B es el producto de los determinantes:
|AB | = |A| |B|.

TEOREMA: #M#
a). Si en un determinante, dos filas (Columnas) son iguales, entonces el
valor del determinante es cero.
b). Si una fila (Columna) de un determinante es cero, el valor del
determinante es cero.
c). Si en un determinante, se intercambian dos filas (Columnas) el valor del
determinante cambia de signo.
d). En un determinante se puede multiplicar una fila (Columna) por una
constante k, entonces el valor del determinante queda multiplicado k-veces.

Demostración:
a b ka b
Si |A| = | | y k|A| = | |
c d kc d
Entonces:
|𝐴| = 𝑘|𝐴|
(ad − bc) = kad − kbc = k(ad − bc)

e). Se puede multiplicar la fila (Columna) de un determinante, por una


constante K, y el resultado de este producto, se le puede agregar a otra fila
(Columna), sin que el valor del nuevo determinante cambie.

Demostración:

|A| = |a b
| = ad − bc
c d
a b
|B| = | | = a(d + kb) − b(c + ka) = ad + kab − bc − kab
c + ka d + kb
= 𝑎𝑑 − 𝑏𝑐

223
Por lo tanto
|𝐴| = |𝐵|
Con la ayuda del teorema anterior se pueden simplificar el cálculo de
determinantes de ordenes grandes, puesto que como estas propiedades son
validas tanto para sus filas como para las columnas, entonces se puede buscar
hacer ceros ,al mayor número de elementos que se puedan para evitar los
cálculos que se generarían si no se buscan estos.

Ejemplo:
1) Hallar el determinante indicado, utilizando, las propiedades enunciadas
en el teorema anterior.

2 3 8 5
1 2 0 7

1 2 2 3
2 1 0 3

Observamos que en la tercera columna hay dos ceros, entonces solamente


generaremos un cero.
Multiplicamos por (4) a la tercera fila y se lo agregamos a la primera

 2 11 0 17
1 2 0 7

1 2 2 3
2 1 0 3

Calculando dicho determinante, por menor respecto a la columna tres, y


utilizando la propiedad de que un determinante (o su menor) multiplicado
por cero, es cero se tiene:
 2 11 0 17
 2 11 17
1 2 0 7
 2 1 2 7
1 2 2 3
2 1 3
2 1 0 3

224
A continuación utilizamos el determinante de orden tres, que resulto del
paso anterior; multiplicamos por (-2) a la fila tres y el resultado lo
agregamos a la fila dos, después multiplicamos por (-11) a la fila tres y lo
sumamos con la fila uno, de tal manera que

−2 11 17 20 0 50
20 50
2|1 2 7 | = 2 | 5 0 13| = 2(−1) | | = −2(260 − 250) = −20
5 13
−2 1 −3 2 1 3

225
3.12.1.”EJERCICIOS PROPUESTOS”

33 – 38. Calcule el determinante de la matriz, si existe.

4 5 −2 1
33.- [ ] 34.-[ ] 35.-[3]
0 −1 3 −2
1 1
3 2 8
36.-[2 5] 37.-[ ] 38.-[ 1]
0 1
2

226
3.13. Calculo de la adjunta por medio de los cofactores.

Esta es una de las aplicaciones que tienen los determinantes, la otra es la


resolución de sistemas de ecuaciones lineales.

Se conoce como adjunta de una matriz cuadrada A, a la transpuesta de la


matriz que se obtiene reemplazando los elementos de A por sus respectivos
cofactores, como lo establece la siguiente:
Definición:

Si A es cualquier matriz n x n, y Cij es el cofactor de aij , se llama adjunta de A,


a la matriz

Adj A = [bij ] donde bij = cji

Esta matriz son los cofactores de A

 C11 C12 C1n 


 
 C21 C22 C2 n 
 
 
 Cn1 Cn 2 Cnn 

La transpuesta de esta matriz, se denomina adjunta de A, y se denota por


Adj. A.

227
Ejemplo:
1 2 −1
𝑆𝑒𝑎 𝐴 = (0 −3 2 )
2 1 5

Los cofactores de los nueve elementos de A son:

−3 2 0 2 0 −3
A11 = + | | = −17 A12 = − | |=4 A13 = + | |=6
1 5 2 5 2 1
2 −1 1 −1 1 2
A21 = − | | = −11 A22 = + | |=7 A23 = − | |=3
1 5 2 5 2 1
2 −1 1 −1 1 2
A31 = + | | = 1 A32 = − | | = −2 A33 = + | | = −3
−3 2 0 2 0 −3
−17 4 6
𝐶𝑜𝑓. 𝐴 = [−11 7 3]
1 −2 −3

La transpuesta de la matriz de los cofactores anteriores proporciona la adjunta


de A:

−17 −11 1
Adj. A = (Cof. A)T = [ 4 7 −2]
6 3 −3

228
3.14. Aplicación de la adjunta, para hallar la matriz
inversa.

Si A es una matriz invertible (no singular), es decir Det. A es diferente de cero


(|𝐴| ≠ 0) Entonces

1
A−1 = adj(A)
det(A)

Donde la Ajunta de A = (Cofactores de A)T

Ejemplo:

1) Calcular, por la adjunta, la inversa de la siguiente matriz.

1 −3 2
A = (2 5 0)
0 −1 −2

Solución.
Empezaremos por hallar el det A,
1 −3 2
det 𝐴 = |2 5 0 | = −10 − 4 − 12 = −26
0 −1 −2

Los cofactores de los nueve elementos de A son:

5 0 2 0 2 5
A11 = + | | = −10 A12 = − | |=4 A13 = + | | = −2
−1 −2 0 −2 0 −1
−3 2 1 2 1 −3
A21 = − | | = −8 A22 = + | | = −2 A23 = − | |=1
−1 −2 0 −2 0 −1

−3 2 1 2 1 −3
A31 = + | | = −10 A32 = − | |=4 A33 = + | | = 11
5 0 2 0 2 5

−10 4 −2
𝐶𝑜𝑓. 𝐴 = [ −8 −2 1 ]
−10 4 11

229
La transpuesta de la matriz de los cofactores anteriores proporciona la adjunta
de A:

−10 −8 −10
Adj. A = (Cof. A)T = [ 4 −2 4 ]
−2 1 11

Aplicando la propiedad de la matriz inversa obtenemos A-1:

1 1 −10 −8 −10
𝐴−1 = (𝑎𝑑𝑗 𝐴) = ( 4 −2 4 ) = Simplificando,
|𝐴| −26
−2 1 11

5 4 5
13 13 13
2 1 2
𝐴−1 = − −
13 13 13
1 1 11
( 13 − 26 − 26)

230
“EJERCICIOS PROPUESTOS”

1.- Aplicar la regla de sarrus, para calcular el valor de los siguientes


determinantes.

3 𝑏 −𝑐
𝑎 −3𝑎
𝑎) | | 𝑏) |−2 1 −1|
2 1
9 𝑐 3𝑏

2.- Obtener el cofactor y el menor, de los elementos a22 y a23 de la matriz.

1 2 −5
𝐴=[ 4 3 6]
−2 1 2

3.- Obtener el valor del determinante

4 6 0 1
0 2 1 0
2 1 5 2
1 6 7 1
a) Desarrollando por cofactores según la tercera columna.

b) Aplicando lo que sería la regla de Sarrus, para un determinante de cuarto


orden.

Y comparar estos resultados con el obtenido por el método por


cofactores.

4.- Calcular el determinante del problema anterior, desarrollando por


cofactores, según el segundo renglón.

5.- Calcular el valor del determinante.


2 0 1 2 1
1 0 2 0 3
 4 1 3 1 5
1 2 0 4 3
2 1 4 0 1
Por el método de condensación.

Transformando a una matriz triangular.

231
a11 a12 0 0
a 21 a 22 0 0
6.- Demostrar que el valor de es independiente de los
b1 b2 a33 0
c1 c2 c3 a 44
valores de b1, b2, c1, c2, y c3

232
233
Ejercicio
1.- Para las siguientes matrices:
Realizar las operaciones que se indican

2i 1 + i 3 −1 i 2i
A= 2 i ; B = 3i i ; C= 2−i 1
−1 2 − i 2 1 − 2i −1 3i

Calcular: A+B, A-B, B-A, 2A-C y 3B-2C

2𝑖 1+𝑖 3 −1 3 + 2𝑖 𝑖
A+B= 2 𝑖 + 3𝑖 𝑖 2 + 3𝑖 2𝑖
−1 2−𝑖 2 1 − 2𝑖 1 3 − 3𝑖

2𝑖 1 + 𝑖 3 −1 2𝑖 − 3 2+𝑖
A-B= 2 𝑖 - 3𝑖 𝑖 2 − 3𝑖 0
−1 2 − 𝑖 2 1 − 2𝑖 −3 1+𝑖

3 −1 2𝑖 1+𝑖 3 − 2𝑖 −2 − 𝑖
B-A= 3𝑖 𝑖 - 2 𝑖 −2 + 3𝑖 0
2 1 − 2𝑖 −1 2−𝑖 3 −1 − 𝑖

2𝑖 1 + 𝑖 𝑖 2𝑖
2A-C= 2 2 𝑖 - 2−𝑖 1
−1 2 − 𝑖 −1 3𝑖

4𝑖 2 + 2𝑖 𝑖 2𝑖 3𝑖 2
4 2𝑖 - 2−𝑖 1 2+𝑖 −1 + 2𝑖
−2 4 − 2𝑖 −1 3𝑖 −1 4 − 5𝑖

234
3 −1 𝑖 2𝑖
3B+2C = 3 3𝑖 𝑖 + 2 2−𝑖 1
2 1 − 2𝑖 −1 3𝑖

9 −3 2𝑖 4𝑖 9 + 2𝑖 −3 + 4𝑖
= 9𝑖 3𝑖 + 4 − 2𝑖 2 4 + 7𝑖 2 + 3𝑖
6 3 − 6𝑖 −2 6𝑖 4 3

2.-Demostrar que si A, B, y C son matrices de mxn cuyos elementos son


números complejos entonces:

A + (B + C) = (A + B) + C

Si las matrices:

i 5 2 1 4 5 9i 2 1
A = (4 2i 0) ; B = (3 7 i ) ; C = ( 3 6 i )
3 −i i 6 3i 3 5i 8 −2i

i 5 2 1 4 5 9i 2 1
(4 2i 0 ) + (3 7 i) + (3 6 i ) =
3 −i i 6 3i 3 5i 8 −2i

i 5 2 1 4 5 9i 2 1
(4 2i 0 ) + (3 7 i) + (3 6 i )
3 −i i 6 3i 3 5i 8 −2i

Efectuando las operaciones tenemos:


A + (B + C) = (A + B) + C

i 5 2 1 + 9i 6 6 1+i 9 7 9i 2 1
(4 2i 0) + ( 6 13 2i ) = ( 7 7 + 2i i ) + (3 6 i )
3 −i i 6 + 5i 8 + 3i 3 − 2i 9 2i 3+i 5i 8 −2i

1 + 10i 11 8 1 + 10i 11 8
( 10 13 + 2i 2i ) = ( 10 13 + 2i 2i ) → L. Q. Q. D
9 + 5i 8 + 2i 3 − i 9 + 5i 8 + 2i 3−i

235
3.- Dadas las matrices A Y B comprobar si A=B:

−i 2 1 −i 2 1
A=[ ] y B=[ ]
3 i 0 3 0 1

No son iguales, a pesar de que son del mismo orden y tienen los mismos
elementos; ya que, aunque se cumplen las igualdades

𝑎11 = 𝑏11
𝑎12 = 𝑏12
𝑎13 = 𝑏13
𝑎21 = 𝑏21

Se tiene además que:

𝑎22 ≠ 𝑏22

a23 ≠ b23

Por lo que A y B no satisfacen la condición de igualdad.

4.- Para las siguientes matrices:


Realizar las operaciones que se indican

3 −5 1 2
−1 5 2i
A= 0 1+i B = −2 −i y C=
7+i 0 3
−2i 4 3 −4

a) A+B
b) A+C

Se tiene que:

3+1 −5 + 2 4 −3
A + B = [0 + (−2) 1 + i + (−i)] = [ −2 1]
−2i + 3 4 + (−4) 3 − 2i 0

Mientras que la adición de A y C no puede efectuarse, ya que las matrices no son


del mismo orden. Se dice por ello que A y C “no son conformables” para la adición
y, en consecuencia, la suma A+C no existe.

236
5.-Realizar el producto matricial indicado.

1 −𝑖 3 𝑖 2
Sea A= 2 −1 1 y B= −3 −1
1 𝑎 −1 1 1

Efectuar la multiplicación

1 −𝑖 3 𝑖 2 3 + 4𝑖 5+𝑖
𝐴•𝐵 == 2 −1 1 −3 −1 → ( 4 + 2𝑖 6 )
1 𝑎 −1 1 1 𝑖 − 3𝑎 − 1 1−𝑎

3 2 4 1
6.- Sea A = y B=
1 −1 −2 −1

Hallar A • B Y B • A

3 2 4 1 8 1
𝐴•𝐵 =( )( )=( )
1 −1 −2 −1 6 2
4 1 3 2 13 7
B•A=( )( )=( )
−2 −1 1 −1 −7 −3

3 1
7.- Sea A = [ ] obtener la inversa
5 2

Se tiene que la matriz

2 −1
X=[ ]
−5 3

Es tal que

2 −1 3 1 1 0
XA = [ ][ ]=[ ]
−5 3 5 2 0 1
3 1 2 −1 1 0
AX = [ ][ ]=[ ]
5 2 −5 3 0 1

237
3 5 0
8.-Sea A = 2  3  1 obtener la inversa.
1 2 0

3 5 0 a 1 2 0 c 1 2 0 c
1
A  2  3 1 b 2  3 1 b 0  7  1 b  2c
1 2 0 c 3 5 0 a 0  1 0 a  3c

1 2 0 c 1 2 0 c
0  1 0 a  3c 0 1 0  a  3c
0  7  1 b  2c 0  7  1 b  2c

1 0 0 c  2a  6c 1 0 0 2a  5c
0 1 0  a  3c 0 1 0  a  3c
0 0  1 b  7a  19c 0 0 1 7a  b  19c

2 0 5
A1  1 0 3
7  1  19

9.-Si A y B son matrices de mxn y nxp, respectivamente y 𝛼 es un número


complejo cualquiera entonces.

𝛼(𝐴𝐵) = (𝐴𝛼)𝐵 = 𝐴(𝛼𝐵)

a) Ilustrar el enunciado anterior mediante un ejemplo.


b) Demostrar dicho enunciado.

a) Supongamos que:

3 8 4 9
A=( ), B = ( ) y α = 2i
4 3 6 5

3 8 4 9 3 8 4 9 3 8 4 9
2𝑖 ( )( ) = 2𝑖 ( ) ( )=( ) 2𝑖 ( )
4 3 6 5 4 3 6 5 4 3 6 5
60 67 6𝑖 16𝑖 4 9 3 8 8𝑖 18𝑖
2𝑖 ( )=( )( )= ( )( )
34 51 8𝑖 6𝑖 6 5 4 3 12𝑖 10𝑖
120𝑖 134𝑖 120𝑖 134𝑖 120𝑖 134𝑖
( )= ( )= ( ) → 𝐋. 𝐐. 𝐐. 𝐃.
68𝑖 102𝑖 68𝑖 102𝑖 68𝑖 102𝑖
238
Demostrar que si A es una matriz de mxn con elementos en C, entonces.

4 2 − 2i 7
AIn = A Si A=( 3 1 2i )
1 − i 3 + 5i 8

Sabemos que I es la matriz de identidad.

4 2 − 2i 7 1 0 0 4 2 − 2i 7
( 3 1 2i ) (0 1 0) = ( 3 1 2i )
1−i 3 + 5i 8 0 0 1 1−i 3 + 5i 8
4 2 − 2i 7 4 2 − 2i 7
( 3 1 2i ) = ( 3 1 2i )
1−i 3 + 5i 8 1−i 3 + 5i 8

10.- Para las siguientes matrices

𝑎11 3 −2 𝑏11 0 1
𝐴 = ( −1 2 −1) ; 𝐵 = (−1 2 𝑏23 )
1 0 0 −2 3 𝑏33

Determinar los valores de a11, b11, b23 y b33 que satisfacen la igualdad.

AB= I

a11 3 −2 b11 0 1 1 0 0
(−1 2 −1) ( −1 2 b23 ) = (0 1 0)
1 0 0 −2 3 b33 0 0 1

𝑎11 𝑏11 + 1 0 𝑎11 + 3𝑏23 − 2𝑏33 1 0 0


( −𝑏11 1 2𝑏23 − 𝑏33 − 1 ) = (0 1 0)
𝑏11 0 1 0 0 1

𝑎11 • 𝑏11 + 1 = 1 𝑎11 + 3𝑏23 − 2𝑏33 = 0


𝑎11 • 𝑏11 = 0 3𝑏23 − 2𝑏33 = 0
𝑎11 = 0 − 2(2𝑏23 − 𝑏33 = 1)
𝑏11 = 0 3𝑏23 − 2𝑏33 = 0

𝑏23 = 2 −4𝑏23 + 2𝑏33 = −2

−𝑏23 = −2

239
𝑏33 = 3 2𝑏23 − 𝑏33 = 1

4 − 𝑏33 = 1 → −𝑏33 = −3

𝑏33 = 3

11.- Encuentra el producto entre las siguientes matrices.

i 4 2 5
3 −1 2
𝐴=( ) • 𝐵 = ( 1 0 1 i )
2 4 𝑖
3 2 1 1

7  3i 16 3 13  i
𝐴•𝐵 =  4  5i 8  2i  8  i 10  3i
( )

12.- hallar la matriz resultante de las siguientes matrices.𝑣 • 𝑤 𝑦 𝑤•𝑣

1 1 a  2 i 1 2 0
3 4 1 i 0 0 1 2
𝑣= ; w=
2 1 0 0 i i 1 0
( 0 1 1 0 ) ( 0 0 1 i )

i  ai  1  ai 5  a  2  2i
2i  3  i 1 i 7
𝑣•𝑤 =
2i 2 5 2
( i  i 0 2 )

i  i  6  i ai  1  3i
2 3 2 0
𝑤•𝑣 =
2  2i  1  5i ai  i  2i  1
( 2 1 i i 0 )

240
5 7 4 7 1 0
13.- si F = −3 | 9 3 3| y G = |0 1 0|
−2 −1 −9 4 2 −3

Calcular

a) F-G

−15 −21 −12 7 1 0 −22 −22 −12


|−27 −9 −9 | − |0 1 0 | = |−27 −10 −9 |
6 3 27 4 2 −3 2 1 30

b) GT + F

7 0 4
𝐺 𝑇 = |1 1 2|
0 0 −3

7 0 4 −15 −21 −12 −8 −21 −8


|1 1 2 | + |−27 −9 −9 | = |−26 −8 −7|
0 0 −3 6 3 27 6 3 24

14.- Encontrar la matriz inversa de A

3 2 1 d −b
A=( ) A−1 = ( )
−1 4 det A −c a
1 4 −2
𝐴−1 = ( )
14 1 3

Comprobación

A−1 A = I

1 4 −2 3 2
( )•( )
14 1 3 −1 4
1 14 0 1 0
( )=( )
14 0 14 0 1

241
15.- Hallar la matriz inversa de A por el método de la adjunta

1 −1 0 + − +
𝐴 = (2 −1 3 ) (− + −)
1 −1 1 + − +
1 −1 0 1 −1
Det A = (2 −1 3|2 −1) = (−1 − 3 + 0) − (0 − 3 − 2) = −4 − (−5) = 1
1 −1 1 1 −1
−1 3
𝐶𝑜𝑓 (1)𝐴 = + | |=2
−1 1
−1 0
𝐶𝑜𝑓 (2)𝐴 = − | |=1
−1 1
−1 0
𝐶𝑜𝑓 (1)𝐴 = + | | = −3
−1 3
2 3
𝐶𝑜𝑓 (−1)𝐴 = − | |=1
1 1
1 0
𝐶𝑜𝑓 (−1)𝐴 = + | |=1
1 1
1 0
𝐶𝑜𝑓 (−1)𝐴 = − | | = −3
2 3
2 −1
𝐶𝑜𝑓 (0)𝐴 = + | | = −1
1 −1
1 −1
𝐶𝑜𝑓 (3)𝐴 = − | |=0
1 −1
1 −1
𝐶𝑜𝑓 (1)𝐴 = + | |=1
2 −1
2 1 −1 2 1 −3
𝐶𝑜𝑓 𝐴 = ( 1 1 0 ) → (𝐶𝑜𝑓 𝐴)𝑇 = ( 1 1 −3)
−3 −3 1 −1 0 1

1 2 1 −3
𝐴−1 = ( 1 1 −3)
1
−1 0 1

16.- Multiplicar A X B

0 −2 1
𝐴 = (1 8 −2) 𝑦 𝐵 = |3 −4 0|
1 2 −3
𝐴 • 𝐵 = (22 −38 7)

242
17.- Obtenga la determinante de A, B, C

2 6 0
𝐴 = |4 0 −2| = (0 − 12 + 0) − (0 + 8 + 192) = −12 − (200) = −212
1 −2 8
1 −2 3
𝐵 = |2 10 −5| = (120 + 40 − 48) − (120 + 40 − 48) = 112 − 112 = 0
4 −8 12
2 5 −5
C = |5 0 10 | = (0 + 50 − 50) − (0 + 40 + 75) = (0 − 115) = −115
1 2 3

18.- Hallar la matriz inversa de A

1 3
A=| | = 5 − 6 = −1
2 5

Determinante de A= -1

a b
Sea A = ( )
c d

Teniendo

1 d −b 1 d −b
A−1 = ( )= ( )
det A −c a ad − bc −c a
1 5 −3
A−1 = ( )
−1 −2 1
AA−1 = A−1 A = I

5 −3 1 3
A−1 = −1 ( )( )
−2 1 2 5
−1 0 1 0
A−1 = −1 ( )=( )
0 −1 0 1
1 0
A−1 = [ ]=I
0 1

243
19.- Hallar la matriz inversa de B

3 4
B=| | = 6 − (−4) = 10
−1 2

Determinante de B= 10

a b
Sea A = ( )
c d

Teniendo

1 d −b 1 d −b
A−1 = ( )= ( )
det A −c a ad − bc −c a
1 2 −4
A−1 = ( )
10 1 3
AA−1 = A−1 A = I

1 2 −4 3 4
A−1 = ( )( )
10 1 3 −1 2
1 10 0
A−1 = ( )
10 0 10
1 0
A−1 = [ ]=I
0 1

20.- Hallar la matriz inversa de C

−2 −1
C=| | = −6 − (−4) = −2
4 3

Determinante de C=-2

a b
Sea A = ( )
c d

Teniendo

1 d −b 1 d −b
A−1 = ( )= ( )
det A −c a ad − bc −c a
1 3 1
A−1 = ( )
−2 −4 −2
AA−1 = A−1 A = I

244
1 3 1 −2 −1
A−1 = ( )( )
−2 −4 −2 4 3
1 −2 0
A−1 = ( )
−2 0 −2
1 0
A−1 = [ ]=I
0 1

21.- Obtener la inversa si existe de cada una de las siguientes matrices por el
método de las operaciones elementales.

1 2 3 1 1 0 0 0 1 2 3 1 1 0 0 0
1 3 3 2 0 1 0 0 0 1 0 1 1 1 0 0
A=
2 4 3 3 0 0 1 0 0 0 3 1 2 0 1 0
1 1 1 1 0 0 0 1 0 1  2 0 1 0 0 1

1 2 3 1 1 0 0 0 1 2 3 1 1 0 0 0
0 1 0 1 1 1 0 0 0 1 0 1 1 1 0 0
0 1  2 0 1 0 0 1 0 0 2 1 2 1 0 1
0 0 3 1 2 0 1 0 0 0 3 1 2 0 1 0

1 0 3 1 3 2 0 0
0 1 0 1 1 1 0 0
1 1 1
0 0 1  1  0 
2 2 2
1 3
0 0 0  1 3 1 
2 2 2

1 1 3
1 0 0 0  0
2 2 2
0 1 0 1 1 1 0 0
1 1 1
0 0 1  1  0 
2 2 2
0 0 0 1 2 3 2 3

245
1 0 0 0 1 2 1 0
0 1 0 1 1 1 0 0
1 1 1
0 0 1  1  0 
2 2 2
0 0 0 1 2 3 2 3

1 0 0 0 1 2 1 0
0 1 0 1 1 1 0 0
0 0 1 0 0 1 1 1
0 0 0 1 2 3 2 3

1 0 0 0 1 2 1 0
0 1 0 0 1 2 2 3
0 0 1 0 0 1 1 1
0 0 0 1 2 3 2 3

1 2 1 0
1 2 2 3
A 1 =
0 1 1 1
2 3 2 3

246
CAPITULO IV
SISTEMAS DE ECUACIONES LINEALES

Introducción

Uno de los problemas prácticos que con más frecuencia aparece en casi todos los
campos de estudio, tales como física, comercio, economía, sociología, etiología,
demografía, genética, todas las ramas de la ingeniería, etc., es el de resolver un
sistema de ecuaciones lineales.
Los modelos matemáticos han jugado un papel un papel muy importante en el
desarrollo de las ciencias, tanto exactas como sociales, ya que permiten describir
de una manera situaciones reales.

Ningún modelo es una descripción exacta de la realidad, sin embargo, los modelos
son útiles para hacer predicciones acerca de ella. En particular, los modelos
lineales proporcionan una buena aproximación de la realidad y tienen la ventaja de
ser fácilmente manejable y de que muy frecuentemente se puedan encontrar las
soluciones analíticas o numéricas de las cantidades de interés.
Se incluyen los elementos más importantes de la teoría para que pueda usarse
independientemente del curso, pero se hace mayor énfasis en los problemas. Es
a través de la solución de éstos que se adquiere un conocimiento más sólido.

247
4.1. Definición de ecuaciones lineales.

El álgebra lineal trata de la generalización de las ecuaciones lineales a n- variables.


Una ecuación en n - incógnitas tiene la forma:

a1 x1 + a2 x2 + ⋯ + an xn = b

donde 𝑎1 , 𝑎2 , … 𝑎𝑛 (llamados coeficientes) y b son constantes numéricas reales y


donde no todos los coeficientes 𝑎𝑖 son iguales a 0, se llama ecuación lineal en las
variables 𝑥1 , 𝑥2 , … 𝑥𝑛 . Por ejemplo, las ecuaciones

x + 5y = 32, 5x − 2y = 2 y x1 + x2 + ⋯ xn = 0

Son ecuaciones lineales. Todas las variables de una ecuación lineal son de primer
grado. La ecuación 3𝑥 2 + 2𝑦 = 5 no es una ecuación lineal en las variables X y Y,
ya que uno de los términos, 3𝑥 2 , es de grado dos.

4.2 Definición de Sistemas de Ecuaciones Lineales(SEL)

Definición.

Se llama sistema de ecuaciones lineales a un conjunto de ecuaciones de la forma:

a11 x1 + a12 x2 + ⋯ + a1n xn = b1

a21 x1 + a22 x2 + ⋯ + a2n xn = b2

⋮ ⋮ ⋮ ⋮ (1)

am1 x1 + am2 x2 + ⋯ + amn xn = bm

donde los términos x1, x2,…xn son las incógnitas y las letras a y b con subíndices
denotan constantes.

248
Una solución del sistema de ecuaciones lineales

a11 x1 + a12 x2 + ⋯ + a1n xn = b1

a21 x1 + a22 x2 + ⋯ + a2n xn = b2

⋮ ⋮ ⋮ ⋮

am1 x1 + am2 x2 + ⋯ + amn xn = bm

Es un conjunto ordenado de n valores k1, k2,…, kn

249
4.3. Clasificación de los Sistemas de ecuaciones lineales de
acuerdo a su tipo de solución como modelo matemático de
problemas

Existen muchas formas de resolver dichos sistemas, empezando por las clásicas de
reducción, sustitución e igualación que son las primeras que nos enseñan, puesto
que son muy fáciles de asimilar.

Un conjunto de n números que verifiquen todas las ecuaciones se llama solución


del sistema. Dado un sistema de ecuaciones, el objetivo principal es hallar todas
sus soluciones, es decir, hallar todos los valores de x1,..., xn que verifican todas las
ecuaciones.

Hay sistemas de ecuaciones que no admiten solución. A este tipo de sistemas les
llamamos “incompatibles”.
Si, por el contrario, un sistema de ecuaciones lineales tiene solución diremos que
es “compatible”
Los sistemas compatibles pueden tener una sola solución, en cuyo caso diremos
que son “determinados”; o más de una solución, en cuyo caso diremos que son
“indeterminados”.
Por lo tanto, los sistemas de ecuaciones lineales pueden clasificarse de la siguiente
manera.

Incompatibles ó Inconsistentes
(no tienen solución)
Sistemas de Determinados (una
Ecuaciones Lineales sola solución)
Compatibles ó
Consistentes (tienen
Indeterminados (más de
una solución)
una solución)

250
4.3.1. La Matriz Aumentada

La siguiente matriz, es una abreviación conveniente de (1).


AX = B
Donde, A, X y B representan los siguientes arreglos matriciales de números

A= a11 a12 … a1n X= X1 Y= b1


a21 a22 … a2n X2 , b2
⋮ ⋮ ⋮ , ⋮ ⋮
am1 am2 amn Xn bm

Las líneas horizontales se llama filas y las líneas verticales se llaman columnas.
Una matriz m x n tiene m filas y n columnas.

Dado un sistema de ecuaciones AX = B, es conveniente introducir la matriz


aumentada (A|B) que es la matriz de los coeficientes A, aumentada con los
elementos de B como una columna adicional.

Por ejemplo, la matriz aumentada para el sistema de ecuaciones (1) es:

a11 a12 … a1n b1


a21 a22 … a2n b2
⋮ ⋮ ⋮ ⋮
am1 am2 … amn bm

La matriz aumentada describe completamente el sistema de ecuaciones.

251
4.3.2 Operaciones Elementales de Renglones

Para resolver un sistema de ecuaciones lineales, es necesario sustituir el sistema


de ecuaciones dado, por otro nuevo, que sea más fácil de resolver:
Se pueden llevar a cabo las siguientes operaciones elementales en un sistema de
ecuaciones sin afectar su solución

1.- Multiplicar cualquier ecuación por una constante diferente de cero.


2.- Intercambiar dos cualesquiera de las ecuaciones
3.- Sumar a cualquier ecuación, otra ecuación multiplicada por una constante
diferente de cero.

Para cualquiera de estas operaciones existe una operación sobre la matriz


aumentada correspondiente del sistema.
1.- Multiplicar cualquier renglón de la matriz por un constante diferente de cero.
2.- Intercambiar dos renglones cualesquiera de la matriz
3.- Sumar a cualquier renglón de la matriz otro, que se encuentre multiplicada por
una constante diferente de cero.

Estas son las llamadas operaciones elementales de renglones en la matriz


aumentada.

252
4.3.3. Forma Escalonada y Forma Escalonada Reducida de una
Matriz

Una matriz está en forma escalonada, si satisface las siguientes condiciones.

1.- El primer número no nulo (avanzando de izquierda a derecha) de cada fila es


un uno, al cual se le llama uno principal.
2.- Cada uno principal está a la derecha de los unos principales de las filas
anteriores.
3.- Todas las filas nulas, si existen, están en la parte inferior de la matriz.
Una matriz se encuentra en forma escalonada reducida si es escalonada (puntos 1,
2 y 3) y además satisface la siguiente condición.
4.- Todo número arriba y abajo de cada uno principal es cero.

La técnica de emplear operaciones elementales sobre renglones para llegar a una


matriz en forma escalonada se llama eliminación de Gauss. El proceso de llegar a
una matriz en forma escalonada reducida se llama eliminación de Gauss-Jordán.

Las matrices siguientes están en forma escalonada

1 5 4 3 1 3 6 10 0 
1 1 3    
  0 1 2 1 0 0 1 4 3 
0 1 2 0 0 1 2 0 0 0 1 1/ 2 
0 0 1
     
0 0 0 0 0 0 0 0 0 

Nota: Se puede observar, que a partir de la segunda fila, el número de ceros hacia
abajo, es mayor que la de fila anterior.

Las siguientes matrices están en forma escalonada reducida

1 0 0 4
  1 0 1 1 0 1 3 
0 1 0 2 ( )  
0 0 1 1 0 1 0 0 1 0 2
 

253
4.4. Solución de ecuaciones lineales por los diferentes
métodos.

En general, para resolver un sistema de ecuaciones con un sistema matricial, se


emplean las operaciones elementales sobre renglones de la matriz, para llegar a
una matriz de forma escalonada o escalonada reducida, las cuales se describen a
continuación.

4.4.1. Método por Eliminación de Gauss

Ejemplo 1. Solución de un sistema usando la forma escalonada


Resolver el siguiente sistema de ecuaciones lineales usando la eliminación de
Gauss.

4𝑥 + 8𝑦 − 4𝑧 = 4
3𝑥 + 6𝑦 + 5𝑧 = −13
−2𝑥 + 𝑦 + 12𝑧 = −17

Primero se escribe el sistema en forma matricial AX=B.

4 8 −4 𝑥 4
[ 3 6 5 ] [𝑦] = [−13]
−2 1 12 𝑧 −17

Ahora escribimos la matriz extendida.


4 8 −4 4
( 3 6 5 |−13)
−2 1 12 −17
Ahora se realizan las operaciones elementales por renglones.
3 F1  F2
 4 8 4 4     1 2 1 1    1 2 1 1
1/ 4F1
2 F1  R3
 3 6 5 13  3 6 5 13 0 0 8 16 
     
 2 1 12 17   2 1 12 17  0 5 10 15
254
 1 2 1 1  1 2 1 1
F2  F3
 0 5 10 15 1/ 5 F  
  
1/ 8 F
 0 1 2 3
2
3

0 0 8 16  0 0 1 2 

Ahora hemos obtenido una matriz equivalente en forma escalonada, cuyo sistema
correspondiente de ecuaciones es:
𝑥 + 2𝑦 − 𝑧 = 1
𝑦 + 2𝑧 = −3
𝑧 = −2

Para resolver el sistema, aplicamos la sustitución hacia atrás.


Es decir, como de la última ecuación Z=-2, entonces, tomando la
Segunda ecuación y sustituyendo a “z” tendremos:

𝑦 + 2(−2) = −3
𝑦=1

Y de la tercera, sustituyendo los valores de “z” e ”y” se obtiene

x + 2(1) − (−2) = 1
𝑥 = −3

Por lo tanto, la solución del sistema es (-3, 1, 2).

255
4.4.2. Método por Eliminación de Gauss-Jordán

La ventaja de usar la forma escalonada reducida es que no se necesita la


sustitución hacia atrás para resolver el sistema, como se vera en el siguiente
ejemplo.
Ejemplo 1.
Solución de un sistema usando la forma escalonada reducida
Resolver el siguiente sistema de ecuaciones lineales usando la eliminación de
Gauss-Jordán.

4𝑥 + 8𝑦 − 4𝑧 = 4
3𝑥 + 6𝑦 + 5𝑧 = −13
−2𝑥 + 𝑦 + 12𝑧 = −17

A partir de la matriz escalonada que se obtuvo del ejemplo anterior, ahora se


continuará usando operaciones elementales sobre renglones para dicha matriz y
así obtener su forma escalonada reducida.

 1 2 1 1  1 2 0 1  1 0 0 3
0 1 2 3 
2F3  F2
 0 1 0 1
2F2  F1
  0 1 01
  F3  F1 
0 0 1 2  0 0 1 2  0 0 1 2 

Se ha llegado a una matriz escalonada reducida, cuyo solución es única, es decir


que:
𝑥 = −3
𝑦=1
𝑧 = −2

Así, se llega a la misma solución del ejemplo anterior, la cual es (-3, 1, -2).

256
Ejemplo 3. Sistema sin solución

Resuelva el siguiente sistema


𝑥 − 3𝑥 + 2𝑧 = 12
2𝑥 − 5𝑦 + 5𝑧 = 14
𝑥 − 2𝑦 + 3𝑧 = 20

Transformamos el sistema a la forma escalonada

2F1  F2
 1 3 2 12  
 F1  F2
  1 3 2 12   1 3 2 12 
 2 5 5 14  0  1 1 10 
1 1 10 
 F2  F3
 0
  
 1 2 3 20  0 1 1 8 0 0 0 18

 1 3 2 12 
0 1 1 10 
 

1/18F3
 0 0 0 1

La ultima matriz esta en forma escalonada. Si se escribe el último renglón como


una ecuación, obtendremos 0𝑥 + 0𝑦 + 0𝑧 = 1, es decir, 0=1, lo cual hace que esta
ecuación no se pueda resolver. Esto quiere decir que el sistema no tiene solución.

Ejemplo 4. Sistema con una cantidad infinita de soluciones


Determine la solución completa del siguiente sistema
−3𝑥 − 5𝑦 + 36𝑧 = 10
−𝑥 + 7𝑧 = 5
𝑥 + 𝑦 − 10𝑧 = −4
Se transforma el sistema su forma escalonada

 3 5 36 10  
F1  F3
  1 1 10 4  
F1  F2
3 F1  F3
  1 1 10 4 
 1 0   1 0  0
 7 5   7 5  1 3 1
 1 1 10 4   3 5 36 10  0 2 6 2 

257
 1 1 10 4 
 1
 0 1 3
2F2  F3

0 0 0 0 

Esta matriz corresponde al sistema:

𝑥 + 𝑦 − 10𝑧 = −4
𝑦 − 3𝑧 = 1

Despejamos las variables iníciales x e y en términos de la variable z:

𝑥 = −𝑦 + 10𝑧 − 4 → 𝑥 = −(3𝑧 + 1) + 10𝑧 − 4 → 𝑥 = 7𝑧 − 5


𝑦 = 3𝑧 + 1

Por lo tanto:

𝑥 = 7𝑧 − 5
𝑦 = 3𝑧 + 1

Para obtener la solución completa, sea z cualquier número real:


𝑥 = 7𝑧 − 5
𝑦 = 3𝑧 + 1

Z= cualquier número real.

Para obtener soluciones específicas, se asigna un valor a Z.


Por ejemplo, si z = 1, entonces:

𝑥 = 7(1) − 5 = 2
𝑦 = 3(1) + 1 = 4

Según lo anterior, una solución del sistema es (2, 4, 1). Se obtiene una solución
distinta si z es igual a 2 y así sucesivamente.
Hay una infinidad de opciones para el valor de z, por lo que hay infinitas soluciones
para el sistema dado.

258
En resumen se puede decir lo siguiente acerca de la resolución de un sistema en
forma matricial:
1.- Si en el último renglón, se presenta una ecuación de la forma
0x1 + 0x2 + ⋯ + 0xn = b, con 𝑏 ≠ 0 el sistema es incompatible.
2.- Si la forma escalonada contiene un renglón que representa la ecuación
0𝑥1 + 0𝑥2 + ⋯ + 0𝑥𝑛 = 0 (Llamada también ecuación nula) el sistema es
compatible indeterminado (tiene muchas soluciones). En este caso, si el número
de filas no nulas r, es menor que el número de variables n; esto es
r < n, entonces hay infinidad de soluciones, y si r = n existe solución única.

259
4.4.3. Método de Cramer.

Este método es uno de los más sencillos, el problema es que solamente es


válido para sistemas cuadrados, es decir sistemas que tengan el mismo
número de ecuaciones que de incógnitas.

Otro “inconveniente” es que a medida que el número de ecuaciones y por lo


tanto de incógnitas aumenta, la solución de los determinantes se complica.
Pero hay que hacer notar que utilizando las propiedades de estos vistas en el
capitulo anterior este problema se minimiza.

Una posible ventaja es que, al calcular el determinante del sistema, si este


resulta ser cero el sistema no tiene solución.

A continuación se enuncia el teorema.

TEOREMA:

Si un sistema de n ecuaciones lineales con n variables 𝑥1 , 𝑥2 … 𝑥𝑛 , es


equivalente a la ecuación matricial Dx = B, y si |D| ≠ 0, sus soluciones son

|Dx1 | |Dx2 | |Dxn |


x1 = |D|
, x2 = |D|
, … , xn = |D|
, donde Dx , es la matriz que se obtiene
reemplazando la i-ésima columna de D por la matriz B, de n x 1.

Los pasos a seguir para calcular los sistemas de ecuaciones según la regla de
Cramer son los siguientes:

1. Calcular el determinante de A.
2. Aplicar la regla de Cramer, que consiste en:
a) ir sustituyendo la primera columna del det (A) por los términos
independientes;
b) dividir el resultado de este determinante entre el det (A) para hallar el valor
de la primera incógnita;
c) continuar sustituyendo los términos independientes en las distintas
columnas para hallar el resto de las incógnitas.

260
Ejemplo 1:
Use la regla de Cramer para resolver el siguiente sistema de ecuaciones.
2𝑥 − 3𝑦 + 4𝑧 = 1
𝑥 + 6𝑧 = 0
3𝑥 − 2𝑦 =5

Solución:
Primero evaluaremos los determinantes que aparecen en la regla de Cramer.

2 −3 4 1 −3 4 2 1 4
|𝐷| = |1 0 6| = −38 |𝐷𝑥 | = |0 0 6| = −78 |𝐷𝑦 | = |1 0 6| = −22
3 −2 0 5 −2 0 3 5 0

2 −3 1
|𝐷𝑧 | = |1 0 0| = 13
3 −2 5

Ahora aplicaremos la regla de Cramer para llegar a la solución:

|Dx | −78 39
x= = =
|D| −38 19

|Dy | −22 11
y= = =
|D| −38 19

|Dz | 13 13
z= = =−
|D| −38 38

La regla de Cramer es un método eficiente para resolver este tipo de sistemas


de ecuaciones lineales. Pero en sistemas con más de tres ecuaciones, la
evaluación de los diversos determinantes que intervienen suele ser una tarea
larga y tediosa. Además, la regla no se aplica si |D| = 0
o, si D no es una matriz cuadrada.

261
4.4.4. Método por cálculo de la inversa

En este método, se debe de escribir el sistema en forma matricial, al igual que


se hace por los métodos de Gauss y Gauss-Jordán, nada más que este método
solamente es posible realizarlo si el sistema es cuadrado de cualquier orden.

Lo primero que se tiene que hacer, es calcular la inversa del sistema por
cualquiera de los métodos planteados en el capitulo anterior

Es decir hay que resolver la ecuación matricial

AX = B

De esta ecuación despejamos a X según las reglas matriciales, obteniéndose

X = A−1 B

Todo lo anterior se puede entender con un ejemplo como el siguiente

Ejemplo:

Resolver el siguiente sistema de ecuaciones lineales utilizando el método de la


inversa

𝑥 − 2𝑦 − 4𝑧 = 1
2𝑥 − 3𝑦 − 6𝑧 = −1
−3𝑥 + 6𝑦 + 15 = 2

Escribiendo el sistema en forma matricial tenemos.

A X = B

1 −2 −4 𝑥 1
[ 2 −3 −6] [𝑦] = [−1]
−3 6 15 𝑧 2

Donde

1 −2 −4
A=[ 2 −3 −6]
−3 6 15

262
Calculamos la inversa de A por uno de los métodos conocidos

Solución: Primero se comienza con la matriz extendida de 3 x 6 cuya mitad


izquierda es A y la mitad derecha es la matriz identidad.

 1 2 4 1 0 0 
 
 2 3 6 0 1 0 
 3 6 15 0 0 1

A continuación, se transforma la mitad izquierda de esta nueva matriz, en la matriz


identidad, con la siguiente secuencia de operaciones elementales sobre renglones
en toda la nueva matriz.
 1 2 4 1 0 0   1 2 4 1 0 0 
  2 F  F   1/ 3 F
 2 3 6 0 1 0 
 3F  F 10
1 2
3
1 2 2 1 0  
2F F 3
3
2

 3 6 15 0 0 1 0 0 3 3 0 1

 1 2 4 1 0 0  1 0 0 3 2 0
  4 F3  F1  
0 1 0 4 1 2 / 3 
2 F2  F1
 0 1 0 4 1 2 / 3
0 0 1 1 0 1/ 3 0 0 1 1 0 1/ 3

Ya se ha transformado la mitad izquierda, en la matriz identidad, esto quiere decir


que se ha puesto la matriz en forma escalonada reducida, y con esto hemos
obtenido en la mitad derecha A−1 .

−3 2 0
2
−4 1 −
A−1 = 3
1
[1 0
3 ]

Como ya hallamos la inversa, nada más nos falta multiplicarla por la matriz B tal
como se ve en el procedimiento siguiente:

−3 2 0 −5
2 1 19
−4 1 − −
X = A−1 B = 3 [−1] = 3
1 2 5
[1 0
3 ] [ 3 ]

263
De tal manera que

−5
𝑥 19

[𝑦] = 3
𝑧 5
[ 3 ]
Así la solución del sistema es (-5, -19/3, 5/3)

4.5. Sistemas de Ecuaciones Homogéneas

Se dice que un sistema de ecuaciones lineales de homogéneo si las constantes del


lado derecho son todas cero. Es decir, el sistema general homogéneo está dado
por
𝑎11 𝑥1 + 𝑎12 𝑥2 + ⋯ + 𝑎1𝑛 𝑥𝑛 = 0

𝑎21 𝑥1 + 𝑎22 𝑥2 + ⋯ + 𝑎2𝑛 𝑥𝑛 = 0

⋮ ⋮ ⋮
am1 x1 + am2 x2 + ⋯ + amn xn = 0

Todo sistema de ecuaciones lineales homogéneo es consistente, ya que una


solución de todos estos sistemas es 𝑥1 = 0, 𝑥2 = 0 … 𝑥𝑛 = 0. Esta solución se
denomina solución trivial; en caso de que haya otras soluciones, se denominan
soluciones no triviales.

Debido a que un sistema lineal homogéneo siempre tiene la solución trivial,


entonces para sus soluciones sólo hay dos posibilidades:
1. El sistema sólo tiene la solución trivial o solución cero.

2. El sistema tiene infinidad de soluciones además de la solución trivial.

264
Ejemplo 1: Resuelva el siguiente sistema
𝑥1 + 𝑥2 − 𝑥3 = 0
4𝑥1 − 2𝑥2 + 7𝑥3 = 0

 1 1 1 0  
4F1  F2
  1 1 1 0  1/ 6F2 1 1 1 0 
 4 2 7 0  0 6 11 0   0 1 11 0
     
6 
Solución:

x1 + x2 − x3 = 0

11 11
𝑥2 − 𝑥3 = 0 → 𝑥2 = 𝑥
6 6 3
11 5
𝑥1 = −𝑥2 + 𝑥3 → 𝑥1 − 𝑥3 + 𝑥3 → 𝑥1 = −
6 6
Por lo tanto:
5
𝑥1 = − 𝑥3
6
11
𝑥2 = 𝑥
6 3
𝑥3 = 𝑧

Donde z es cualquier número real.

Un sistema homogéneo con más incógnitas que ecuaciones tiene un número


infinito de soluciones.

265
4.6. Aplicaciones a la ingeniería, de los Sistemas de
Ecuaciones Lineales

Aplicación 1. Ingeniería Civil.

Ejemplo

Análisis de precios de materiales para construcción.

Suponga que se quiere comparar el costo de ciertos materiales. La siguiente tabla,


que puede ser vista como una matriz, da el costo en pesos de cada uno de los
materiales en tres bancos distintos.

Arena Grava Cemento Cal Arcilla


Banco 1 70 40 13 30 330
Banco 2 85 38 10 28 310
Banco 3 75 42 12 30 325

Si se compran 5 pesos de arena, 3 de grava, 13 de cemento, 4 de cal y 2 de


arcilla, podemos representar las cantidades compradas por la matriz

 5
 3
 
B  10 
 
 4
 2 

El costo total está dado por el producto

 5
70 40 13 30 330   3 1380 
 
AB   85 38 10 28 310  10   1371
 
 75 42 12 30 325  4  1391
 2 
Vemos que el costo total en el banco es 20 pesos más bajo que en el banco 1 y
20 pesos menor que en el banco 3.
Esta forma de representar el problema por medio de matrices, brinda una forma
conveniente y resumida de enunciar y resolver el problema.

266
Aplicación 2. Ingeniería en producción.

Una compañía manufactura tres productos. Sus costos de producción se dividen en


tres categorías. En cada categoría, se da un estimado del costo de producir una
unidad de cada producto. También se hace un estimado del número de unidades
que se fabricarán de cada producto por trimestre. Estos estimados se presentan en
las tablas 1 y 2. La compañía desea presentar en su junta de accionistas una tabla
que muestre los costos totales de cada trimestre en cada una de las tres
categorías: materias primas, mano de obra y costos generales.

Tabla 1. Costos de producción por unidad (dólares)

PRODUCTO
Costos A B C
Materias primas 0.10 0.30 0.15
Mano de obra 0.30 0.40 0.25

Generales y 0.10 0.20 0.15


mixtos

Tabla 2. Número de unidades producidas por trimestre.


Temporada
Producto Verano Otoño Invierno Primavera
A 4000 4500 4500 4000
B 2000 2600 2400 2200

C 5800 6200 6000 6000

267
Solución. Consideraremos el problema en términos de matrices. Cada tabla puede
representarse por una matriz.

0.10 0.30 0.15  4000 4500 4500 4000 


M  0.30 0.40 0.25 y P   2000 2600 2400 2200 
 
0.10 0.20 0.15  5800 6200 6000 6000 

Si formamos el producto MP, la primera columna de MP representará los costos del


trimestre de verano.
Materias primas:(0.10)(4000) + (0.30)(2000) + (0.15)(5800) = 1870
Mano de obra: (0.30)(4000) + (0.40)(2000) + (0.25)(5800) = 3450
Costos generales: (0.10)(4000) + (0.20)(2000) + (0.15)(5800) = 1670
y mixtos:
Los costos del trimestre de otoño se presentan en la segunda columna de MP.
Materias primas: (0.10)(4500) + (0.30)(2600) + (0.15)(6200) = 2160
Mano de obra: (0.30)(4500) + (0.40)(2600) + (0.25)(6200) = 3940
Costos generales: (0.10)(4500) + (0.20)(2600) + (0.15)(6200) = 1900
y mixtos:
Las columnas 3 y 4 de MP representan los costos de los trimestres de invierno y
primavera,
1870 2160 2070 1960 
MP  3450 3490 3810 3580 
1670 1900 1830 1740 

Los registros del renglón 1 de MP representan el costo total de las materias primas
para cada uno de los cuatro trimestres. Los registros de los renglones 2 y 3
representan el costo total de mano de obra y general, respectivamente, en cada
uno de los cuatro trimestres.
Los gastos anuales en cada categoría se pueden determinar si se suman los
registros de cada renglón. Los números de cada una de las columnas se pueden
sumar para obtener los costos de producción totales de cada trimestre. La tabla 3
representa un resumen de los costos de producción totales.

268
Temporada

Verano Otoño Invierno Primavera Año

Materias Primas 1870 2160 2070 1960 8060


Mano de obra 3450 3940 3810 3580 14780
Generales y mixtos 1670 1900 1830 1740 7140
Costos totales de 6990 8000 7710 7280 29980
producción

Aplicación 3: Ingeniería Civil

Cálculo de las fuerzas que actúan sobre los elementos de una armadura.
1.- El criterio que rige el análisis de una armadura estable y determinada
estáticamente es el equilibrio de las cargas aplicadas, de las fuerzas que actúan
sobre las barras y de las reacciones de los apoyos, así como de cualquier porción
de la estructura que se aísle como cuerpo libre.
2.- El equilibrio de las fuerzas concurrentes en un nudo se establece, si se
satisfacen las condiciones ∑ 𝐹𝑥 = 0 𝑦 ∑ 𝐹𝑦 = 0. Las reacciones y las fuerzas que
actúan sobre las barras se calculan mediante el sistema de ecuaciones que se
obtienen de aplicar las ecuaciones anteriores a cada uno de los nudos de la
estructura.
3.-Antes de utilizar las dos ecuaciones de equilibrio en un nudo, debe asignarse
una dirección a cada fuerza desconocida que actúan en él. Por tanto, se define el
carácter de los esfuerzos que transmiten dicha fuerza; el carácter supuesto, es
decir, tensión o compresión, debe ser el mismo en los dos nudos que definen al
miembro.
Ejemplo:
Calcule las reacciones y las fuerzas que inducen en los miembros de la estructura
de la Fig. 1(a) las cargas que actúan sobre ella.

269
En la Fig. 1 (b) se identifican las fuerzas que actúan sobre las barras y las
reacciones de una armadura. Se supuso que todos los miembros de la estructura
están sometidos a tensión; las fuerzas de la Fig. 1 (b) representan la acción de los
elementos sobre los nudos. En la Fig. 1 (c) se define el diagrama de cuerpo libre
del nudo a de la estructura. Las fuerzas externas se representan en términos de
sus componentes x y y, las fuerzas que actúan sobre las barras se representan
en tal forma que indican la acción de los miembros sobre los nudos.

d
4k

3 5 12´

a b c
1 2

2k

9´ 9´
Fig. 1 (a). Sistema Estructural

y d
4k

P3 P5
P4

R1 P1 P2 x
a b c

R2 2k R3
Fig. 1(b). Identificación de las reacciones y de las fuerzas internas.

270
y

5
4 4
P3 3 P3
5
R1 P1 R2
a
R1 x
a P1
R2

Fig. 1(c). Diagrama de cuerpo libre del nudo (a)

Al aplicar las condiciones ∑ 𝐹𝑥 = 0 y ∑ 𝐹𝑦 = 0 a cada nudo se obtiene el sistema


de ecuaciones siguiente:

Nudo a [Fig. 1 (c)]


3
∑ 𝐹𝑥 = 0 𝑅1 + 𝑃1 + 5 𝑃3 = 0
4
∑ 𝐹𝑦 = 0 𝑅2 + 5 𝑃3 = 0

Nudo b

∑ 𝐹𝑥 = 0 𝑃1 − 𝑃2 = 0
∑ 𝐹𝑦 = 0 𝑃4 − 2 = 0

Nudo c

3
∑ 𝐹𝑥 = 0 𝑃2 + 5 𝑃5 = 0
4
∑ 𝐹𝑦 = 0 𝑅3 + 5 𝑃5 = 0

Nudo d
3 3
∑ 𝐹𝑥 = 0 4 + 5 𝑃3 − 5 𝑃5 = 0

4 4
∑ 𝐹𝑦 = 0 𝑃 + 5 𝑃5 + 𝑃4 = 0
5 3

Las ecuaciones anteriores se expresan en forma matricial como sigue:

271
1 0 0 1 0 3/ 5 0 0   R1   0 
0
 1 0 0 0 4/5 0 0   R2   0 
0 0 1 0 0 0 0 4 / 5  R3   0 
     
0 0 0 1 1 0 0 0   P1   0 
 
0 0 0 0 1 0 0 3 / 5  P2   0 
     
0 0 0 0 0 3/ 5 0 3 / 5  P3   4 
0 0 0 0 0 0 1 0   P4   2 
     
0 0 0 0 0 4/5 1 4 / 5  P5   0 

Al escribir las ecuaciones en forma matricial se cambió su orden para que la


diagonal de la matriz de coeficientes no contenga elementos nulos.
3
R1 + P1 + P =0
5 3
4
R 2 + P3 = 0
5
4
R 3 + P5 = 0
5
P1 − P2 = 0
3
P2 + P5 = 0
5
3 3
P − P = −4
5 3 5 5
P4 = 2
4 4
P + P4 + P5 = 0
5 3 5

Ahora se resolverá el sistema por el método de Gauss-Jordán.

1 0 0 1 0 3/ 5 0 0 0
0 0 0 
 1 0 0 0 4/5 0 1 0 0 0 0 3/ 5 0 3 / 5 0 
0 0 1 0 0 0 0 4 / 5 0 0 1 0 0 0 4/5 0 0 0 
  
0 0 0 1 1 0 0 0 0  F4  F1
  0 0 1 0 0 0 0 4 / 5 0
 F5  F1   F5  F4
0 0 0 0 1 0 0 3 / 5 0 1 1 
  0 0 0 0 0 0 0 1
F6
0 0 0 0 0 3/ 5 0 3 / 5 4  0 0 0 0 1 0 0 3 / 5 0 3/ 5
0 0 0 0 0 0 1 0 2  
  0 0 0 0 0 3/ 5 0 3 / 5 4 
0 0 0 0 0 4/5 1 4 / 5 0  0 0 0 0 0 0 1 0 2
 
0 0 0 0 0 4/5 1 4 / 5 0 

272
1 0 0 0 0 3/ 5 0 3/ 5 0 1 0 0 0 0 0 0 0 4
0 0  0
 1 0 0 0 4/5 0 0
 1 0 0 0 0 0 4 / 5 16 / 3
0 0 1 0 0 0 0 4/5 0 0 0 1 0 0 0 0 4/5 0
   
0 0 0 1 0 0 0 3/ 5 0
0 0 0 1 0 0 0 3/ 5 0
0 0 0 0 1 0 0 3/ 5 0 0 0 0 0 1 0 0 3/ 5 0
  3/ 5 F  F  
0 0 0 0 0 1 0 1 20 / 3  6 1
 1 20 / 3
4/ 5 F6  F2 0 0 0 0 0 1 0
0 2 4/ 5F6  F8

0
0
0
0
0
0
0
0 0 1
0 4/5 1 4/5
0

0 
0

0 0 0 0 0 1 0 2
 
 F7  F8

0 0 0 0 0 4/5 1 4/5 0 

1 0 0 0 0 0 0 0 4 1 0 0 0 0 0 0 0 4
0 4 / 5 16 / 3 0
 1 0 0 0 0 0
 1 0 0 0 0 0 4 / 5 16 / 3
0 0 1 0 0 0 0 4/5 0 0 0 1 0 0 0 0 4/5 0
   
0 0 0 1 0 0 0 3/5 0
0 0 0 1 0 0 0 3/ 5 0
0 0 0

0 0 0 1 0 0 3/5
 0 0 0 1 0 0 3/ 5 0
0 0 0 0 0 1 0 1 20 / 3  
0 0 0 0 0 1 0 1 20 / 3
0 2

0 0 0 0 0 1 0

1
F8 0 0 0 0 0 0 1 0 2 4 / 5 F8  F2
0 0 0 0 0 0 0 8 / 5 10 / 3 
(8/5)    
4 / 5 F8  F3
0 0 0 0 0 0 0 1 25 /12 
1 0 0 0 0 0 0 0 4
1 0 0 0 0 0 0 0 4
0 11/ 3
 1 0 0 0 0 0 0 0 1 0 0 0 0 0 0 3.67 
0 0 1 0 0 0 0 0 5 / 3 
  0 0 1 0 0 0 0 0 1.67 
0 0 0 1 0 0 0 3/ 5 0  
0 0 0 1 0 0 0 0 1.25
0 0 0 0 1 0 0 3/ 5 0
  0 0 0 0 1 0 0 0 1.25
0 0 0 0 0 1 0 1 20 / 3  
0 0 0 0 0 0 1 0 0 4.58
2 3/ 5 F8  F4

0 0 0 0 0 1 0
 
3/ 5 F8  F5
 0 0 0 0 0 0 1 0 2
0 0 0 0 0 0 0 1 25 /12   
F8  F6 0 2.08
  0 0 0 0 0 0 1
Por lo tanto el resultado se expresa como:

d
 R1  4.00k   4.0k   4k
 R   3.67 k   
 2    3.7k  
 R3   1.67 k   1.7k  
     
P
 1   1.25k  1.2 k Compresión
 4.6 k C 2.1 k T
 P2   1.25k   1.2k Compresión 
     
 P3   4.58k   4.6k Compresión  2.0 k T
 P   2.00k   2.0k Tensión 
 4    
 P5   2.08k   2.1k Tensión 
4.0 k
a 1.2 k C b 1.2 k C c

3.7 k 2k 1.7 k

273
Aplicación 4. Ingeniería Civil

Ejemplo:
Un ingeniero civil involucrado en la construcción, requiere 4 800, 5 810 y 5 690 m 3
de arena, grava y cemento, respectivamente, para un proyecto de construcción. La
composición de estos bancos es:

Banco 1 Banco 2 Banco 3


Arena % 52 20 25
Grava % 30 50 20

Cemento % 18 30 55

¿Cuántos metros cúbicos se debe transportar desde cada banco para cumplir con
las necesidades del ingeniero?
Sean X, y Y, z la cantidad de metros cúbicos del banco 1, 2 y 3 que se
transportarán de cada material. Esto quiere decir que se tendrán que extraer 0.52
x m3 de arena del primer banco, 0.20 y m3 de arena del banco 2 y 0.25 z m3 de
arena del banco 3. Por lo tanto se tendrá un total de 0.52𝑥 + 0.20𝑦 + 0.25𝑧 arena
en total, Como las necesidades de arena son 4800 m3 se obtiene la primera
ecuación de las presentadas abajo. De igual forma se hace para las cantidades de
grava y cemento.
0.52𝑥 + 0.20𝑦 + 0.25𝑧 = 4800
0.30𝑥 + 0.50𝑦 + 0.2𝑧 = 5810
0.18𝑥 + 0.30𝑦 + 0.55𝑧 = 5690
Ahora se resolverá el sistema por el método de Gauss.
0.52 0.20 0.25 4800   1 0.3846 0.4807 9230.7 
0.30 0.50 0.2 5810  1/ 0.52F1
 0.30 5810  
0.3 F  F
   0.50 0.20 1
0.18 F  F
1
2
3

 0.18 0.30 0.55 5690  
 0.18 0.30 0.55 5690 

274
 1 0.3846 0.4807 9230.7   1 0.3846 0.4807 9230.7 
0 0.3846 0.0557 3040.7   1 0.145 7906.30 
   0
1/ 0.3846 F
 
1/ 0.4299F3

2
0.2307 F  F
2 3

0 0.2307 0.4634 4028.4  0 0 0.4299 2204.40 

 1 0.3846 0.4807 9230.7 


0 1 0.145 7906.30 

0 0 1 5127.70 

Por lo tanto al sustituir y despejar los valores se obtiene:


z = 5127.70 m3 de material del banco 3
y = 7162.78 m3 de material del banco 2
x = 4011.00 m3 de material del banco 1

275
Aplicación 5. Ingeniería Química/Petrolera.

Ejemplo:
Empleo de la conservación de la masa para determinar concentraciones en estado
estable de un sistema de reactores acoplados.
El balance de masa representa un ejercicio contable para la sustancia particular
que habrá de modelarse. Esto se expresa como:
Acumulación = entradas – salidas Ec. (1.5.3)
Si las entradas son mayores que las salidas, la masa de la sustancia dentro del
volumen aumenta. Si las salidas son mayores que las entradas, la masa disminuye.
Si las entradas son iguales a las salidas, la acumulación es cero y la masa
permanece constante. Para esta condición estable, o en estado uniforme, la
ecuación (1.5.3) se puede expresar como:
Entradas = Salidas
Se toma el producto del caudal Q en metros cúbicos por minuto y la concentración
c en miligramos por metro cúbico. Para cada tubería.
En la Fig. 3 se muestran cinco reactores interconectados o acoplados, por lo tanto,
se requieren también cinco ecuaciones de balance de masa para caracterizar el
sistema.

Q55 = 2
Q15 = 3
c5
Q54 = 2

Q25 = 1

Q01 = 5 Q12 = 3 Q24 = 1 Q44 = 11


c1 c2 c4

c01 = 10
Q23 = 1
Q34 = 8
Q31 = 1
Q03 = 8 c3
c03 = 20

Fig. 3
276
Para el reactor 1, la razón de flujo de masa que entra es
Q 01 C01 + Q 31 C3
y la razón de flujo de masa de salida es
Q12 C1 + Q15 C1
Ya que el sistema se halla en estado uniforme, los flujos de entrada y salida deben
ser iguales:
𝑄01 𝐶01 + 𝑄31 𝐶3 = 𝑄12 𝐶1 + 𝑄15 𝐶1
Al sustituir y despejar se obtiene:
5(10) + 𝐶3 = 3𝐶1 + 3𝐶1
𝟔𝑪𝟏 − 𝑪𝟑 = 𝟓𝟎
Se procede de la misma manera con los reactores 2, 3, 4 y 5 y se obtienen las
ecuaciones siguientes:
6C1 − C3 = 50 Reactor 1
−3C1 + 3C2 = 0 Reactor 2
−C2 + 9C3 = 160 Reactor 3
−C2 − 8C3 + 11C4 − 2C5 = 0 Reactor 4
−3C1 − C2 + 4C5 = 0 Reactor 5

Al expresarlo en forma matricial y resolverlo por el método de Gauss – Jordan se


obtiene:
 6 0 1 0 0 50  1/ 6 F1
  1 0 0.166 0 0 8.33 3 F1  F2
 
 3 3 0 0 0 0   3 3 0 0 0 0 
3 F1  F3
  
 0 1 9 0 0 160   0 1 9 0 0 160 
   
 0 1 8 11 2 0  0 1 8 11 2 0
 3 1 0 0 4 0   3 1 0 0 4 0 

 1 0 0.166 0 0 8.33 1 0 0.166 0 0 8.33


0 3 0.5 0 0 25 0 1 0.166 0 0 8.33
   1/8.83 F3

0 1 9 0 0 160   1/3 F2
F2  F3 , F2  F4 , F2  F5 0 0 8.83 0 0 168.33
0.166 F3  F2 ,0.166 F3  F1

    8.167 F3  F4
 
 0 1 8 11 2 0 0 0 8.167 11 2 8.33 0.667 F3  F5

0 1 0.5 0 4 25 0 0 0.667 0 4 33.33

277
1 0 0 0 0 11.5 1 0 0 0 0 11.5
0 1 0 0 0 11.5 0 1 0 0 0 11.5
 
0 0 1 0 0 19.06   1/11F4
1/ 4 F ,0.181F  F
 0 0 1 0 0 19.06 
  5 5 4
 
0 0 0 11 2 164.00  0 0 0 1 0 17.00 
0 0 0 0 4 46.046   0 0 0 0 1 11.51

Por lo tanto el resultado es:


 c1   11.5mg / min 
c   11.5mg / min 
 2  
 c3   19.06mg / min 
   
c4   17.0mg / min 
 c5  11.51mg / min 

Se puede comprobar al sustituir la concentración de cualquier reactor obtenido en


sus respectivas ecuaciones. Por ejemplo para el reactor 1 para que los flujos de
entrada y salida sean iguales se tiene que cumplir con:
Q 01 C01 + Q 31 C3 = Q12 C1 + Q15 C1
Al despejar se obtenía
𝟔𝑪𝟏 − 𝑪𝟑 = 𝟓𝟎
y sustituyendo las concentraciones obtenidas se observa que el sistema está en
equilibrio.
𝐦𝐠 𝐦𝐠
𝟔 (𝟏𝟏. 𝟓 ) − (𝟏𝟗. 𝟎𝟔 ) = 𝟓𝟎
𝐦𝐢𝐧 𝐦𝐢𝐧

278
Aplicación 6. Ingeniería Eléctrica.

Un problema común dentro de la ingeniería eléctrica involucra la determinación de


corrientes y voltajes en varios puntos en circuitos de resistores. Estos problemas
se resuelven usando las leyes para corrientes y voltajes de Kirchhoff:
1.- En todos los nodos la suma de las corrientes de entrada es igual a la suma de
las corrientes de salida.
2.- En torno a todo ciclo cerrado, la suma algebraica del voltaje debe ser igual a la
suma algebraica de las caídas de voltaje o tensión.

Ejemplo:
Considere la red de la Fig. 4.

6 ohms 8 ohms

i1 i2
+ -
12 voltios 24 voltios
- i3 +
i1 i2

Fig. 4

El símbolo - + representa una batería. La corriente va del negativo al


positivo como la muestra la Fig. 4. El símbolo representa una resistencia.
Por la primera ley de Kirchhoff la suma de las corrientes en la unión B es
i1 − i2 − i3 = 0. igualmente en la unión A obtenemos la ecuación equivalente −i1 +
i2 + i3 = 0. De acuerdo con la segundo ley de Kirchhoff en el circuito cerrado ACBA
tenemos que 6𝑖1 + 4𝑖3 = 12, y en el circuito cerrado BDAB tenemos 8𝑖2 −
4𝑖3 = 24. El circuito DACB resulta de la ecuación 6𝑖1 + 8𝑖2 = 36.

279
Para encontrar las corrientes 𝑖1 , 𝑖2 , 𝑒 𝑖3 , se resolverá el siguiente sistema.
i1 − i2 − i3 = 0
6i1 + 4i3 = 12
8i2 − 4i3 = 24
6i1 + 8i2 = 36
Resolviendo el sistema, obtenemos los siguientes resultados.
 1 1 1 0   1 1 1 0   1 0 0.667 2 
6 0 4 12  0 6 10 12  0 1 1.667 2 
  
6 F  F
 
1 2  
1/ 6F
  2 
8 F2  F3

0 8 4 24  6 F  F 0 8 4 24  F  F
1 4
0 8 4 24  14 F2  F4
2 1

     
6 8 0 36  0 14 6 36  0 14 6 36 

1 0 0.667 2 1 0 0 2.3077  1 0 0 2.3077 


0 1 1.667 2  1/17.33F 0 1 1.667 2  1.667 F  F 0 1 0 2.7695
  
3
 3 2
 
0 0 17.33 8 0.667 F  F 0
3 1
0 1 0.461 17.33 F  F
3 4
0 0 1 0.461
     
0 0 17.33 8 0 0 17.33 8 0 0 0 0

Por lo tanto las soluciones son 𝑖1 = 2.3077, 𝑖2 = 2.7695, 𝑦 𝑖3 = −0.461, donde la


medida de la corriente es el amperio. El valor negativo de 𝑖3 , indica que el flujo de
la corriente es opuesto a la dirección escogida.

280
Aplicación 7. Ingeniería Eléctrica
Ejemplo:
Determinar las corrientes de la red presentada en la Fig. 5.
8 volts

4 ohms
i1

2 ohms
De la primera ley de Kirchhoff se obtiene:
A B

i2
𝑖1 − 𝑖2 + 𝑖3 = 0 (Nodo A)
3 ohms 2 ohms
−𝑖1 + 𝑖2 − 𝑖3 = 0 (Nodo B)

Por la segunda ley:


4𝑖1 + 2𝑖2 = 8 (Ciclo superior)
9 volts
2𝑖2 + 5𝑖3 = 9 (Ciclo Inferior)

i3
Fig. 5.

La red se puede representar mediante la matriz aumentada:

 1 1 1 0  1 1 1 0  1 1 1 0
 1 1 1 0  0 0 0 0  F  F 0 1 2.5 4.5 
 F F

1
 2  
2 4   
6 F  F
2 3

 4 2 0 8 4 F  F 1 3
0 6 4 8 1/ 2 F 2
0 6 4 8  (1/19) F3

     
 0 2 5 9 0 2 5 9 0 0 0 0
 1 1 1 0
0 1 2.5 4.5

0 0 1 1
 
 0 0 0 0

Al resolver por sustitución hacia atrás vemos que i1 = 1, i2 = 2, i3 = 1. Donde la


medida de la corrientes es el amperio.

281
Aplicación 8. Ingeniería de Tránsito.
Ejemplo:
En la sección del centro de cierta ciudad dos series de calles de un solo sentido
forman intersecciones, como se ilustra en la figura 6. El volumen de tránsito por
hora promedio que entra y sale de esta sección de la ciudad en horas pico está
indicado en el diagrama.
Determine la intensidad de tránsito entre cada una de las cuatro intersecciones.

450
310

610 x1
640
A D

x2
x4

520 x3 600
B C

480
390

Fig. 6
En cada intersección o crucero, el número de vehículos que entran debe ser el
mismo que el número de vehículos que salen de esa sección. Por ejemplo, en la
intersección A el número de automóviles que entran es 𝑥1 + 450 y el número de
vehículos que salen es 𝑥2 + 610. Por lo tanto
x1 + 450 = x2 + 610 (interseccion A)
De la misma manera
x2 + 520 = x3 + 480 (interseccion B)
x3 + 390 = x4 + 600 (interseccion C)
x4 + 640 = x1 + 310 (interseccion D)

282
Al despejar obtenemos las ecuaciones:
x1 − x2 = 160
x2 − x3 = −40
x3 − x4 = 210
x4 − x1 = −330

Al expresarlo en forma matricial y resolver por el método de Gauss – Jordán se


observa lo siguiente:

 1 1 0 0 160  1 1 0 0 160 
 0 
1 1 0 40  F  F 0 1 1 0 40  F  F
   1 4
 2
 1

 0 0 1 1 210  F F
0
2 4
0 1 1 210  F  F3 1

   
 1 0 0 1 330  0 0 1 1 210 
1 0 0 1 330  1 0 0 1 330 
0 1 1 0 40  F  F 0 1 0 1 170 
  3
 2
0 0 1 1 210  F  F 3 4
0 0 1 1 210 
   
0 0 1 1 210  0 0 0 0 0

El sistema es consistente y como hay un variable libre, hay muchas soluciones


posibles.
El diagrama de la circulación del tránsito no ofrece información suficiente para
determinar 𝑥1 , 𝑥2 , 𝑥3 , 𝑥4 en forma única. Si se conociera la intensidad de tránsito
entre un par cualquiera de intersecciones, se podría calcular fácilmente el tránsito
en las arterias restantes. Por ejemplo, si la intensidad de tránsito entre las
intersecciones C y D arroja un promedio de 200 automóviles por hora, entonces
𝑥4 = 200. Después se pueden calcular 𝑥1 , 𝑥2 , 𝑥3 en términos de 𝑥4 .

x1 = x4 + 330 = 530
x2 = x4 + 170 = 370 Automóviles.
x3 = x4 + 210 = 410

283
4.7. Ejercicios.

1.- Determinar en cada problema si la matriz esta en forma escalonada, en forma


escalonada reducida, en ambas formas o en ninguna.
1 5 0 6 0 1 6 0 7 0
0 1 0 0 3
0 1 0 1 2 7 9
2. 0 0 1 2
0 1 1 0 8 9
1.  3.  4.  
0 0 0 0 1 0 0 0 0 1 0 1 2 5
  0 1 0 8  
0 0 0 0 0 0 0 0 0 0

2.- Calcular la forma escalonada de las siguientes matrices.

1  6 0 0 3  2
0 0 1 3 2 0 10 12 2 100
4 7 
6. 0 0 1 0 7. 5 10 3 75 
1 0
5. 
0 0 0 1 5 8
  0 0 0 1 9 10 5 115
0 0 0 0 0 0

1  6 0 0 3  2
 1  3 4 7 0  1  3 7 1
7
8. 0 1 2 2 10. 0 1 4 0
0 1 0 4
9. 
0 0 0 1 5 8
0 0 1 5   0 0 0 1
 0 0 0 0 0 0

3.- Calcule la solución del sistema de ecuaciones lineales, por eliminación de Gauss
o de Gauss – Jordán.

11. 𝑥 + 2𝑦 − 𝑧 = −2 12. 25𝑥 + 8𝑦 − 9𝑧 = 1 13. 10𝑥 + 10𝑦 + 20𝑧 = 5


𝑥+𝑦+𝑧 =0 𝑥 + 𝑦 + 8𝑧 = 2 25𝑥 + 𝑦 − 21𝑧 = 3
2𝑥 + 9𝑦 − 𝑧 = 1 5𝑥 + 6𝑦 − 𝑧 = −2 5𝑥 − 2𝑦 + 15𝑧 = 2

14. 𝑥 + 𝑦 + 𝑧 = 4 15. 𝑥 − 2𝑦 + 𝑧 = 1 16. 𝑥 + 𝑦 + 10𝑧 = 0


−𝑥 + 2𝑦 + 2𝑧 = 4 𝑦 + 2𝑧 = 5 𝑦+𝑧 =1
4𝑥 + 𝑦 − 3𝑧 = 1 𝑥 + 𝑦 + 3𝑧 = 8 𝑥−𝑦−𝑧 =5

284
17. 𝑥 + 𝑦 + 6𝑧 = 3 18. 𝑥 + 𝑦 + 𝑧 = 2 19. −𝑦 − 𝑧 = 2
𝑥 + 𝑦 + 3𝑧 = 3 2𝑥 − 3𝑦 + 2𝑧 = 4 −𝑥 + 𝑦 + 3𝑧 = 5
𝑥 + 2𝑦 + 4𝑧 = 7 4𝑥 + 𝑦 − 3𝑧 = 1 𝑥+𝑦−𝑧=6

4.- Resuelva cada sistema de ecuaciones lineales siguientes por cualquier método.

20. 𝑥 + 2𝑦 − 3𝑧 = −5 21. 𝑥 + 𝑦 − 𝑧 − 𝑤 = 6 22. 5𝑥 + 6𝑦 − 3𝑧 = 1


−2𝑥 − 4𝑦 − 6𝑧 = 10 2𝑥 − 𝑦 + 3𝑧 + 𝑤 = 0 10𝑥 + 10𝑦 + 10𝑧 = 10
3𝑥 + 7𝑦 − 2𝑧 = −13 𝑥−𝑦−𝑧−𝑤 =1 22𝑥 + 14𝑦 + 𝑧 = −6
2𝑥 + 2𝑦 + 2𝑧 + 𝑤 = 5

5.- Un doctor recomienda a un paciente que tome diariamente 50 g de niacina, 50


de riboflavina y 50 de tiamina, para corregir su deficiencia vitamínica. Al buscar en
su botiquín, el paciente encuentra píldoras vitamínicas de tres marcas. En la tabla
aparecen las cantidades de las vitaminas por píldora.

VitaMax Vitron Vitaplus


Niacina (mg) 5 10 15
Riboflavina (mg) 15 20 0
Tiamina (mg) 10 10 10

¿Cuántas pastillas de cada marca debe tomar cada día para cumplir con la receta?

6.- Una fábrica de muebles produce mesas, sillas y aparadores de madera. Cada
mueble requiere tres pasos de producción; corte de madera, armado y acabado.
La cantidad de horas necesarias para cada operación y mueble se ve en la
siguiente tabla.

285
Mesa Silla Armario
Corte (h) ½ 1 1
Armado (h) ½ 1½ 1
Acabado 1 1½ 2

Los obreros de la fábrica pueden dedicar 300 horas al corte, 400 al armado y 590
al acabado, cada semana laboral.

¿Cuántas mesas, sillas y armarios deben producirse para ocupar todas las horas
laborales disponibles?

286
4.8. Aplicación de la regla de Cramer

Ahora se resolverá el problema del paracaidista visto anteriormente, por la


regla de Cramer.

Se tiene el sistema de ecuaciones

70 1 0 a 636
A = [ 60 −1 1 ] X = [T ] B = [518]
40 0 −1 R 307

Evaluando los determinantes que aparecen en la regla de Cramer:

70 1 0
|D| = |60 −1 1 | = (70 + 40) − (−60) = 170
40 0 −1

636 1 0
|Dx | = |518 −1 1 | = (636 + 307) − (−518) = 1461
307 0 −1

70 636 0
|Dy | = |60 518 1 | = (−36260 + 25440) − (21490 − 38160) = 5850
40 307 −1

70 1 636
|Dz | = |60 −1 518| = (−21490 + 20720) − (−25440 + 18420) = 6250
40 0 307

Aplicando la regla de Cramer para llegar a la solución:

|Dx | 1461
x= = = 8.5941
|D| 170
|𝐷𝑦 | 5850
𝑦= = = 34.4117
|𝐷| 170

|𝐷𝑧 | 6250
𝑧= = = 36.7647
|𝐷| 170

Nos podemos dar cuenta que llegamos a la misma solución.

287
4.9. Aplicaciones de la matriz inversa

Aplicación 1. Ingeniería Civil.

Para la armadura estáticamente determinada de la fig.1(a) y (b): establézcanse la


matriz estática [𝐴], determínese la matriz [𝐴−1 ]; luego resuélvase la matriz de
fuerzas internas {F} de la matriz {P} conocida empelando la ecuación
{F}= [𝐴−1 ]{𝑃}.

9k

20´

18k

3 @ 15´= 45´

(a)

P2 P4

F2
P1 P3

F1
F7 F8 F3
F9

P6 P8
R1 F4 P5 F5 P7 F6
P9

R2
R3
(b) Fig. 1

288
Solución. La matriz estática [A] es simplemente una compilación de las
ecuaciones de estática. Por ejemplo, usando ∑ Fx = 0 y ∑ Fy = 0 para el diagrama
de cuerpo libre del nudo U1 tenemos:

P1 = 0.6 F1 − 1.0F2 − 0.6F8

P2 = 0.8 F1 + 1.0F7 + 0.8F8

F 1 2 3 4 5 6 7 8 9

P
1 + - -
0.6 1.0 0.6
2 + + +
0.8 1.0 0.8
3 + -
1.0 0.6
4 + +
0.8 1.0
5 + -
1.0 1.0
6 -
1.0
7 + - +
1.0 1.0 0.6
8 - - 1.0
0.8
9 + +
0.6 1.0
[A]9X9

Cada columna de la matriz [𝐴−1 ] muestra simplemente las fuerzas en las barras
debidas a una carga unitaria P.

289
Calculando la matriz inversa:

0.6 1 0 0 0 0 0 0.6 0 1 0 0 0 0 0 0 0 0
 0.8 0 0 0 0 0 1 0.8 0 0 1 0 0 0 0 0 0 0 

 0 1 0.6 0 0 0 0 0 0 0 0 1 0 0 0 0 0 0
 
 0 0 0.8 0 0 0 0 0 1 0 0 0 1 0 0 0 0 0
 0 0 0 1 1 0 0 0 0 0 0 0 0 1 0 0 0 0  
(1/ 0.6) F
 1

  0.8 F  F 1 2

 0 0 0 0 0 0 1 0 0 0 0 0 0 0 1 0 0 0
 0 0 0 0 1 1 0 0.6 0 0 0 0 0 0 0 1 0 0
 
 0 0 0 0 0 0 0 0.8 1 0 0 0 0 0 0 0 1 0
 0 0 0.6 0 0 1 0 0 0 0 0 0 0 0 0 1
 0 0

 1 1.666 0 0 0 0 0 1.0 0 1.666 0 0 0 0 0 0 0 0


0 1.333 0 0 0 0 1 1.6 0 1.333 1 0 0 0 0 0 0 0 

0 1 0.6 0 0 0 0 0 0 0 0 1 0 0 0 0 0 0
 
 0 0 0.8 0 0 0 0 0 1 0 0 0 1 0 0 0 0 0 F2  F3
 
0 0 0 1 1 0 0 0 0 0 0 0 0 1 0 0 0 0  1.666 F2  F1
 
0 0 0 0 0 0 1 0 0 0 0 0 0 0 1 0 0 0
0 0 0 0 1 1 0 0.6 0 0 0 0 0 0 0 1 0 0
 
0 0 0 0 0 0 0 0.8 1 0 0 0 0 0 0 0 1 0
0 0 0 0 0 0 0 0 1
 0 0.6 0 0 1 0 0 0 0

1 0 1.0 0 0 0 0 1 0 1.666 0 1.666 0 0 0 0 0 0 


0 1 0.6 0 0 0 0 0 0 0 0 1 0 0 0 0 0 0 

0 1.333 0 0 0 0 1 1.6 0 1.333 1 0 0 0 0 0 0 0
 
0 0 0.8 0 0 0 0 0 1 0 0 0 1 0 0 0 0 0
….
0 0 0 1 1 0 0 0 0 0 0 0 0 1 0 0 0 0
 
0 0 0 0 0 0 1 0 0 0 0 0 0 0 1 0 0 0
0 0 0 0 1 1 0 0.6 0 0 0 0 0 0 0 1 0 0
 
0 0 0 0 0 0 0 0.8 1 0 0 0 0 0 0 0 1 0
0 0 0 0 0 0 0 1
 0 0.6 0 0 1 0 0 0 0 0

Procediendo de la misma manera, llegamos a la matriz siguiente:

290
[A−1 ] =

1 0 0 0 0 0 0 0 0 0.555 0.833 0.555 0.4166 0 0.833 0 0.4166 0 


0 1 0 0 0 0 0 0 0 0.333 0.25 0.666 0.5 0 0.25 0 0.5 0 

0 0 1 0 0 0 0 0 0 0.555 0.4166 0.555 0.833 0 0.4166 0 0.8333 0 
 
0 0 0 1 0 0 0 0 0 0.666 0.5 0.666 0.25 1 0.5 1 0.25 1
0 0 0 0 1 0 0 0 0 0.666 0.5 0.666 0.25 0 0.5 1 0.25 1
 
0 0 0 0 0 1 0 0 0 0.333 0.25 0.333 0.5 0 0.25 0 0.5 1
0 0 0 0 0 0 1 0 0 0 0 0 0 0 1 0 0 0
 
0 0 0 0 0 0 0 1 0 0.555 0.4166 0.555 0.416 0 0.4166 0 0.416 0 
0 0.444 0.333 0.333 0 0.666 0 
 0 0 0 0 0 0 0 1 0.444 0.333 0

1 - 10.0
1 0.0 2 + 1.5
2 0.0 3 - 12.5
3 + 9.0 4 + 15.0
{P}9X1 4 0.0 F 9 x1   A1  P  5 + 15.0
5 0.0 6 + 7.5
6 - 18.0 7 + 18.0
7 0.0 8 - 12.5
8 0.0 9 + 10.0
9 0.0

291
Aplicación:

Suponga que un equipo de tres paracaidistas están conectados por una cuerda
ligera mientras van en caída libre a una velocidad de 5 m/s (Fig. 1). Calcule la
tensión en cada sección de la cuerda y la aceleración del equipo, con los siguientes
datos:

Paracaidista Masa, kg. Coeficiente de


arrastre kg/s
1 70 10
2 60 14
3 40 17

Solución: Los diagramas de cuerpo libre para cada paracaidista están


representados en la fig. 2. Sumando las fuerzas en la dirección vertical y usando la
segunda ley de Newton da como resultado un conjunto de tres ecuaciones lineales
simultáneas:

m1 g − T − c1 v = m1 a

m2 g + T − c2 v − R = m2 a

m3 g − c3 v + R = m3 a

Estas ecuaciones tienen 3 incógnitas: a, T y R. Después de sustituir los valores


conocidos, las ecuaciones se pueden expresar en forma matricial como:

𝑚
(𝑔 = 9.8 )
𝑠2

70a + T = 636

60a − T + R = 518

40a − R = 307

70 1 0 a 636
A = [60 −1 1 ] X = [T] B = [518]
40 0 −1 R 307

292
Fig. 2

293
La solución de la ecuación matricial AX = B es:

X = A−1 B

Por lo tanto, encontrando la matriz inversa:

 70 1 0 1 0 0 
A1   60 1 1 0 1 0  
(1/ 70) F
60 F  F 1
 1
2

 40 0 1 0 0 1

 1 0.0142 0 0.0142 0 0   1 0.0142 0 0.0142 0 0 


0 1 0.538 0.4615 0.538 0   0 1.857 1 0.857 1 0 
40 F1  F3

 7/13 F2 

0 0.571 1 0.571 0 1  40 0 1 0 0 1

Se procede de la misma forma, hasta llegar a la matriz inversa que es:

1 1 1
170 170 170
10 7 7
A−1 = 17

17

17
, al hacer x = A−1 B, obtenemos la solución del sistema:
4 4 13
[ 17 − ]
17 17

1 1 1
170 170 170
10 7 7 636 8.5941
A−1 = − − [518] = [ 34.41 ]
17 17 17 307 36.7647
4 4 13
[ 17 −
17 17]

El resultado es a=8.5941 m/s2, T=34.41 N y R=36.7647N

294
Ejercicios Propuestos

4.1 –. Aplique la regla de Cramer para resolver los siguientes sistemas.

4a.- 2𝑥 − 𝑦 = −9 4b.- 6𝑥 + 12𝑦 = 33 4c.- 𝑥 − 6𝑦 = 3


𝑥 + 2𝑦 = 8 4𝑥 + 7𝑦 = 20 3𝑥 + 2𝑦 = 1

1 1
4d.- 𝑥 + 𝑦 = 1 4e.- 0.4𝑥 + 1.2𝑦 = 0.4 4f.- 10𝑥 − 17𝑦 = 21
2 3
1 1 3
𝑥− 𝑦=− 1.2𝑥 + 1.6𝑦 = 3.2 20𝑥 − 31𝑦 = 39
4 6 2

4k.- 𝑥 − 𝑦 + 2𝑧 = 0 4l.- 5𝑥 − 3𝑦 + 𝑧 = 6 4m.- 2𝑥 − 𝑦 = 5


3𝑥 + 𝑧 = 11 4𝑦 − 6𝑧 = 22 5𝑥 + 3𝑧 = 19
−𝑥 + 2𝑦 = 0 7𝑥 + 10𝑦 = −13 4𝑦 + 7𝑧 = 17

4.2.- Realice el mismo ejercicio del paracaidista, pero ahora use cinco
paracaidistas con las siguientes características.

Paracaidista Masa, Coeficiente de arrastre


kg. kg/s
1 50 12
2 80 12
3 60 14
4 70 16
5 90 10

El paracaidista tiene una velocidad de 9 m/s.

295
296
1.-Resolver el siguiente sistema de ecuación

𝑥+𝑦+𝑧+𝑤 =2

2𝑥 − 𝑦 + 2𝑧 − 𝑤 = −5

3𝑥 + 2𝑦 + 3𝑧 + 4𝑤 = 7

𝑥 − 2𝑦 − 3𝑧 + 2𝑤 = 5

1 1 1 1 2 1 1 1 1 2

2 −1 2 −1 −5 1 −2 −3 2 5

3 2 3 4 7 2 −1 2 −1 −5

1 −2 −3 2 5 3 2 3 4 7

1 1 1 1 2 1 1 1 1 2

0 −3 −4 1 3 0 −3 −4 1 3

0 −3 0 −3 −9 0 0 4 −4 − 12

0 −1 0 1 1 0 0 4/3 2/3 0

1 1 1 1 2

0 −3 −4 1 3

0 0 1 −1 −3

0 0 0 2 4

X Y Z W

 2𝑤 = 4 → 𝑤 = 2
𝑧 − 𝑤 = −3 → 𝑧 − 2 = −3

 → 𝑧 = −3 + 2 = −1
−3𝑦 − 4𝑧 + 𝑤 = 3 → −3𝑦 − 4(−1) + 2 = 3

−3𝑦 + 4 + 2 = 3 → −3𝑦 + 6 = 3 → −3𝑦 = −3


297
 𝑦=1
𝑥+𝑦+𝑧+𝑤 =2→𝑥+1−1+2=2

𝑥+2 =2 →𝑥 = 2−2 = 0

 𝑥=0

2.- Resolver el siguiente sistema de ecuación

x + y − z − 2w = −2 1 1 −1 −2 −2

𝑥 + 𝑦 − 2𝑧 − 𝑤 = 2 1 1 −2 −1 2

𝑥 + 2𝑦 − 𝑧 − 2𝑤 = −3 1 2 −1 −2 −3

𝑥+𝑦−𝑧−𝑤 =0 1 1 −1 −1 0

1 1 −1 −2 −2 1 1 −1 −2 −2

0 0 −1 1 4 0 1 0 0 −1 *-1

0 1 0 0 −1 0 0 1 −1 −4

0 0 0 1 2 0 0 0 1 2

1 0 −1 −2 −1 1 0 0 −3 −5

0 1 0 0 −1 0 1 0 0 −1

0 0 1 −1 −4 0 0 1 −1 −4

0 0 0 1 2 0 0 0 1 2 *3,1

1 0 0 0 1

0 1 0 0 −1

0 0 1 0 −2

0 0 0 1 2

x y z w

298
 𝑤=2
 𝑧 = −2
 𝑦 = −1
 𝑥=1

3.- 2𝑥 + 𝑦 − 𝑧 = −1 2 1 −1 0 −1

2𝑦 + 𝑧 − 𝑤 = 7 0 2 1 −1 7

2𝑥 −𝑧 = −3 2 0 −1 0 −3

2𝑦 −𝑤 = 6 0 2 0 −1 6

2 1 −1 0 −1 2 1 −1 0 −1
1 1 7
0 2 1 −1 7 0 1 2
−2 2

1 1 3
0 −1 0 0 −2 0 0 −
2 2 2

0 2 0 −1 6 0 0 −1 0 −1

2 1 −1 0 −1
1 1 7
0 1 −
2 2 2

0 0 1 −1 3

0 0 0 −1 2

X y z w

 −𝑤 = 2 → 𝑤 = −2
𝑧 − 𝑤 = 3 → 𝑧 − (−2) = 3

 𝑧=1
1 1 7
𝑦+ 𝑧− 𝑤 =
2 2 2
1 7 3 7
𝑦+ +1= →𝑦+ =
2 2 2 2
 𝑦=2

299
2𝑥 + 2 − 1 + 0 = −1

2𝑥 + 1 = −1 → 2𝑥 = −2

 𝑥 = −1

4.- Resolver el siguiente sistema de ecuaciones por el método de cramer

2x − y + z = 3 2 −1 1 3 1 −3 1

3x + 2y − 2z = 1 3 2 −2 1 2 −1 1

x − 3y + z = −2 1 −3 1 −2 3 2 −2

1 −3 1 1 −3 1

0 5 −1 0 5 −1
14
0 11 −5 0 0 −
5

14
D = (1)(5) (− ) = −14
5

X1

3 −1 1 3 −1
|1 2 −2| 1 2 = (6 − 4 − 3) − (−4 + 18 − 1) = −1 − 13 = −14
−2 −3 1 −2 −3
−14
X1 = =1
−14

X2

2 3 1 2 3
|3 1 −2| 3 1 = (2 − 6 − 6) − (1 + 8 + 9) = −10 − 18 = −28
1 −2 1 1 −2

−28
X2 = =2
−14

300
X3

2 −1 3 2 −1
|3 2 1 | 3 2 = (−8 − 1 − 27) − (6 − 6 + 6) = −36 − 6 = −42
1 −3 −2 1 −3

−42
X3 = =3
−14

5.- resolver el siguiente sistema de ecuaciones lineales:

x + 2y − 3z = −5

−y + 2z = 5

−2x + y = −11

3x + z = 13

1 2 −3 −5 1 2 −3 −5

0 −1 2 5 0 −1 2 5

−2 1 0 − 11 0 5 −6 − 21

3 0 1 13 0 −6 10 28

1 2 −3 −5 1 2 −3 −5

0 −1 2 5 0 −1 2 5

0 0 4 4 0 0 4 4

0 0 −2 −2 0 0 0 0

x y z

4z = 4 → 𝐳 = 𝟏

−𝑦 + 2𝑧 = 5 → −𝑦 + 2(1) = 5

−𝑦 = 5 − 2 → −𝑦 = 3

301
𝒚 = −𝟑

x + 2y − 3z = −5

𝑥 + 2(−3) − 3(1) = −5

𝑥 − 6 − 3 = −5 → 𝑥 − 9 = −5

𝒙=𝟒

6.- resolver el siguiente sistema de ecuaciones lineales:

x + 2y − 3z + 2w = −1

−x − 2y + 2z − 5w = 1

2x + 4y − 5z + 7w = −2

1 2 −3 2 −1 1 2 −3 2 −1

−1 −2 2 −5 1 0 0 −1 −3 0

2 4 −5 7 −2 0 0 1 3 0

1 2 −3 2 −1

0 0 −1 −3 0

0 0 0 0 0

x y z w

302
−𝑧 − 3𝑤 = 0 → −𝑧 = 3𝑤

𝒛 = −𝟑𝒘

𝒙 + 𝟐𝒚 − 𝟑𝒛 + 𝟐𝒘 = −𝟏

𝑥 + 2𝑦 − 3(−3𝑤) + 2𝑤 = −1

𝑥 + 2𝑦 + 9𝑤 + 2𝑤 = −1

𝑥 + 2𝑦 + 11𝑤 = −1

𝑥 = −1 − 2𝑦 − 11𝑤

7.- Resolver el siguiente sistema de ecuaciones lineales:

2x + 3y + z = 2

2x − y + z = 1

4x − 2y + 2z = 4

2 3 1 2 *-1,-2 2 3 1 2 2 3 1 2

2 −1 1 1 0 −4 0 −1 0 −4 0 −1

4 −2 2 4 0 −8 0 0 0 1 0 0 *4

2 3 1 2 2 3 1 2 2 0 1 2

0 0 0 −1 0 1 0 0 *-3 0 1 0 0

0 1 0 0 0 0 0 −1 0 0 0 −1

No tiene solución x y z

303
8.- Resolver el siguiente sistema de ecuaciones lineales:

2x + 2y − 5z = 7

3x − 3y + 6z = 9

x + y − 2z = 3

2 2 −5 7 1 1 −2 3

3 −3 6 9 3 −3 6 9

1 1 −2 3 2 2 −5 7

1 1 −2 3

0 −6 12 0

0 0 −1 1

x y z

−𝑧 = 1 → 𝐳 = −𝟏

−6𝑦 + 12𝑧 = 0 → −6𝑦 + 12(−1) = 0

−6𝑦 − 12 = 0 → −6𝑦 = 12

𝒚 = −𝟐

x + y − 2z = 3

x − 2 − 2(−1) = 3

x−2+2= 3→x+0=3

𝑥=3

304
9.- 3x-2y=16
2x+y=6

SOLUCIÓN:

3  2 16 1/ 3 1  2 / 3 16 / 3  2 1  2 / 3 16 / 3 
2 1 6   2 6   0 7 / 3  14 / 3 3 
  1

1  2 / 3 16 / 3 1  2 / 3 16 / 3 1 0 4 
0
 1  2  1/ 7  0 1  2  2 / 3  0 1  2

x=4; y=-2

10.- 4x + 3y= - 1
2x – y = 2

SOLUCIÓN:

4 3  1 1  1 / 2 1  4 1  1 / 2 1
2 1 2  1/ 2   1  0  5 1/ 5 
   4 3  5

1  1 / 2 1 1 0 1 / 2
0  1 1/ 2 0 1 1 
 1 

X=1/2; Y=-1

11.- 4x +y = -1
x + 3y = 8

SOLUCIÓN:

4 1  1 0  11  33 1/11 1 3 8 1 0  1
1 3 8  4  1 3 8   0 1 3  3  0 1 3 
   

X=-1; Y=3

305
12.- 2x + 3y = - 5
3x + 4y = 6

SOLUCIÓN:

2 3  5 1/ 2 1 3/ 2  5 / 2 3 1 3 / 2  5 / 2
3 0
3 4
  6   4  6     1 / 2 3 / 2  2 

1 3/ 2  5 / 2 1 0 2
0 0 1  3
 1  3  3 / 2 

X=2; Y=-3

306

También podría gustarte